SlideShare a Scribd company logo
1 of 163
Download to read offline
See discussions, stats, and author profiles for this publication at: https://www.researchgate.net/publication/267151510
Advanced Financial Accounting - Volume 01
Book · January 2014
DOI: 10.13140/2.1.3194.1126
CITATIONS
0
READS
38,436
1 author:
Some of the authors of this publication are also working on these related projects:
Corporate Governance Practices, Customer centric approach and Firm performance View project
Corporate Governance Mechanism and Working capital Management:a study of Selected Listed
Companies in Sri Lanka View project
Alagathurai Ajanthan
University of Jaffna
12 PUBLICATIONS 19 CITATIONS
SEE PROFILE
All content following this page was uploaded by Alagathurai Ajanthan on 21 October 2014.
The user has requested enhancement of the downloaded file.
1 | P a g e
A.Ajanthan Joint Venture & Consignment Accounts
CONSIGNMENT ACCOUNTS
Consignment
The sales activity of any business can be organized in different ways. With the customers
spread all over, the business entity cannot afford to have only minimum selling points nor can
it have its own resources to have the outlets all over. The business volumes cannot be limited
in any case. The core competence of a manufacturing company is to produce a good quality
product. It creates a network of its own outlets, dealers, commission agents, institutions etc to
distribute its products efficiently and effectively. Thus the selling may be handled directly
through own salesmen or indirectly through agents. In case of direct selling, the company
usually has depots all over. The stocks are transferred to these depots and from their finally
sold to ultimate customers. This involves huge expenses and problems of maintaining the
same on a permanent basis. Hence, the firm could appoint agents to whom stocks will be
given. These agents distribute the products to ultimate customers and receive commission
from the manufacturer. One such way of indirect selling is selling through consignment
agents. The relationship between consignor and consignee is that of Principal-Agent
relationship.
Consignment takes place where goods are transferred from the owner (consignor) to an agent
(consignee) for the purpose of sale by the consignee on behalf of the consignor. It is
important to understand that the relationship of principal (consignor) and agent (consignee)
exists. Because of this agency relationship, ownership of the goods does not transfer to the
consignee.
The consignee, as the selling agent, is entitles to a commission for selling the goods;
expenses may be incurred by both parties; and periodically or on completion of the
consignment, settlement is effected between the parties. If any goods remain unsold then they
are generally returned to the consignor.
Consignment is a fairly common commercial transaction, perhaps more common than many
people may think. Examples include:
ï‚· A manufacturer supplies stock of a new product on consignment to a local distributor.
ï‚· A primary producer forwards produce on consignment to an agent.
ï‚· A car sales yard in a prominent position may accept motor vehicles on consignment
from other motor dealers or from the general public.
2 | P a g e
A.Ajanthan Joint Venture & Consignment Accounts
Main Terms of Consignment Trade
Consignment: The transfer of goods by one party called a consignor, to another party called
the consignee, to be sold by the latter on behalf of the former. The ownership of the goods is
retained by the consignor while the possession of the goods is transferred to the consignee.
The consignment is outward consignment for the person who sends the goods and an
inward consignment for the person who receives the goods for sale.
Consignor: The party who sends the goods to agents for sale, e.g., a manufacturer or whole
seller.
Consignee: The party to whom the goods are sent for sale.
Ordinary Commission or Consignee’s Remuneration: When the goods are sold by the
consignee, he is paid a commission for his services at a fixed rate on the proceeds of the
goods sold by him. In addition to this commission, he is to be reimbursed for all expenses
incurred by him in connection with the consignment sales. Usually these expenses are in the
nature of dock charges, custom duties, carriage, godown rent, advertisement, insurance of the
goods while in his possession etc.
Del Credere Commission: This is additional commission payable to the consignee for taking
over additional responsibility of collecting money from customers. Usually the consignor
advises the consignee to sell the goods consigned to him for cash only, because if such goods
are sold on credit by the consignee and if any amount becomes irrecoverable from the debtors
the loss will fall upon the consignor as the consignee acted as an agent only in effecting the
sales, he does not become responsible for any debts. But sometimes an arrangement is made
between the consignor and the consignee whereby the later guarantees payment and
undertakes responsibility for bad debts. For this the consignee receives an additional
commission known as ‗‗del credere commission’’ on the total sales. When del-credere
commission is given to the consignee, the consignee will make payment to the consignor,
whether he himself receives the payment or not from the purchaser(s).
Overriding Commission: This type of commission is allowed to the consignee in addition to
the normal commission (as distinct from del credere commission). The idea seems to be to
provide addition incentive to the consignee for the purpose of creating market for new
products.
Account Sales: This is a summary of the transactions of the consignee. It is a means of
conveying information to the consignor and shows the gross proceeds of sale of the goods,
3 | P a g e
A.Ajanthan Joint Venture & Consignment Accounts
expenses incurred by the consignee, commission due and the net amount owing to the
consignor.
The following example shows a specimen of an account sale (Account sales of 100 Sony
Radios consigned to Mayuran Traders, Colombo by Alagu Traders, Jaffna.
Particulars Amount (LKR)
Sale Proceeds:
100 Radios sold at Lkr 9000 each
Less: Expenses:
Freight
Carriage
Godown rent and selling expenses
Less: Commission @10% on sale proceeds
(18,00,000 × 10/100)
Less: Advance (Bank Draft)
Balance due to Alagu traders remitted
5,000
2,100
4,300
18,00,000
(11,400)
17,88,600
(1,80,000)
16,08,600
(2,00,000)
14,08,000
E & O.E. For Mayuran Traders
Colombo MAYURAN
31st
January, 2010 Managing Partner
Advance against Consignment: Until the goods are sold by the consignee, he is not
indebted to the consignor and is not expected to pay for them. This results in a part of the
consignor's Capital being locked up for a period. To overcome his difficulty, the consignee
often remits a sum of money in advance to the consignor. This may be done in the form of an
acceptance of a bill of exchange drawn by the consignor on the Consignee or a simple bank
draft. An advance is readily sent against consignment by the consignee to the consignor when
the consignment goods have become popular in the consignee‘s place.
Pro-forma Invoice: When goods are consigned to an agent they are generally accompanied
by a document called a ‗Pro-forma invoice’ giving indication of the price of the goods at
which the consignee ought to sell the goods. Pro-Forma Invoice is a statement which is
similar to that of an invoice, but it is called proforma because it does not make the consignee
responsible to pay the amount named therein.
4 | P a g e
A.Ajanthan Joint Venture & Consignment Accounts
The consignor generally mentions a higher price than his cost so that consignee does
not know the profit of the consignor.
Difference between Invoice and Account sales
Account sales Invoice
Prepared by the consignee. Prepared by the seller.
All expenses and commission are deducted in
account sales.
In invoice, expenses are added but discount
and commission are deducted.
All expenses incurred by the consignee are
borne by the consignor.
After sale, expenses are paid by the buyer.
The relationship between two parties remains
as principal and agent.
The relationship between two parties is that
of debtor and creditor.
Operating Cycle of Consignment Arrangement
i. Goods are sent by consignor to the consignee
ii. Consignee may pay some advance or accept a bill of exchange
iii. Consignee will incur expenses for selling the goods
iv. Consignee maintains records of all cash and credit sale.
v. Consignee prepares a summary of results called as Account sales
vi. Consignor pays commission to the consignee
Features of Consignment
The following are the salient features of consignment:
1. Objects: Goods are forwarded by the consignor to the consignee with an objective of
sale at a profit.
2. Ownership: In consignment, the consignee does not buy the goods. He merely
undertakes to sell them on behalf of the consignor. Hence, the ownership in the goods
remains with consignor till it is sold by the consignee.
3. Relationship: The relationship between the consignor and the consignee is that of a
principal and an agent, and not of a debtor and creditor. An agent becomes in debited
for amounts realized on behalf of the principal.
4. Risk: The consignor should bear all the risks connected with the goods until it is sold.
5 | P a g e
A.Ajanthan Joint Venture & Consignment Accounts
5. Expenses: As consignment is not a sale, whatever the consignee does is on behalf of
the consignor. Thus, the consignor should reimburse all legitimate expenses incurred
by the consignee for selling and receiving the goods.
6. Stock of goods: Any stock remaining unsold with the consignee belongs to the
consignor.
7. Commission: The consignee agrees to sell the goods for an agreed rate of
commission. He is therefore, allowed to deduct his commission due from the sale
proceeds.
8. Possession: The goods will be in the possession of the consignee until it is sold on
behalf of the consignor.
9. Repossession: The consignor can repossess the goods from the consignee at any time.
10. Profit or loss: Since the consignee acts on behalf of the consignor, the profit or loss on
sale of goods belongs to the consignor.
Difference between Sale and Consignment
1. When goods are sold by one to another, the property in the goods immediately passes
to the buyer, whereas when goods are sent on consignment, the property in the goods
remains with the consignor. Only the possession is transferred to the consignee.
2. When goods are sold by one to another, it becomes a relationship of a buyer and seller
or a debtor and a creditor between the two persons, whereas when goods are consigned
by one to another, it becomes a relationship of a principal and an agent between the
consignor and the consignee.
3. When goods are sold, the buyer cannot return the goods to the seller whereas when
goods are sent on consignment the goods are returnable, if they remain unsold.
4. The risk in the goods is not transferred to the consignee despite the transfer of
possession of goods. Any damage or loss to the goods is therefore borne by consignor.
But in the case of sale, the risk is immediately transferred to the buyer even when the
goods are still in the possession of the seller.
5. The expenses, in respect of freight, cartage, insurance, etc. Are met by the consignor in
a consignment transaction, but in the case of sale the expense are borne by the
purchaser unless otherwise provided in the agreement.
6. The transfer of possession (i.e. Delivery of goods) is essential in a consignment
transaction. In a sale, however, the goods may be delivered at a later date. The
consignee will be treated as a debtor only when goods or part of them have been sold by him.
6 | P a g e
A.Ajanthan Joint Venture & Consignment Accounts
But if goods remain unsold, the consignee will send them back to the Consignor and the
Consignor will pay the Consignee all the expenses he has incurred in keeping the goods in
safety and in attempting to push the goods in the market.
Expenses on Consignment
i. Non-recurring expenses: The expenses which do not arise repeatedly for a particular
consignment are called non-recurring expenses. Non-recurring expenses are incurred for
bringing goods to the godown of the consignee. Such expenses are generally incurred on
the consignment as a whole. The non-recurring expenses are incurred partly by the
consignor and partly by the consignee. The consignor usually incurs expenses, such as
packing, cartage, loading charges, freight, etc., on sending the goods to the consignee. But
the consignee usually incurs expenses, such as dock dues, customs duty, clearing charges,
etc., on receiving the goods from the consignor.
ii. Recurring expenses: The indirect expenses incurred repeatedly on the same consignment
are called recurring expenses. Recurring expenses are incurred after the goods have
reached the consignee‘s place or godown. Advertising, discount on bills, commission on
collection of cheques, travelling expenses of salesman, bad debts, etc., are some examples
of recurring expenses incurred by the consignor. On the other hand, godown rent, godown
insurance, sales promotion, etc., are the examples of recurring expenses incurred by the
consignee.
Accounting for Consignment Business
The consignor and consignee keep their own books of accounts. The consignor may send
goods to many consignees. Also, a consignee may act as agent for many consignors. It is
appropriate that both of them would want to know profit or loss made on each consignment.
Books of the Consignor
The transactions relating to ach consignment are recorded in such a way that the profit or loss
of each consignment can be ascertained separately. It requires the preparation of a special
account known as consignment account. A consignment account is a nominal account
prepared to find out the profit or loss of a consignment. The account is debited with the cost
of goods sent, expenses incurred by the consignor and consignee, and the commission due to
the consignee. But the account is credited with the amount of sales affected and also with
closing stock, if any. The balance of this account is either profit or loss.
7 | P a g e
A.Ajanthan Joint Venture & Consignment Accounts
In addition to the consignment account, the consignor also prepares the personal account of
the consignee to ascertain the amount due by the consignee. This account is debited with the
amount of sales affected by the consignee and credited with the amount of any advance
received from him, expenses incurred by him and commission payable on sales. The balance
in this account is the amount due by the consignee. Let us see the entries in the books of
consignor as well as consignee.
Situations Consignor’s Books
On sending goods Consignment A/c …………...Dr
Goods Sent on Consignment….……Cr
On expenses for sending goods (by the consignor) Consignment A/c ……..….....Dr
Cash/ Bank A/c………………....…. Cr
On an advance made by the consignee Cash / Bank / Bills Receivable A/c
……Dr
Consignee‘s Personal A/c…...…..Cr
Bills received from the consignee discounted with
the bank
Bank A/c ……..….....Dr
Discount A/c ….........Dr
Bills Receivable A/c……….……Cr
On expenses incurred by consignee Consignment A/c …………...Dr
Consignee‘s Personal A/c……….…Cr
On sales made by the consignee Consignee‘s Personal A/c .….Dr
Consignment A/c………………..... Cr
For consignee‘s commission Consignment A/c …………...Dr
Consignee‘s Personal A/c…….…... Cr
Goods returned by the consignee Goods Sent on Consignment...Dr
Consignment A/c ……………….....Cr
Bad debts incurred (when a consignee is entitled to
del credere commission, no entry for bad debts is
to be passed as such a loss is to be borne by the
consignee himself. Otherwise, the loss on account
of bad debts should be borne by the consignor.
Consignment A/c …………....Dr
Consignee‘s Personal A/c…….…... Cr
Remittance by the consignee in full settlement Cash / Bank / Bills Receivable A/c
……Dr
8 | P a g e
A.Ajanthan Joint Venture & Consignment Accounts
Consignee‘s Personal A/c…….…... Cr
Profit or loss on consignment
(a) If there is a profit
(b) If there is a loss
Consignment A/c …………....Dr
Profit and loss A /c………………...Cr
Profit and loss A /c…………Dr
Consignment A/c …………….…....Cr
Closing entry for goods sent on consignment Goods Sent on Consignment…Dr
Trading A / c………………….…….Cr
On closing stock/ unsold Stock with the consignee Consignment Stock A/c ........Dr
Consignment A/c………………..… Cr
The Consignment account in the books of consignor will ultimately show the net profit or loss on
account of consignment business. It must be noted that a separate consignment account must be
opened for different agents. This will enable him to know profit or loss on each consignment.
Books of the Consignee
Situations Consignee’s Books
Goods received from the consignor No Entry
Expenses incurred by the consignor No Entry
Advance made by the consignee Consignor‘s Personal A/c...Dr
Bank / Cash / Bills Payable A/c………Cr
Bill discounted by the consignor with the bank No Entry
Sales of goods by the consignee Cash A / c (cash sales)….…Dr
Consignment debtors A / c
(credit sales)……………...Dr
Consignor‘s Personal A/c.....................Cr
Expenses incurred by the consignee Consignor‘s Personal A/c...Dr
Cash/ Bank A/c…….…………....….. .Cr
Commission due to the consignee Consignor‘s Personal A/c...Dr
Commission A/c……………..…..…...Cr
Return of goods to the consignor No Entry
Payment received from debtors Cash/ Bank A/C…………. Dr
Consignment debtors A/C……..……..Cr
Bad debts incurred
9 | P a g e
A.Ajanthan Joint Venture & Consignment Accounts
(a) In case consignee does not get del
credere commission, all bad debts have
to be borne by the consignor himself.
(b) In case del credere commission is paid
to the consignee, bad debts are to be
borne by him.
When the bills payable accepted in favor of the
consignor is met on the due date
Consignor‘s Personal A/c...Dr
Consignment debtors A/C…….……...Cr
Bad debts A /c…………….Dr
Consignment debtors A/C…………....Cr
Bills payable A /c………...Dr
Bank A/c…….………………….….....Cr
Unsold stock in possession of the consignee No Entry
Profit or loss on consignment No Entry
*Note: The discount on bills may be accounted for in one of two ways;
ï‚· As a normal operating expenses item and charged against the profit and loss account;
or
ï‚· As a special expense item related to the consignment and therefore charged to the
consignment account.
The method of accounting depends on whether the advance is
interpreted as a method of financing the business generally or whether it is regarded as
a transaction particularly related to the consignment activity.
Format of Consignment Account
LKR LKR
To Consignment stock (opening
balance if any)
xxx By Consignee‘s Personal Account
(Amount of gross proceeds (sales)
realized by the Consignee)
Xxxx
To Goods Sent on Consignment xxxx By Goods Sent on Consignment
(Difference in cost of goods sent and
the proforma Invoice price)
Xx
To Cash/bank (Expenses incurred
by the consignor)
xx By Abnormal loss (Whether insured or
not)
Xxx
To Consignee's Personal Account
(Expenses paid by the Consignee–
total amount) (Commission,
including del-credere payable to the
xx By Goods sent on Consignment
(Returned by the Consignee)
Xx
10 | P a g e
A.Ajanthan Joint Venture & Consignment Accounts
consignee)
To Stock Reserve (Difference in the
value of closing stock marked at
Pro-forma invoice or loaded price
& cost price)
xx By Stock Reserve (Difference between
the cost and pro-forma invoice price on
the opening balance of consignment)
Xx
To Goods Sent on Consignment
(Difference between cost price and
Pro-forma invoice price on the
goods returned by the consignee)
xx By General Profit and Loss Account
(For consignment loss)
xx
To General Profit and Loss Account
(For Consignment profit)
xxx
Illustration: 1
Aju stores of Jaffna consigned on 1st
January, 2010, 50 cases of goods at Lkr.200 each to
Riyash Traders of Warakkapola for sale on commission at 10% on gross sales. Aju stores
paid Lkr.500 for packing, freight and insurance. Riyash Traders took delivery of the goods on
11th
January, 2010, after accepting a 15 days bill for Lkr. 5,000 and paid Lkr. 150 for
carriage. They sold 40 cases of goods @ Lkr. 250 and balance for Lkr. 260 each. Their sales
expenses amounted to Lkr. 200. On 31st
January, 2005, Riyash Traders forwarded an account
sale together with a draft for the balance.
Prepare account sales rendered by Riyash Traders and also give
journal entries and ledger accounts in the books of Aju stores and Riyash Traders.
Solution:
Account sales of 50 cases of goods received and sold on behalf of Aju stores, Jaffna.
Particulars Amount(LKR)
Sale Proceeds:
40 cases sold at Lkr 250 each
10 cases sold at Lkr 260 each
Less: Expense:
Carriage
Sales expenses
Commission @ 10%
Net proceeds
10,000
2,600
150
200
1,260
12,600
(1,610)
10,990
11 | P a g e
A.Ajanthan Joint Venture & Consignment Accounts
Less: Advance (Bill)
Balance sent by Draft
(5,000)
5,990
Books of Aju Stores (Consignor)
Journal Entries
Description Dr
(LKR)
Cr
(LKR)
1. Consignment to Warakkapola A/c
Goods sent on Consignment A/c
(Sent goods on consignment to Riyash Traders, Warakkapola)
10,000
10,000
2. Consignment to Warakkapola A/c
Bank A/c
(Expenses incurred on the Consignment)
500
500
3. Bill receivable A/c
Riyash Traders A/c
(Advance received from the Agent in the form of Bill)
5,000
5,000
5. Consignment to Warakkapola A/c
Riyash Traders A/c
(paid carriage and sales expenses by consignee)
350
350
6. Bank A/c
Bills receivable A/c
(The bill met on due date)
5,000
5,000
4. Riyash Traders A/c
Consignment to Warakkapola A/c
(Gross sale proceeds as per Account Sales)
12,600
12,600
7. Consignment to Warakkapola A/c
Riyash Traders A/c
(Commission on gross sales payable @ 10%)
1,260
1,260
8. Consignment to Warakkapola A/c
Profit and Loss A/c
(Transferred profit on consignment to profit and loss A/c)
490
490
9. Bank A/c
Riyash Traders A/c
(Amount received in draft along with account sales)
5,990
5,990
12 | P a g e
A.Ajanthan Joint Venture & Consignment Accounts
10. Goods sent on Consignment A/c
Trading A/c
(Goods sent on consignment A/c closed by transfer to trading A/c)
10,000
10,000
Ledgers
Consignment to Warakkapola Account
Dr Cr
Goods sent on Consignment A/c
Bank A/c (expenses)
Riyash Traders A/c
Carriage 150
Sales expenses 200
Riyash Traders: Commission
P & L A/c (Transfer)
10,000
500
350
1,260
490
Riyash Traders A/c
(Sale proceeds)
12,600
12,600 12,600
Riyash Traders A/c
Dr Cr
Consignment to Warakkapola A/c 12,600 Bill Receivable A/c
Consignment to Warakkapola A/c
Consignment to Warakkapola A/c
Bank A/c
5,000
350
1260
5,990
12,600 12,600
Bill Receivable A/c
Dr Cr
Riyash Traders A/c 5,000 Bank A/c 5,000
Goods sent on Consignment Account
Dr Cr
Trading A/c (Transfer) 10,000 Consignment to Warakkapola A/c 10,000
13 | P a g e
A.Ajanthan Joint Venture & Consignment Accounts
Profit & Loss A/c
Dr Cr
Consignment to Warakkapola A/c 490
Books of Riyash Traders (Consignee)
Journal Entries
Description
Dr
(LKR)
Cr
(LKR)
1. Aju stores A/c
Bank / Cash A/c
(Paid expenses on the Consignment received)
350
350
2. Aju stores A/c
Bills payable A/c
(Acceptance of bill drawn against the consignment)
5,000
5,000
3. Bills payable A/c
Bank A/c
(The bill met on due date)
5,000
5,000
4. Bank A/c
Aju stores A/c
(Sales effected for the Consignment received)
12,600
12,600
5. Aju stores A/c
Commission A/c
(Commission receivable on the goods sold)
1,260
1,260
6. Aju stores A/c
Bank A/c
(Amount remitted as final settlement)
5,990
5,990
Ledgers
Aju Stores A/c
Dr Cr
Bank A/c (Expenses)
Bills payable A/c
Commission A/c
Bank A/c (amount remitted)
350
5,000
1,260
5,990
Bank A/c (Sale proceeds) 12,600
14 | P a g e
A.Ajanthan Joint Venture & Consignment Accounts
12,600 12,600
Bills payable A/c
Dr Cr
Bank A/c 5000 Aju stores A/c 5,000
Commission A/c
Dr Cr
Aju stores A/c 1,260
Unsold Stock at Balance Sheet Date
Where all consigned goods are not sold by the end of the consignor‘s financial year, it is
necessary to obtain an ‘account sales’ from the consignee. This is a document detailing
particulars of transactions and the quantity of unsold stock on hand at that date.
Unsold stock is valued to enable the profit on the consignment up to
balance sheet date to be ascertained, and included with revenue from other trading activities.
The basis for valuation of this stock is cost price unless deterioration or obsolescence requires
the adoption of net realizable value. Determination of cost price involves a consideration not
only of the original purchase price of the goods but also of any expenses in transporting the
goods to the place of sale-the consignee‘s store. Thus it is proper to include the following in
valuing unsold stock: (carriage and freight, loading charges, customs duty, clearing charges,
dock dues, carriage paid up to the godown and unloading charges).
ï‚· Purchase price
 Inward charges to the consignor‘s place of business
ï‚· Outward charges related to the dispatch to the consignee
ï‚· Inward charges incurred by the consignee
Expenses incurred by the consignee in selling the goods such as
advertisement, salesman‘s salaries and commission, storage, insurance against fire or theft are
not included in the valuation of unsold stock. These expenses do not relate to the goods
unsold and are recorded as marketing expenses. In other words it can be said that all direct
expense or all expenses made whether by the consignor or by the consignee in placing the
goods in a saleable condition (all expenses till the goods reach the godown of the consignee)
will be taken into account while valuing the closing stock. In most cases the amount of
15 | P a g e
A.Ajanthan Joint Venture & Consignment Accounts
expenses of both the consignor and the consignee to be added to purchase price is calculated
as a proportion of the total relevant expenses of the whole consignment.
The balance of consignment stock account is a current asset
(appears in the asset side of Balance Sheet). At the commencement of the next financial
period, consignment stock will be transferred to the consignment account, as a debit to
enable the profit or loss on the sale of the remainder of the consignment to be determined.
Illustration: 2
Suppose the Consignor sends to the Consignee, 2,000 Samsung mobile at Lkr.40 per unit and
pays Costa duty, Lkr.3, 000; marine insurance, Lkr.1, 500. The Consignee pays, at the time of
taking delivery, unloading charges of Lkr.500. The Consignee also pays godown rent Lkr.450
and advertisement Lkr.1, 500.if you assume that 400 Samsung mobile remain unsold, the
value of its will be calculated as follows;
LKR
400 Samsung mobile, i.e., 400 @ Rs.40 16,000
1/5th
of Lkr.3,000, Costa duty 600
1/5th
of Lkr.1,500, Marine Insurance 300
1/5th
of Lkr.500, unloading charges paid by the Consignee 100
Total value of unsold Stock 17,000
The rule regarding valuation is cost or market price whichever is lower. In the market price
of the unsold stock is more than Rs.17, 000, it will be valued at Rs.17, 000. If however, the
market price is less than Rs.17, 000, it will be valued at the market price. Any loss or
depreciation of stock should be duly taken into account.
The unsold stock valued in the above manner will now be brought into books by
passing an entry, as
Consignment Stock A/c ……………..Dr
Consignment A/c………………………….…. Cr
Note: If the pro-forma invoice was made out at a price higher than the cost, stock will also be
valued at invoice and not at cost. But it is wrong to show unsold stock in Balance Sheet at a
figure higher than the cost. Hence for the difference (i.e., difference between value of stock at
invoice price and value of stock at cost) reserve must be created, entry is as follows;
Consignment A/c ……………………..Dr
Stock Reserve A/c……………………….….. Cr
16 | P a g e
A.Ajanthan Joint Venture & Consignment Accounts
Illustration: 3
Y consigns goods to X valued at 8000 cost price. Expenses incurred by Y are: freight 40;
insurance 100; cartage 20.Commission is allowed at 5% on sales. An advance of 5000 is
made by the consignee. X incurs the following expenses: duty 80; cartage inward 40;
advertising 200; and cash sales amounted to 7600. At balance date one-quarter of the goods
are unsold. Calculate the value of unsold goods.
LKR
¼ th
of cost price (Lkr 8,000) 2,000
¼ th
of consignor‘s expenses [Lkr 160 (freight 40; insurance 100; cartage 20)] 40
¼ th
of consignee‘s relevant expenses [Lkr 120 (duty 80; cartage inward 40)]
Total value of unsold Stock
30
2,070
Illustration: 4
Ramu of Cochin consigned goods of the cost of Lkr.10000 to his agent, Ajith of Agra and
incurred Lkr.2000 for packing, forwarding and freight. Ajith took delivery of the goods after
spending Lkr.3000 for duty and clearing charges. He sold 3 / 4th
of the goods for Lkr.15000
for which he was entitled to a commission of 5%. His sales expenses amounted to Lkr.300.
Prepare consignment account after showing the valuation of unsold stock.
Solution:
Valuation of stock:
LKR
Cost of stock at pro forma invoice = 10,000 * ¼ 2500
Add: proportionate non-recurring expenses:
Incurred by Ramu 2,000
Incurred by Ramu 3,000
5,000 * ¼ 1,250
Value of stock 3,750
Consignment to Agra A/c
Dr Cr
Goods sent on Consignment A/c
Bank A/c (Packing charges)
Ajith A/c (Duty + selling charges)
Ajith A/c: Commission
P & L A/c (Transfer)
10,000
2,000
3,300
750
2,700
Ajith A/c
(Sale proceeds)
Stock on consignment
15,000
3,750
17 | P a g e
A.Ajanthan Joint Venture & Consignment Accounts
18,750 18,750
Del Credere Commission and Bad Debts
Sometimes the consignor allows an extra commission to the consignee in order to cover the
risk of collection from customer on account of credit sales which is known as Del Credere
Commission. Naturally, if debt is found to be irrecoverable the same must be borne by the
consignee. There will be no effect in the books of consignor. In short, credit sales will be
treated as cash sales to consignor. If no Del credere commission is given by the consignor to
the consignee, the amount of Bad debts must be borne by the consignor.
When there is no Del credere Commission;
In the books of consignor:
Consignment A/c………………..Dr
Consignee‘s A/c……………………Cr
In the books of consignee:
Consignor‘s A/c………………….Dr
Debtors A/c………………………...Cr
Illustration: 5
Prasana Furniture‘s, Palghat consigned 100 chairs of Lkr 800 each to their agent Sudharaka
Furniture‘s at Kelaniya for sale on commission at 5% on gross sale effected. Expenses at
Palghat were Lkr.1500 for carriage and Lkr.1000 for insurance. Sudharaka Furniture‘s took
delivery of the chairs after accepting a three-month bill for Lkr.40000 drawn against the
consignment, which the consignor discounted for Lkr.38000.
The consignees paid Lkr.150 for loading and unloading and Lkr. 600 for freight and
carriage. They sold 70 chairs @ Lkr.850 for cash and 10 chairs on credit @ Lkr. 1000.
A customer who bought two chairs became insolvent and nothing could be recovered
from him. The balance of debt was fully collected. The sales expense of Sudharaka
Furniture‘s amounted to Lkr. 360.
Required to prepare ledger accounts in the books of consignor and journal entries in
the books consignee
18 | P a g e
A.Ajanthan Joint Venture & Consignment Accounts
Solution:
Ledger accounts (in Prasana Furniture’s)
Consignment to Kelaniya A/c
Dr Cr
Goods sent on Consignment A/c
Bank A/c (expenses)
Sudharaka Furniture‘s A/c
Sudharaka Furniture‘s A/c
(Commission)
Sudharaka Furniture‘s A/c
(Bad debts)
80,000
2,500
1,110
3,475
2,000
Sudharaka Furniture‘s A/c:
Cash sales 59,500
Credit sales 10,000
Stock on consignment
Profit & loss A/c
(Loss transferred)
69,500
16,650
2,935
89,085 89,085
Sudharaka Furniture’s A/c
Dr Cr
Consignment to Kelaniya A/c 69,500 Bill Receivable A/c
Consignment to Kelaniya A/c
Consignment to Kelaniya A/c
Consignment to Kelaniya A/c
Balance c / d
40,000
1,110
3,475
2,000
22,915
69,500 69,500
Bill Receivable A/c
Dr Cr
Sudharaka Furniture‘s A/c 40,000
40,000
Bank A/c
Discount
38,000
2,000
40,000
Goods sent on Consignment Account
Dr Cr
Trading A/c (Transfer) 80,000 Consignment to Kelaniya A/c 80,000
19 | P a g e
A.Ajanthan Joint Venture & Consignment Accounts
Profit & Loss A/c
Dr Cr
Consignment to Kelaniya A/c 2935
Valuation of stock:
Number of chairs in stock = 100 – 80 = 20 chairs
LKR
Original cost of 20 chairs (800 * 20) = 16,000
Add: proportionate non – recurring expenses:
Incurred by consignor 2500
Incurred by consignee 750
3250 * (20 / 100) = 650
Stock value 16,650
*The entry for bad debt is:
Consignment A /c………………Dr
Sudharaka Furniture‘s A/c………………Cr
Books of Sudharaka Furniture’s
Journal Entries
Description
Dr
(LKR)
Cr
(LKR)
1. Prasana Furniture‘s A/c
Bank / Cash A/c
(Paid expenses on the Consignment received)
1,110
1,110
2. Prasana Furniture‘s A/c
Bills payable A/c
(Acceptance of bill drawn against the consignment)
40,000
40,000
3. Bank A/c
Debtors A/c
Prasana Furniture‘s A/c
(The amount of total sales)
59,500
10,000
69,500
4. Prasana Furniture‘s A/c
Debtors A/c
2,000
2,000
20 | P a g e
A.Ajanthan Joint Venture & Consignment Accounts
(The amount of bad debts)
5. Bank A/c
Debtors A/c
(The amount collected from debtors)
8,000
8,000
When there is Del credere Commission;
The consignee has to bear any amount of bad debts due to the insolvency of a debtor, if there
is a provision for payment of del credere commission. In such a situation, the consignor need
not make any entry in his book for bad debts. But the bad debt is a loss to the consignee.
Hence, it should be recorded in his book through the following entry:
Bad debt A/c……………..Dr
Debtors A/c……………………..Cr
Note: if there is any loss on account of non-payment of a customer on dispute regarding
quality of goods should be borne by consignor although there is a provision for dl credere
commission.
Illustration: 6
Amirtha Paints, Jaffna, consigned 500 tins of paints to Arvind Paints, Cochin at Lkr. 60. They
spent Lkr. 400 for packing and Lkr. 600 for freight and insurance, and drew against the
consignment a bill for the amount o 80% of the cost of goods sent. On getting the acceptance,
Amirtha Paints discounted the bill at a cost of Lkr. 1200.
Arvind Paints, Cochin sold 400 tins of paints at Lkr.80 of which 50 tins were on
credit. Their sales expenses amounted to Lkr. 300. They were to get a commission of 4% plus
2% del credere commission.
A customer who bought 10 tins of paints on credit became insolvent and only Lkr.
400 was realized from him in full settlement.
Prepare consignment account and consignee‘s account in the books of consignor, and
also show journal entries in the books of consignee.
Solution:
Ledger of Amirtha Paints
Consignment to Cochin A/c
Dr Cr
21 | P a g e
A.Ajanthan Joint Venture & Consignment Accounts
Goods sent on Consignment A/c
Bank A/c (expenses)
Arvind paints A/c
Arvind paints A/c
(Commission)
Profit & loss A/c
30,000
1000
300
1920
4980
Arvind paints A/c
Stock on consignment
32000
6200
89,085 89,085
Valuation of stock:
Original cost of stock 60*100 = Lkr. 6000
Proportionate expense of the consignor 1000 / 500 *100 = Lkr. 200
Value of stock = Lkr. 6200
Arvind Paints A/c
Dr Cr
Consignment to Cochin A/c 32,000 Bill Receivable A/c
Consignment to Cochin A/c
Consignment to Cochin A/c
Balance c / d
24,000
300
1,920
5,780
32000 32000
Journal of Arvin Paints, Cochin
(Consignee)
Description
Dr
(LKR)
Cr
(LKR)
1. Amirtha paints A/c
Bank / Cash A/c
(Paid expenses on the Consignment received)
300
300
2. Amirtha paints A/c
Bills payable A/c
(Acceptance of bill drawn against the consignment)
24,000
24,000
3. Bank A/c
Debtors A/c
Amirtha paints A/c
28,000
4,000
3,2000
22 | P a g e
A.Ajanthan Joint Venture & Consignment Accounts
(The amount of total sales)
4. Amirtha paints A/c
Commission A/c
(Effected the commission on sales)
1,920
1,920
5. Bank A/c
Bad debts A/c
Debtors A/c
(50% of the amount of Lkr. 800 realized in full settlement due from
a customer)
400
400
800
Invoicing Goods at a Price Higher than Cost
When the consignor does not want to reveal the actual cost of goods to the consignee, the
goods are invoiced at a price which is higher than the cost price. Such a price is known as
invoice price. In other words, invoice price is equal to the cost plus a certain amount of profit.
The difference between invoice price and the cost price is termed as loading.
In this connection, it should be noted that invoice price is not the same thing as
selling price. The invoice price is the price at which the consignor sends the goods to the
consignee. Selling price, on the contrary, is the price at which the consignee sells the goods to
the customers. It is to be observed that IP (invoice price) is higher than CP (cost price),
whereas SP (selling price) is higher than the CP as well as the IP. However, if the consignor
directs the consignee to sell the goods at invoice itself, then the SP and IP will be the same.
Loading
The amount of profit which is added to the cost in order to arrive at the invoice price is
known as loading. In other words, loading is the difference between the invoice price and the
cost price.
Loading = IP – CP
For example, the invoice price is Lkr. 5000 and the cost price is Lkr.3750. Calculate the
amount of loading.
Loading = IP – CP or Number of units * (IP per unit – CP per unit)
= 5000 – 3750
= Lkr. 1250
23 | P a g e
A.Ajanthan Joint Venture & Consignment Accounts
Items which involve loading
Usually loading is involved in all such items which are recorded at the invoice price in the
consignment account. Some such items are as follows:
1. Opening stock
2. Goods sent on consignment
3. Goods returned by the consignee
4. Closing stock
Adjustment on Loading
The usual adjustments required on loading are as follows:
1. Opening stock: It is always shown on the debit side of the consignment account.
Hence, the difference between the invoice price and the cost price of the stock will be
shown on the credit side of the consignment account through the following entry:
Stock reserve A/c…………………….Dr
Consignment A /c………………………….Cr
2. Goods sent on consignment: Such goods are shown on the debit side of the
consignment account. Thus the difference between invoice price and cost price of
goods sent on consignment will be shown on the credit side of the consignment
account through the following entry:
Goods sent on consignment…………Dr
Consignment A /c…………………………Cr
3. Goods returned by the consignee: The return of goods is shown on the credit side of
the consignment account. Therefore the adjustment for the loading will be made on
the debit side of consignment account through the following entry:
Consignment A /c……………………Dr
Goods sent on consignment…….………..Cr
4. Closing stock: it is shown on the credit side of consignment account. Hence, the
adjustment for the loading will be made on the debit side through the following entry:
Consignment A /c……………………Dr
Stock reserve A/c………………………… Cr
In practice, loading done at a fixed percentage of profit on cost bears a fixed relation with the
profit on invoice price of the goods.
24 | P a g e
A.Ajanthan Joint Venture & Consignment Accounts
For example, in case goods of the costs of Lkr.5000 are consigned at a profit of 25%
on cost, the invoice price of the product will be 5000 + 25% of 5000.
Invoice price of the product = 5000 + 1250 = Lkr. 6250
The same amount of loading is obtained on applying the percentage of profit on invoice price.
It is ascertained as follows:
Cost of goods is assumed to be 100.
Loading (profit) on cost = 25
Invoice price (100 + 25) = 125
Loading (profit) on invoice price = 25 / 125 =1 / 5 = 20%
Loading on the invoice price of Lkr. 6250
= 6250 *25 /15 = Lkr. 1250
Or
= 6250 *1 /5 = Lkr. 1250
Or
= 6250 *20 / 100 = Lkr. 1250
In the light of the above example, it is clear that 25% (1 / 4) of profit on cost of a
product is equal to 20% (1 / 5) of the invoice price of that product.
Illustration: 7
Ambika Electronics, Jaipur, consigned 1000 radios to Lakshmi Electronics, Agra, for sale on
commission of 5% including 1% del credere commission. The cost price of a radio was
Lkr.2400. But the invoice was made at Lkr. 3000. The expenses at Jaipur amounted to Lkr.
54000 and that at Agra before reaching the goods at godown was Lkr.46000.
Lakshmi Electronics sold 800 radios @ Lkr. 3200, the sales expenses being Lkr. 28000.
The consignee sent a draft for the amount due along with the
account sales. Give entries and accounts in the books of both the parties.
Books of Ambika Electronics
Journal Entries
Description
Dr
(LKR)
Cr
(LKR)
1. Consignment to Agra A/c
Goods sent on Consignment A/c
(Invoice price of radios sent on consignment)
3,000,000
3,000,000
25 | P a g e
A.Ajanthan Joint Venture & Consignment Accounts
2. Consignment to Agra A/c
Bank A/c
(Expenses incurred on the Consignment)
54,000
54,000
3. Consignment to Agra A/c
Lakshmi Electronics A/c
(paid carriage and sales expenses by consignee)
74,000
74,000
4. Lakshmi Electronics A/c
Consignment to Agra A/c
(The amount of sales made by the consignee)
2,560,000
2,560,000
5. Consignment to Agra A/c
Lakshmi Electronics A/c
(Commission on gross sales payable @ 5 %)
128,000
128,000
6. Stock on consignment A/c
Consignment to Agra A/c
(The amount of unsold goods in stock)
620,000
620,000
7. Goods sent on Consignment A/c
Consignment to Agra A/c
(Loading on goods sent on consignment)
600,000
600,000
8. Consignment to Agra A/c
Stock reserve A/c
(Loading on consignment stock)
120,000
120,000
9. Consignment to Agra A/c
Profit and Loss A/c
(Transferred profit on consignment to profit and loss A/c)
404,000
404,000
10. Bank A/c
Lakshmi Electronics A/c
(Amount received in draft along with account sales)
2,358,000
2,358,000
Consignment to Agra A/c
Dr Cr
26 | P a g e
A.Ajanthan Joint Venture & Consignment Accounts
Goods sent on Consignment A/c
Bank A/c (Expenses)
Lakshmi Electronics A/c (Exp.)
Lakshmi Electronics A/c
(Comm.)
Stock Reserve A/c
(Loading in stock)
P & L A/c (Transfer)
3,000,000
54,000
74,000
128,000
120,000
404,000
Lakshmi Electronics A/c
(Sale proceeds)
Consignment Stock A/c
Goods sent on Consignment A/c
(Loading in goods sent)
2,560,000
620,000
600,000
3,780,000 3,780,000
Lakshmi Electronics A/c
Dr Cr
Consignment to Agra A/c 2,560,000 Consignment to Agra A/c
Consignment to Agra A/c
Bank
74,000
128,000
2,358,000
2,560,000 2,560,000
Goods sent on Consignment Account
Dr Cr
Consignment to Agra A/c
Trading A/c
600,000
2,400,000
3,000,000
Consignment to Agra A/c 3,000,000
3,000,000
Consignment Stock Account
Dr Cr
To Consignment to Agra A/c 620,000 Balance c /d 620,000
620,000 620,000
Valuation of stock:
Invoice price of stock (Lkr. 3000*200 units) 600,000
Add: proportionate non-recurring expenses:
Incurred by consignor 54,000
Incurred by consignee 46,000
27 | P a g e
A.Ajanthan Joint Venture & Consignment Accounts
100,000 = 100,000 / 1,000*200 = 20,000
Value of stock 620000
Stock reserve = loading in one unit * number of units in stock
= 600 * 200 = Lkr. 120,000
Stock Reserve A/c
Dr Cr
Balance c /d 120,000 Consignment to Agra A/c 120,000
120,000 120,000
Books of Lakshmi Electronics
Journal Entries
Description
Dr
(LKR)
Cr
(LKR)
1. Ambika Electronics A/c
Bank / Cash A/c
(Expenses met by the consignor)
74,000
74,000
3. Bank A/c
Ambika Electronics A/c
(The amount of total sales)
2,560,000
2,560,000
4. Ambika Electronics A/c
Commission A/c
(Effected the commission on sales)
128,000
128,000
5. Ambika Electronics A/c
Bank A/c
(Remitted the amount with account sales)
2,358,000
2,358,000
Ambika Electronics A/c
Dr Cr
Bank A/c
Commission A/c
Bank A/c
74,000
128,000
2,358,000
Bank A/c 2,560,000
2,560,000 2,560,000
28 | P a g e
A.Ajanthan Joint Venture & Consignment Accounts
Illustration: 8
Riyash of Warahapolai sent to his agent, Ashan of Puttalam, 500 articles costing Lkr.15 per
article at an invoice price of Lkr.20 per article. The following payments were made by Riyash
in this connection:
Freight and carriage Lkr. 450
Miscellaneous expenditure Lkr. 50
Ashan sent a bank draft for Lkr.3, 000 as an advance against the
Consignment. Ashan sold 300 articles at a flat rate of Lkr.28 per article and sent an Account
Sales showing deduction for storage charges Lkr.50, insurance Lkr.100 and his Commission
of 3% plus 2% Del Credere on gross sale proceeds, and remitted the amount due on
consignment. Ashan also informed Riyash that 50 articles were damaged in transit and thus
they were valued at Lkr.550. Record the above transactions in the books of the consignor and
consignee using cost price basis.
Solution:
Books of Riyash (Consignor)
Journal
Description
Dr
(LKR)
Cr
(LKR)
1. Consignment to Puttalam A/c
Goods sent on Consignment A/c
(500 articles sent to Ashan, Agent, and Cost being Lkr.15 per article).
7,500
7,500
2. Consignment to Puttalam A/c
Bank A/c
(Expenses incurred on the Consignment)
Freight & Carriage Lkr. 450
Miscellaneous Exp. Lkr. 50 500
500
500
3. Bank A/c
Ashan A/c
(Advance received from the Agent in the form of Bank Draft.)
3,000
3,000
4. Ashan A/c
Consignment to Puttalam A/c
(Sales affected by Ashan as per Account Sales.)
8,400
8,400
5. Consignment to Puttalam A/c
Ashan A/c
570
570
29 | P a g e
A.Ajanthan Joint Venture & Consignment Accounts
(Expenses incurred by Ashan Lkr.150 and Commission due to him,
Lkr.420 (5% of Lkr.8, 400).
6. Bank A/c
Ashan A/c
(Amount due from the consignee received.)
4,830
4,830
7. P & Loss A/c
Consignment to Puttalam A/c
(Abnormal Loss on 50 damaged Articles)
*250
*250
8. Consignment Stock A/c
Consignment to Puttalam A/c
(Value of stock unsold at Puttalam) Lkr.
150, goods articles, @ Lkr.15 2,250
Add: Freight and carriage( 450/500*150) 135
Miscellaneous expenditure(50/500*150) 15
50 damaged articles 550
2,950
2,950
2,950
9. Consignment to Puttalam A/c
Profit & Loss Account
(Profit on consignment transferred to Profit & Loss Account)
3,030
3,030
10. Goods sent on Consignment A/c
Trading A/c
(Goods sent on consignment A/c closed by transfer to trading
Account)
7,500
7,500
Working Notes:
Calculation of Abnormal Loss
LKR
Cost @ Lkr.15*50 articles 750
Proportionate Expenses:
Freight and carriage (Lkr.450/500*50) 45
Miscellaneous expenditure (Lkr.50/500*50) 5 50
800
30 | P a g e
A.Ajanthan Joint Venture & Consignment Accounts
Damaged 50 articles have been have been valued at Lkr.550 Thus, there is a loss of Lkr.250*,
(800- 550). Such a loss would be recorded as follows;
Profit and Loss A/c ……………………..Dr 250
Consignment A/c……………………………....Cr 250
Ledgers
Consignment to Puttalam Account
Dr Cr
Goods sent on Consignment A/c
Bank A/c (expenses)
Ashan A/c
Expenses 150
Commission 420
P & L A/c (Transfer)
7,500
500
570
3,030
Ashan A/c
(Sale proceeds)
Profit & Loss A/c
(Abnormal Loss)
Consignment Stock A/c
8,400
250
2,950
11,600 11,600
Ashan A/c
Dr Cr
To Consignment to Puttalam A/c 8,400 Bank A/c
Consignment to Puttalam A/c
Bank A/c
3,000
570
4,830
8,400 8,400
Bank A/c
Dr Cr
Ashan A/c
Ashan A/c
3,000
4,830
Consignment to Puttalam A/c 500
Goods sent on Consignment Account
Dr Cr
Trading A/c (Transfer) 7,500 Consignment to Puttalam A/c 7,500
31 | P a g e
A.Ajanthan Joint Venture & Consignment Accounts
Profit & Loss A/c
Dr Cr
Consignment to Madras A/c 250 Consignment to Puttalam A/c 3,030
Books of Ashan (Consignee)
Journal
Description
Dr
(LKR)
Cr
(LKR)
1. Riyash A/c
Bank A/c
(Advance sent to the Consignor against consignment)
3,000
3,000
2. Riyash A/c
Bank A/c
(Expenses incurred on the Consignment on behalf of Riyash)
Storage 50
Insurance 100 150
150
150
3. Bank A/c
Riyash A/c
(Sale of 300 articles @ Rs.28 each out of the Consignment.)
8,400
8,400
4. Riyash A/c
Commission A/c
(5% Commission on Sales made on behalf of Riyash; 3%
Commission + 2% Del Credere Com.)
420
420
5. Riyash A/c
Bank A/c
(Amount due to Riyash remitted).
4,830
4,830
Ledgers
Riyash A/c
Dr Cr
Bank A/c (Advance)
Bank A/c (Expenses)
Commission A/c
Bank A/c (amount remitted)
3,000
150
420
4,830
Bank A/c (Sale proceeds) 8,400
32 | P a g e
A.Ajanthan Joint Venture & Consignment Accounts
8,400 8,400
Bank A/c
Dr Cr
Riyash A/c 8,400 Riyash A/c
Riyash A/c
Riyash A/c
3,000
150
4,830
Commission A/c
Dr Cr
Riyash A/c 420
Illustration: 9
Riyash of Warahapolai sent to his agent, Ashan of Puttalam, 500 articles costing Lkr.15/- per
article at an invoice price of Lkr.20 per article. The following payments were made by
Riyash in this connection:
Freight and carriage Lkr. 450
Miscellaneous expenditure Lkr. 50
Ashan sent a bank draft for Lkr.3, 000 as an advance against the
Consignment. Ashan sold 300 articles at a flat rate of Lkr.28 per article and sent an Account
Sales showing deduction for storage charges Lkr.50, insurance Lkr.100 and his Commission
of 3% plus 2% Del Credere on gross sale proceeds, and remitted the amount due on
consignment. Ashan also informed Riyash that 50 articles were damaged in transit and thus
they were valued at Lkr.550. Record the above transactions in the books of the consignor and
consignee using invoice price basis.
Solution:
Books of Riyash (Consignor)
Journal
Description
Dr
(LKR)
Cr
(LKR)
1. Consignment to Puttalam A/c
Goods sent on consignment A/c
(500, articles consigned at an invoice price of Lkr.20 each (cost
10,000
10,000
33 | P a g e
A.Ajanthan Joint Venture & Consignment Accounts
Lkr.15)
2. Consignment to Puttalam A/c
Bank A/c
(Expenses incurred on the Consignment)
Freight & Carriage Lkr. 450
Miscellaneous Exp. Lkr. 50 500
500
500
3. Bank A/c
Ashan A/c
(Advance received from the Agent in the form of Bank Draft.)
3,000
3,000
4. Ashan A/c
Consignment to Puttalam A/c
(Sales affected by Ashan as per Account Sales.)
8,400
8,400
5. Consignment to Puttalam A/c
Ashan A/c
(Expenses incurred by Ashan Lkr.150 and Commission due to him,
Lkr.420 (5% of Lkr.8, 400).
570
570
6. Bank A/c
Ashan A/c
(Amount due from the consignee received.)
4,830
4,830
7. P & Loss A/c
Consignment to Puttalam A/c
(Abnormal Loss on 50 damaged Articles)
250
250
8. Consignment Stock A/c
Consignment to Puttalam A/c
(Value of stock unsold at Puttalam) Lkr.
150, goods articles, @ Lkr.20 3,000
Add: Freight and carriage( 450/500*150) 135
Miscellaneous expenditure(50/500*150) 15
50 damaged articles 550
3,700
3,700
3,700
9. Goods sent on Consignment A/c
Consignment to Puttalam A/c
Excess amount included in invoice price of articles sent to Puttalam
2,500
2,500
34 | P a g e
A.Ajanthan Joint Venture & Consignment Accounts
(Rs.5 each) credited on consignment A/c)
10. Consignment to Puttalam A/c
Stock Reserve A/c
[Reserve credited equal to excess amount above cost (Lkr.5 per
articles*(200-50)]
750
750
11. Consignment to Puttalam A/c
Profit and Loss A/c
(Transfer of Profit on Consignment)
3,030
3,030
12. Goods sent on Consignment A/c
Trading A/c
(Goods sent on Consignment A/c closed by transfer to Trading A/c)
7,500
7,500
Ledgers
Consignment to Puttalam A/c
Dr Cr
Goods sent on Consignment A/c
Bank A/c (expenses)
Ashan A/c
Expenses 150
Commission 420 570
Stock Reserve A/c
P & L A/c (Transfer)
10,000
500
570
750
3,030
Ashan A/c
(Sale proceeds)
Profit & Loss A/c
(Abnormal Loss)
Consignment Stock A/c
Goods sent on Consignment A/c
8,400
250
3,700
2,500
14,850 14,850
Goods sent on Consignment Account
Dr Cr
Consignment to Puttalam A/c
Trading A/c
2,500
7,500
10,000
Consignment to Puttalam A/c 10,000
10,000
Stock on Consignment Account
Dr Cr
35 | P a g e
A.Ajanthan Joint Venture & Consignment Accounts
To Consignment to Puttalam A/c 3,600 Consignment to Puttalam A/c
Balance sheet (Transfer)*
750
2,850
3,600 3,600
*In the Balance Sheet the stock on consignment will be shown at Rs.2, 850 [(Lkr.3, 000 –
Reserve (Lkr.750)]
Expenses on consignment to be borne by consignee
In case expenses connected o a particular consignment at the place of the consignee is to be
borne by him, he will be given only commission. In such a case, the expense met by the
consignee is not recovered or reimbursed. But the consignee should treat it as his business
expense, and is to be transferred to his profit and loss account. However, there will be no
entry in the books of consignor for the expenses met by consignee.
Illustration: 10
Pradap Traders consigned goods of the cost of Lkr. 30000 to their agent, Sancha Agencies
Uduppiddy, at a profit of 20% on cost. Consignee was allowed a commission of 8% on gross
sales for which he would bear all the expenses at his place.
Pradap traders spent Lkr. 1500 for freight and got an acceptance for Lkr. 15000 from
the consignee. Sancha Agencies paid Lkr. 600 for advertisement and Lkr. 400 for sales
expenses. They sold ¾ th
of the goods at a profit of 33 1/3% on original cost of it.
Prepare consignment account the books of Pradap Traders and show journal entries in
the books of Sancha Agencies.
Solution:
In the Books of Pradap Traders
Consignment to Uduppiddy Account
Dr Cr
Goods sent on Consignment A/c
Bank A/c (expenses)
Sancha Agencies A/c (Commission)
Stock Reserve A/c
P & L A/c (Transfer)
36,000
1,500
2,400
1,500
3,975
Sancha Agencies A/c
(Sale proceeds)
Consignment Stock A/c
Goods sent on Consignment A/c
(Loading)
30,000
9,375
6,000
14,850 14,850
36 | P a g e
A.Ajanthan Joint Venture & Consignment Accounts
Working Notes:
1. Loading on goods sent:
Invoice price – cost price (36,000 – 30,000) = 6,000
Or
36,000 * 20 / 100 = 6,000
2. Value of stock:
Stock at invoice price, 36,000 * 1 / 4 = 9,000
Add: proportionate expenses of consignor 1,500 * 1 / 4 = 375
Value of stock = Lkr.9375
3. Stock reserve:
Stock reserve = invoice price of stock – cost price of stock
= [36,000*1/4] – [30,000*1/4]
= 9,000 – 7,500
= Lkr. 1500
Or
= 9,000*20/120
= Lkr. 1500
Books of Sancha Agencies (Consignee)
Journal Entries
Description
Dr
(LKR)
Cr
(LKR)
1. Pradap Traders A/c
Bills payable A/c
(Accepted the bill drawn against the consignment)
15,000
15,000
2. Advertisement A/c
Sales expenses A/c
Bank A/c
(Expenses incurred by consignee cannot be recovered from
consignor)
600
400
1,000
3. Bank A/c
Pradap Traders A/c
(The amount of sales)
30,000
30,000
4. Pradap Traders A/c 2,400
37 | P a g e
A.Ajanthan Joint Venture & Consignment Accounts
Commission A/c
(8% Commission on Sales)
2,400
Losses on Consignment
In case the goods sent on consignment are lost or damaged in transit or otherwise, the loss is
that of the consignor and not of the consignee. Accordingly the consignor will have to make
the entries for such loss. There are two types of losses which may arise in case of a
consignment transaction, viz., Normal Loss and Abnormal Loss.
Normal Loss
Normal loss is natural, unavoidable and inherent in the nature of goods or commodities sent
on consignment (due to evaporation, leakage & breaking the bulk into pieces). This type of
loss is a part of the cost of the consignment, so the consignor does not make separate entry
for such a loss. However, the normal loss has to be taken into consideration while valuating
the unsold consignment stock in the hand of the consignee. Since normal loss is a charge
against gross profit. No additional adjustment is required for this purpose. Moreover, the
same is a part of cost of goods, when valuation of unsold stock is made in case of
consignment account the quantity of such loss (not the amount) should be deducted from the
total quantity of the goods received by the consignee in good condition
The accounting treatment of normal loss is to charge the total cost of the goods to the
remaining goods after the normal loss. In other words, the value of the unsold stock is
calculated in proportion to the total cost of the goods consigned.
Total cost of the goods sent
Value of unsold stock = × Unsold quantity
Units of Goods sent – Normal losses (units)
…………….. (1)
Or
Unsold quantity
Value of unsold stock = × Total value of goods sent
Good quantity received by consignee
……………… (2)
38 | P a g e
A.Ajanthan Joint Venture & Consignment Accounts
Illustration: 11
Suppose 10,000 tons of coal is dispatched. The cost of 1 tons of coal is Lkr.80 and the freight
incurred is Lkr.36, 000. To the Consignor the total cost comes to Lkr.8, 36,000. In the nature
of coal some shortage is unavoidable. Suppose the Consignee receives only 9,500 tones. It is
legitimate to say that the cost is Lkr.8, 36,000 for 9,500 tons. In that case the Consignor can
properly say that the cost of 1 tons of coal is Lkr.8, 36,000/9500 or Lkr 88. If 2,000 tons of
coal is left unsold with the Consignee, the value of stock will be 2,000×88 is equal to Lkr.1,
76,000.
Illustration: 12
From the following particulars ascertain the value of unsold stock on consignment.
LKR
Goods sent (1,000 kgs) 20,000
Consignor‘s expenses 4,000
Consignee‘s non-recurring expenses 3,000
Sold (800 kgs) 40,000
Loss due to natural wastage (100 kgs)
Solution:
Value of unsold stock ` LKR
Total cost of goods sent 20,000
Add: Consignor‘s expenses 4,000
Non-recurring expenses 3,000
Cost of 900 kgs (1,000 kgs – 100 kgs) 27,000
∴Value of unsold stock 100 kgs (1,000 – 800 – 100) will be;
27,000
× 100 kgs
(1000 kgs-100 kgs)
= Lkr .3, 000 …………….. (1)
100 kgs
× 27,000
39 | P a g e
A.Ajanthan Joint Venture & Consignment Accounts
900 kgs
= Lkr .3, 000 …………….. (2)
Illustration: 13
Mr. Achchu Consigned to Mr. Kajan 10,000 kgs of flour, costing Lkr.33, 000. He spent
Lkr.880 as forwarding charges. 12% of the Consignment was lost in weighing and handling.
Mr. Kajan sold 8,200 kgs of flour at Lkr.6 per kg, his selling expenses being Lkr.3, 300 and
Commission 5% on sales. Prepare the Consignment Account.
Ledger of Mr. Achchu
(Consignment Account)
Dr Cr
Goods sent on Consignment A/c
Bank (forwarding Charges)
Mr. Kajan A/c
Selling Expenses 3,300
Commission 2,460
(@5% on Rs.49,200)
P & L A/c (Transfer)
33,000
880
5,760
11,870
Mr. Kajan A/c
[ (Sale proceeds, 8,200×6]
Consignment Stock A/c
49,200
2,310*
51,510 51,510
Working Notes:
1. Calculation of Closing Stock: Kgs
Total quantity of flour consigned 10,000
Less: Normal Loss 12% 1,200
Sales 8,200 (9,400)
Closing Stock 600
2. Valuation of Closing Stock:
Total Cost of the goods sent + non recurring expenses
× Closing Stock (units)
Units of Goods sent – Normal losses (units)
40 | P a g e
A.Ajanthan Joint Venture & Consignment Accounts
Lkr.33, 000 + Lkr.880
× 600
10,000 - 1,200
= Lkr 33880/8800*600
= Lkr 2,310*
Abnormal Loss
It arises due to abnormal factors or circumstances such as fire, theft pilferage, sabotage,
negligence, inefficiency, etc. Before ascertaining the result of the consignment, value of
abnormal loss should be adjusted. The method of calculation is similar to the method of
calculating unsold stock. Sometimes insurance company admits the claim in part or in full.
The same should also be adjusted against such abnormal loss. This loss is calculated by
adding proportionate direct expenses incurred by the consignor and the consignee as the case
may be to the original cost of the goods. The accounting entry is:
Abnormal Loss A/c…………………….Dr
Consignment A/c……………………………..Cr
In case the stock is insured, the amount of claim admitted by the insurance company should
be reduced from the abnormal loss and only the net loss amount should be debited to
abnormal loss or P&L A/c, the entry will be:
Insurance Company A/c (with the amount of claim admitted)
Profit and Loss (Abnormal Loss A/c) (with the amount of loss)
Consignment A/c (with the amount of Total abnormal loss)
Dr
Dr
Cr
The procedure for calculating the abnormal loss and the valuation of the remaining stock is
summarized as under:
(i) Calculation of Abnormal loss:
Add:
Cost of goods lost xxxx
Proportionate expenses of the goods lost xxxx
xxxx
Less: Any amount of claim (xxxx)
(If any received from the insurance company) xxxx
41 | P a g e
A.Ajanthan Joint Venture & Consignment Accounts
(ii) Valuation of Closing Stock
(1) Cost of the goods
Closing Stock
× Cost of total goods consigned
Total goods consigned
Add: Proportionate Non-recurring (direct) expenses incurred before the loss
Closing Stock
× Expenses incurred before the loss
Total goods consigned
Add: Proportionate expenses (Direct only) incurred after the loss
Quantity unsold
× Expenses incurred after the loss
(Total quantity sent - Goods lost)
Illustration: 14
Aju smart of Jaffna dispatched 1,000 shirts at Lkr.700 each to Mohan Bros of Colombo, the
consignors paid freight Lkr.7, 500, cartage Lkr.500 and insurance Lkr.2, 500. Mohan Bros.
received only 900 shirts and incurred the following expenses.
LKR
Freight and other Expenses 1, 00,000
Cartage 5,000
Sales expenses 6,000
The consignee sold 600 shirts only. You are required to calculate the value of closing stock.
Solution:
Calculation of the value of unsold stock
Shirts received 900- shirts sold 600 = unsold stock 300
(i) Cost of unsold stock 300 × 700 =
2,10,000
42 | P a g e
A.Ajanthan Joint Venture & Consignment Accounts
(ii) Add: Proportionate Expenses Paid by consignor
3/10* 10,500(7500 + 500 + 2500) = 3,150
(iii) Add: Proportionate Expense Freight 1,00,000
Paid by consignee Cartage 5,000
105,000
1, 05, 000
× 300 = 35,000
900 248,150
Illustration: 15
S of Bombay consigned 10,000 Liter of oil to D of Calcutta. The cost of oil was Lkr.2 per
Liter. S paid Lkr.5, 000 as freight and insurance. During transit 250 Liter were accidentally
destroyed for which the insurers paid directly to the consignors Lkr.450 if full settlement of
the claim. D reported that 7,500 Liter was sold @ Lkr.3 per Liter. The expenses being on
godown rent Lkr. 200, on advertisement Rs.1, 000 and on salesman salary Lkr.2, 000. D is
entitled to a commission of 3% plus 1.5% Del credere. D reported a loss of 100 Liter due to
leakage. D settled the accounts by bank draft. Prepare the accounts in the books of S.
Consignment to Calcutta A/c
Dr Cr
Goods sent on Consignment A/c
Bank- Freight & Insurance
D A/c
Expenses (200+1000+2000)
Commission:
Ordinary(22500* 3%) 675
Del Credere(22500*1.5%) 338
20,000
5,000
3,200
1,013
D A/c
[ (Sale proceeds, 7500×3]
Bank (Insurance company)
P & L A/c (Abnormal loss)
Consignment Stock A/c
P & L A/c (Transfer)
22,500
450
175
5,430
658
29,213 29,213
Goods sent on Consignment A/c
Dr Cr
Trading A/c 20,000 Consignment to Calcutta A/c 20,000
43 | P a g e
A.Ajanthan Joint Venture & Consignment Accounts
Consignment Stock A/c
Dr Cr
Consignment to Calcutta A/c 5,431 Balance c/d 5,431
D A/c
Dr Cr
Consignment to Calcutta A/c 22,500 Consignment to Calcutta (Exp.)
Consignment to Calcutta A/c (com.)
Bank A/c
3,200
1,013
18,287
8,400 8,400
Working Notes:
(A) Cost of Goods destroyed LKR
Cost of 10,000 Liter @Lkr. 2 20,000
Freight 5,000
Total cost of 10,000 Liter 25,000
If 250 Liter were accidentally destroyed,
25,000/10,000*250 = Lkr.625
B) Value of stock still unsold Liter:
Quantity received by D 9,750 [10,000-250(Accidental loss)]
Less: Normal leakage (100)
9650
Cost of 9,650 Liter = Lkr. 25, 000 – Lkr. 625 = Lkr. 24, 375, So
Cost of 2,150 Liter = 24375/9650*2150 = Lkr. 5430
Illustration: 16
Mithuna Traders of Jaffna purchased 10,000 Bags @100 per Bag. Out of these 6,000 Bags
were sent on consignment to Nantha Traders of Kilinochchi at the selling price of 120 per
Bag. The consignors paid 3,000 for packing and freight. Nantha Traders sold 5,000 Bags
@125 per Bag and incurred 1,000 for selling expenses and remitted 5,00,000 to Jaffna on
44 | P a g e
A.Ajanthan Joint Venture & Consignment Accounts
account. They are entitled to a commission of 5% on total sales plus a further of 25%
commission on any surplus price realized over 120 per Bag. 3,000 Bags were sold at Jaffna
@ 110 per Bag. Owing to fall in market price, the value of stock of Bags in hand is to be
reduced by 5%. You are required to prepare;
(i) Consignment Account, and
(ii) Nantha Traders Account.
Solution:
Consignment to Kilinochchi A/c
Dr Cr
Goods sent on Consignment A/c
Bank A/c - (Packing and Freight)
Nantha Traders A/c:
Selling Expenses
Commission (w1)
P & L A/c
6,00,000
3,000
1,000
37,500
79,000
Nantha Traders A/c
[ (Sale proceeds, 5000×125]
Consignment Stock A/c (w2)
6,25,000
95,500
7,20,500 7,20,500
Note: 3,000 Bags which were sold at Jaffna @110 per Bag are not to be taken into
consideration since it is not a consignment transaction and hence the same is extended from
Consignment Account. Although the consignor purchased 10,000 Bags, only 6,000 Bags are
related to consignment transaction, balance is not to be taken into Consignment Account at
all.
Nantha Traders Account
Dr Cr
Consignment to Kilinochchi A/c 6,25,000 Bank A/c (Advance)
Consignment to Kilinochchi A/c
(Selling expenses)
Consignment to Kilinochchi A/c
(Commission)
Bank A/c
5,00,000
1,000
37,500
86,500
6,25,000 6,25,000
Workings Notes:
45 | P a g e
A.Ajanthan Joint Venture & Consignment Accounts
1. Calculation of commission payable to Nantha Traders:
LKR
Total Sales @ 125 per Bag
Less: Amount 120 per Bag
Surplus Price Realized
5% Commission on total Sales (Lkr 625,000*5%)
25% Commission on surplus price realized (Lkr 25,000*25%)
6,25,000
(6,00,000)
25,000
31,250
6,250
Total commission payable 37,500
2. Valuation of unsold stock:
Since market price has fallen by 5%, valuation of unsold stock on consignment will be
calculated as under:
LKR
Total Cost (1,000 x 100)
packing and freight(3000/6000*1000)
Less: 5% in reduction
1,00,000
500
(5,000)
Stock on Consignment 95,500
Illustration: 17
A company sent 300 bales of cotton to its consignee at profit 20% on sale. The cost of each
bale to company is Lkr.600 per bale. The following are the expenses incurred in connection
with this consignment:
(a) Lkr.900 paid by the consignor for dispatching goods.
(b) Lkr.2, 000 paid by the consignee by way of freight, duty and landing charges.
(c) Lkr.1, 000 paid by the consignee by way of godown rent, salaries of salesman.
Required: The valuation of stock at the end (at invoice price) if the consignee sells
away 2/3rd
of the consignment.
Solution:
Total bales sent 300
Less: bales sold 2/3rd of 300 (200)
Bales unsold 100
Cost price of 100 bales at Rs.600 per bale 60,000
46 | P a g e
A.Ajanthan Joint Venture & Consignment Accounts
Add: Profit at 20% on sale or 25% on cost 15,000
75,000
Add (1/3rd
direct expenses):
Expenses paid by Consignor 900
Expenses paid by Consignor 2,000
1/3rd thereof 2900 2900*1/3 967
75,967
Note: In the consignment account, stock reserve account will appear at Rs.15, 000 on the
debit side.
Illustration: 18
Alagu sold goods on behalf of Aju Sales Corporation on consignment basis. On 1st
January,
2002 he had with him a stock of Lkr.20, 000 on consignment. During the year, he received
goods worth Lkr.2, 00,000. Alagu had instructions to sell goods at cost plus 25% and was
entitled to a commission of 4% on sales in addition to 1% del credere commission. During
the year ended 31st
December, 2002 cash sales were Lkr.1, 20,000; credit sales Lkr.1, 05,000;
Alagu‘s expenses relating to consignment Lkr.3, 000 being salaries and insurance & bad
debts amounted to Lkr.3, 000.
Required: Prepare necessary accounts in the books of Aju Sales Corporation. (Consignor)
Solution:
Consignment Account
Dr Cr
Consignment Stock b/d
Goods sent on Consignment A/c
Alagu A/c (Commission)
Alagu A/c (Commission)
Alagu A/c (salaries and insurance)
P & L A/c
20,000
2,00,000
9,000
2,250
3,000
30,750
Alagu A/c
Cash Sales 1,20,000
Credit Sales 1,05,000
Consignment Stock A/c
2,25,000
40,000
2,65,000 2,65,000
Alagu A/c
Dr Cr
47 | P a g e
A.Ajanthan Joint Venture & Consignment Accounts
Consignment A/c (sales) 2,25,000 Consignment A/c
(Commission)
Consignment A/c
(Commission)
Consignment A/c
(salaries and insurance)
Balance c/d
9,000
2,250
3,000
2,10,750
2,25,000 2,25,000
Working Notes:
(1) Calculation of Consignment Stock
Sale Price = 100 + 25 = 125
Cost of Sales = Sales ×100/125
= 2, 25,000 × 100/125
= Lkr.1, 80,000
Cost of the goods available for sale = Lkr. 20,000(op. stock) + Lkr.2, 00,000 = Lkr.2,
20,000
Hence stock at the end = Lkr.2, 20,000 - Lkr.1, 80,000 = Lkr.40, 000
(2) Since Alagu is paid del-credere commission, bad debts of Rs.3, 000 would be borne by
him.
Illustration: 19
On 10 January 2010 Kumar Sangakara of Galle consigned 1000 calculators to Mahela,
Kadawatta. The goods are invoiced at Lkr 30 per unit, the cost price being Lkr 20 per unit.
Expenses incurred are: insurance Lkr 150; freight Lkr 1000; cartage and packing Lkr
300. The agent is to receive ordinary commission of 5% and del-credere commission of
4%.Mahela receive the goods on 31 January and pays cash for; freight and cartage Lkr
350;advertising Lkr 250. Repacking of calculators cost is Lkr 200. Mahela sent Kumar
Sangakara a cheque for Lkr 5000 as an advance on 31 January 2010. The following sales are
made by Mahela to 30 June:
Date Cash Credit
2010 Feb 6 50 @ Lkr 40 Lkr 2,000 30 @ Lkr 50 Lkr 1,500
Mar 18 90 @ Lkr 50 Lkr 4,500 150 @ Lkr 40 Lkr 6,000
48 | P a g e
A.Ajanthan Joint Venture & Consignment Accounts
Apr 20 330 @ Lkr 40 Lkr 1,200 100 @ Lkr 40 Lkr 4,000
Jun 9 97 @ Lkr 50 Lkr 4,850
Mahela took three calculators for their own stock to be accounted for at the current selling
price of Lkr 50. Kumar Sangakara balanced the accounts at the end of June and received an
account sale from the consignee to this date:
Prepare:
a) The account sales received by the consignor on 30 June 2010
b) Ledger accounts in the books of the consignor
c) Ledger accounts in the books of the consignee
Solution:
Date
Account Sales
30 June 2010
Units for Cash Units on Credit Total Value
Feb 6
Mar18
Apr20
Jun 9
50 @ Lkr 40 Lkr 2,000 30 @ Lkr 50 Lkr 1,500
90 @ Lkr 50 Lkr 4,500 150 @ Lkr 40 Lkr 6,000
330 @ Lkr 40 Lkr 13,200 100 @ Lkr 40 Lkr 4,000
97 @ Lkr 50 Lkr 4850
Taken from own stock 3 @Lkr 50
Less: Expenses & charges LKR
Freight and cartage 350
Advertising 250
Repacking machine 200
Commission 2,270
Net proceeds
Less: Advance 31 January
Cheque enclosed Lkr 28130
Signed Mahela - Manager
3,500
10500
17,200
4,850
*150
36,200
(3,070)
33,130
(5,000)
28,130
*Where goods are to be used by the consignee in some other business activity, the debit is to
the purchase account. If the goods are taken for private purposes the drawings account is
debited.
49 | P a g e
A.Ajanthan Joint Venture & Consignment Accounts
**Commission calculated thus: 36200 × 5% = 1810
11500 × 4% = 460 2270
Ledger of Kumar Sangakara
Consignment A/c - Mahela, Kadawatta
Dr Cr
Goods sent on Consignment A/c
(1000*20)
Bank A/c – expenses
Insurance 150
Freight 1,000
Cartage & packing 300
Mahela A/c
Freight & Cartage 350
Advertising 250
Repacking goods 200
Commission 2,270
P & L A/c
20,000
1,450
3,070
14,980
Mahela A/c
Cash Sales 24,700
Credit Sales 11,500
Consignment Stock A/c
150 units @ lkr 20 3,000
(+) 2000/1000*150 300
36,200
3,300
39,500 39,500
Mahela A/c (Consignee)
Dr Cr
Consignment A/c (sales) 36,200 Bank A/c- Advance
Consignment- Mahela A/c
(expenses & commission)
Bank A/c - settlement
5,000
3,070
28,130
36,200 36,200
Bank A/C
Dr Cr
50 | P a g e
A.Ajanthan Joint Venture & Consignment Accounts
Mahela A/c - advance
Mahela A/c - settlement
5,000
28,130
Consignment A/c
Insurance 150
Freight 1,000
Cartage & packing 300 1450
Consignment Stock A/c
Dr Cr
Consignment A/c 3300
Goods sent on Consignment A/c
Dr Cr
Trading A/c 20,000 Consignment A/c 20,000
Ledger of Mahela
Kumar Sangakara A/c (Consignor)
Dr Cr
Bank A/c- advance
expenses
commission received
Bank - settlement
5,000
800
2,270
28,130
Bank A/c 24550
Account Receivable 11500
Purchases
36,050
150
36,200 36,200
Commission Received
Dr Cr
Kumar Sangakara A/c 2,270
Bank A/c
Dr Cr
51 | P a g e
A.Ajanthan Joint Venture & Consignment Accounts
Kumar Sangakara A/c
Mahela A/c - settlement
24,550
28,130
Kumar Sangakara A/c- advance
Kumar Sangakara A/c- expenses
Freight & Cartage 350
Advertising 250
Repacking goods 200
Kumar Sangakara A/c- settlement
5,000
800
28,130
Accounts Receivable
Dr Cr
Kumar Sangakara A/c 11,500
Illustration: 20
ARA & Co consigned 1,000 tin of Ghee costing Lkr.60 per tin to their agents, Anusha Stores,
at Calcutta. The agents sold 400 tin at Lkr.80 per tin for cash, 400 tins at Lkr.82 per tin on
credit and they took over the balance to their own stock at Lkr.82 per tin. ARA & Co paid
freight and carriage Lkr.500 and miscellaneous expenses Lkr.200. They drew on Anusha
Stores at 3 Months for Lkr.45, 000, which was duly accepted by the later. The expenses
incurred by the Anusha Stores were:
LKR
Carriage 50
Octroi 40
Storage 110
Miscellaneous 100
They were entitled to 5% commission and 2% del credere commission on total gross sale
proceeds. They sent their account sales to their principal showing as a deduction there from
their commission and the various expenses incurred by them a month later. All the debtors
except one who owed Lkr.200 paid cash and the Anusha Stores remitted the amounts due on
consignment. You are required to show;
a) The journal entries in the books of the consignor and
b) Consignment account in the consignor‘s ledger
52 | P a g e
A.Ajanthan Joint Venture & Consignment Accounts
Solution:
Journal Entries
(In the books of Consignor)
Description
Dr
(LKR)
Cr
(LKR)
1. Consignment A/c
Goods sent on consignment A/c
(being the goods sent on consignment)
60,000
60,000
2. Consignment A/c
Bank A/c
(Being the expenses incurred by consignor on account of consignment)
700
700
3. Consignment Account
Anusha Stores A/c
(Being the expenses incurred by consignee on account of consignment)
300
300
4. Anusha Stores A/c
Consignment A/c
(Being the sale effected by the consignee.)
81,200
81,200
5. Consignment A/c
Anusha Stores A/c
(Being the commission on sales).
5,684
5,684
6. Consignment A/c
Profit & Loss A/c
(Being the profit on consignment transferred to profit and loss account)
14,516
14,516
7. Goods sent on consignment A/c
Purchase A/c
(Being the value of goods sent on consignment)
60,000
60,000
8. Bills Receivable A/c
Anusha Stores A/c
(Being the bill drawn on consignment)
45,000
45,000
Ledger
Consignment of Calcutta Account
Dr Cr
53 | P a g e
A.Ajanthan Joint Venture & Consignment Accounts
Goods sent on Consignment A/c
Bank-Expenses
Anusha Stores A/c- Expenses
Anusha Stores A/c- Commission
P & L A/c
60,000
700
300
5,684
14,516
Anusha Stores A/c
Cash Sales (400*80) 32,000
Credit Sales (400*82) 32,800
Balance of stock taken (200*82)
64,800
16,400
81,200 81,200
Illustration: 21
On January 1, 2002, A of Delhi sent on consignment to B of Bombay 200 packets of coffee,
costing Lkr.80 and invoiced pro forma at Lkr.100 each. The freight and other charges paid
by A amounted to Lkr.640. A sent the documents through Bank and drew upon B a bill for
Lkr.10, 000 and discounted the same with the Bank for Lkr.9, 800. The bill was met on
maturity.
On March 15, B sent Account sales (together with the amount due) showing that 150
packets had realized Lkr.100 each and 25 packets Lkr.110 each and 25 packets were shown
as unsold stock. B incurred Lkr.400 as expenses for the entire consignment. B is entitled to a
commission of 6%.
On March 31 B informed A that 15 packets were damaged due to bad packing and it
was estimated that the selling price of the damaged packets would be about Lkr.20 per
packet. Both A and B close their books on March 31.
Prepare ledger accounts in the books of A and B.
Solution:
Books of A, Delhi
Consignment of Bombay Account
Dr Cr
Goods sent on Consignment A/c
Bank-Expenses
B A/c- Expenses
B A/c – Commission
Stock Reserve Account
P & L A/c
20,000
640
400
1,065
200
1,725
B A/c
Goods sent on consignment
(loading)
Abnormal Loss (1)
Stock on Consignment (2)
Stock of damaged goods
17,750
4,000
648
1,032
600
24,030 24,030
54 | P a g e
A.Ajanthan Joint Venture & Consignment Accounts
B’s Account
Dr Cr
Consignment A/c (sales) 17,750 Bills Receivable
Consignment A/c - expenses
Consignment A/c commission
Bank A/c - settlement
10,000
400
1,065
6,785
36,200 36,200
Goods sent of Consignment Account
Dr Cr
Consignment account Loading
Purchase/ Trading A/c
4,000
16,000
Consignment A/c 20,000
36,200 36,200
Books of B
A A/c
Dr Cr
Bills Payable
Bank-Expenses
Commission A/c
Bank
10,000
400
1,065
6,785
Bank
Balance c/d
17,750
500
18,250 18,250
Note:
(i) Stock at the end (At Invoice Price) Lkr.
10 Packets @ Lkr.100 (Invoice Price) 1,000
Add: Proportionate expenses incurred by A i.e. 1/20th of Lkr.640 32
1,032
(ii) Abnormal Loss
Cost of 15 packets damaged (15*80) 1,200
Add: Proportionate expenses 640/200*15 48
1248
Less: Value of 15 packets @ Lkr.20 per Packet (600)
55 | P a g e
A.Ajanthan Joint Venture & Consignment Accounts
648
(iii) Since 10 Packets are still in the stock-in-hand, advance to that extent has not been
adjusted. Hence Lkr.500 is carried forward i.e.
10,000 ×10/200= Lkr. 500
Where Normal and Abnormal Losses occur simultaneously
If both normal and abnormal losses occur simultaneously in connection with the same
consignment, the computation of the value of closing stock involves the following
procedures:
1. Take the total cost of goods consigned and add all the non-recurring expenses
(incurred by the consignor and consignee).
2. Deduct the quantity and cost of abnormal loss from the total number of goods
consigned and the cost as obtained in (1) above, respectively.
3. Deduct the quantity of normal loss from the quantity worked out in (2) above
without making any adjustment in cost.
4. Calculate cost per unit of goods units by dividing the cost (remaining after
deducting the cost of abnormal loss) by the number of goods units.
5. Multiply the number of unsold units with the cost per unit obtained in (4) above to
arrive at the value of unsold stock.
Illustration: 22
Vegetables Oils Ltd., Polannaruwa, consigned 10,000 Liters of Ghee costing Lkr.20 per Liter
to Ranga and Co. of Galle on 1st January 2012. Oils Ltd paid Lkr.50, 000 as freight and
insurance. 250 Liters of Ghee were destroyed on 10-1-2012 in transit. The insurance claim
was settled at Lkr.4, 500 and was paid directly to the consignors. Ranga and Co. took
delivery of the consignment on 20th
January 2012 and accepted a bill drawn upon them by
Oils Ltd for Lkr 1, 00,000 for 3 months. On 31st
March 2012 Ranga and co. reported as
Follows.
(i) 7,500 Liters were sold at Lkr.30 per Liter.
(ii) Other expenses were: Godown rent Lkr.2, 000; Wages Lkr.20, 000 Printing and
Stationary including advertising Lkr.10, 000.
(iii) 250 Liters were lost due to leakage.
56 | P a g e
A.Ajanthan Joint Venture & Consignment Accounts
Ranga and Co are entitled to a commission of 4.5% on all the sales affected by them.
They paid the amount due in respect of consignment on 31st
March itself.
Show the consignment account, the account of Ranga and Co. and loss-in-transit
account in the books of consignor for the year ended 31st March 2002.
Solution:
Consignment to Galle Account
Dr Cr
Goods sent on Consignment A/c
Bank A/c - freight and insurance
Ranga and Co A/c
[(Expenses + Commission),
(2,000+20,000+10,000+10,125)
2,00,000
50,000
45,125
Ranga and Co A/c (7500*30)
Loss-in-transit (w1)
Stock on Consignment A/c (w2)
Profit & Loss A/c
2,25,000
6,250
51,316
9,559
2,92,125 2,92,125
Loss-in-Transit A/c
Dr Cr
Consignment A/c 6,250 Insurance Co. A/c
Profit & Loss A/c
4,500
1,750
6,250 6,250
Ranga and Co A/c
Dr Cr
Consignment A/c (sales)
Balance c/d
2,25,000
20,000
Bill Receivable
Consignment A/c
(Expenses and Commission)
Bank A/c
100,000
42,125
1,02,875
2,45,000 2,45,000
Working Notes:
(1) Cost of ghee destroyed in transit LKR.
Cost of 10,000 Kg of ghee @ Lkr. 20 2, 00,000
Freight and Insurance 50,000
Total cost of 10,000 Kg 2, 50,000
57 | P a g e
A.Ajanthan Joint Venture & Consignment Accounts
Cost of 250 Kg 2, 50,000/10,000*250 (6,250)
Cost of 9,750 Kg of ghee 2, 43,750
(2) Value of stock at the end Kg
Quantity of ghee received by the consignee 9,750
Less: Quantity lost through leakage (Normal Loss) (250)
Quantity Available for sale 9,500
Total Cost of 9,500 Kg 2, 43,750
Cost of 2,000 Kg 243,750/9,500*2,000 51,316
(3) Since 2000 Kg (9500 – 7500) of ghee has not been sold.
Proportionate amount of advance is (100,000×1/5) Lkr.20, 000 will not be adjusted.
Illustration: 23
5,000 shirts were consigned by Raizada & Co. of Delhi to Zing of Tokyo at cost of 375 each.
Raizada & Co. paid freight 50,000 and Insurance 7,500. During the transit 500 shirts were
totally damaged by fire. Zing took delivery of the remaining shirts and paid 72,000 on custom
duty. Zing had sent a bank draft to Raizada & Co. for 2, 50,000 as advance payment. 4,000
shirts were sold by him at 500 each. Expenses incurred by Zing on godown rent and
advertisement etc. amounted to 10,000. He is entitled to a commission of 5%. One of the
customer to whom the goods were sold on credit could not pay the cost of 25 shirts.
Prepare the Consignment Account and the Account of Zing in the books of Raizada &
Co. (Zing settled his account immediately. Nothing was recovered from the insurer for the
damaged goods).
Solution:
Consignment A/c
Dr Cr
Goods sent on Consignment A/c
Bank A/c - freight and insurance
Zing A/c:
[(Custom Duty +Godown Rent,
Adv. Etc+ Commission),
18,75,000
57500
1,82,000
Zing A/c
(3,975 x 500)
Consignment Debtors A/c
[Credit Sales (25 x 500)]
Abnormal Loss A/c (w 1)
19,87,500
51,316
12,500
1,93,250
58 | P a g e
A.Ajanthan Joint Venture & Consignment Accounts
(72,000+10,000+100000)]
Consignment Debtors A/c - Bad
Debts
Profit and Loss A/c
12,500
2,67,500
Stock on Consignment A/c(w2) 2,01,250
9,559
23,94,500 23,94,500
Zing A/c
Dr Cr
Consignment A/c (Sales) 19,87,500 Bank Draft A/c (Advance)
Consignment A/c
(Expenses and Commission)
Bank A/c (Final Settlement)
2,50,000
1,82,000
15,55,500
19,87,500 19,87,500
Abnormal Loss A/c
Dr Cr
Consignment A/c 1,93,250 Profit & Loss A/c 1,93,250
1,93,250 1,93,250
Working Notes:
1.Valuation of goods Lost-in-transit and unsold Stock: Lkr
Total Cost 18, 75,000
Add: Consignor‘s Expenses 57,500
Total Cost of 5,000 Shirts 19, 32,500
Less: Lost-in-transit 1932500/5000*500 1, 93,250
Add: Non-recurring Ex. of Consignee 72,000
Total Cost of 4,500 Shirts 18, 11,250
2. Value of under Stock `18, 11,250 /4500*500 = Lkr 201,250
Note: Since Del Credere Commission is not given by the consignor to the consignee, amount
of bad debt is to be charged against Consignment Account.
59 | P a g e
A.Ajanthan Joint Venture & Consignment Accounts
Illustration: 24
Lubrizols Ltd. of Mumbai consigned 1,000 barrels of lubricant oil costing Liters 800 per
barrel to Central Oil Co. of Kolkata on 1.1.2012. Lubrizols Ltd. paid Lkr 50,000 as freight
and insurance. 25 barrels were destroyed on 7.1.2012 in transit. The insurance claim was
settled at Lkr 15,000 and was paid directly to the consignor. Central Oil took delivery of the
consignment on 19.1.2012 and accepted a bill drawn upon them by Lubrizols Ltd., for Lkr 5,
00,000 for 3 months. On 31.3.2012 Central Oil reported as follows:
(i) 750 barrels were sold as Lkr 1,200 per barrel.
(ii) The other expenses were:
LKR
Clearing charges 11,250
Godown Rent 10,000
Wages 30,000
Printing, Stationery, Advertisement 20,000
25 barrels of oil were lost due to leakage which is considered to be normal loss. Central Oil
Co. is entitled to a commission of 5% on all the sales affected by them. Central Oil Company
paid the amount due in respect of the consignment on 31st
March itself. Show the
Consignment Account, the Account of Central Oil Co., and the Lost –in-Transit Account as
they will appear in the books of Lubrizols Ltd.
Solution:
Consignment to Kolkata Account
Dr Cr
Goods sent on Consignment A/c
Bank A/c - freight and insurance
Central Oil Co A/c:
[(Freight +Godown Rent+
Wages+ Printing etc),
(11250+10,000+30000+20000)]
Commissions @5%
Profit and Loss A/c
8,00,000
50,000
71,250
45,000
1,31,842
Central Oil Co. A/c
(750 x 1200)
Abnormal Loss A/c
Stock on Consignment A/c
9,00,000
21,250
1,76,842
10,98,092 10,98,092
60 | P a g e
A.Ajanthan Joint Venture & Consignment Accounts
Central Oil Co. Ltd. A/c
Dr Cr
Consignment to Kolkata A/c
(Sales)
9,00,000 Bills Receivable A/c (advance)
Consignment to Kolkata A/c
Expenses
Commission
Bank A/c (amount due)
5,00,000
1,82,000
71,250
45,000
2,83,750
9,00,000 9,00,000
Abnormal Loss A/c
Dr Cr
Consignment to Kolkata A/c 21,250 Bank-Insurance Claim A/c
Profit & Loss A/c
15,000
6,250
21,250 21,250
Workings Notes:
1.Valuation of goods Lost-in-transit and unsold Stock:
Lkr
Total Cost (1000*800) 8, 00,000
Add: Consignor‘s Expenses 50,000
Total Cost of 1,000 barrels 8, 50,000
Less: Lost-in-transit 850000/1100*25 (21,250)
Add: Non-recurring Ex. of Consignee 11,250
Total Cost of 950 barrels 8, 40,000
2. Value of under Stock `840000 /950*200 = Lkr 1, 76, 842
Illustration: 25
Mr. X, the consignor, consigned goods to Mr. Y 100 Radio sets valued Lkr 50,000. This was
made by adding 25% on cost. Mr. X paid Lkr 5,000 for freight and insurance. 20 sets are lost
– in- transit for which Mr. X received Lkr 5,000 from the Insurance Company.
Mr. Y received remaining goods in good condition. He incurred Lkr 4,000 for freight and
miscellaneous expenses and Lkr 3,000 for godown rent. He sold 60 sets for Lkr 50,000. Show
the necessary ledger account in the books of Mr. X assuming that Mr. Y was entitled to an
61 | P a g e
A.Ajanthan Joint Venture & Consignment Accounts
ordinary Commission of 10% on sales and 5% Del Credere Commission on sales. He also
reported that Lkr 1,000 were proved bad. Prepare the necessary Accounts.
Solution:
Consignment A/c
Dr Cr
Goods sent on Consignment A/c
Bank A/c - freight and insurance
Y A/c:
Freight and Misc. Expenses
Godown Rent
Commission (ordinary) @ 10%
Del credere Commission @ 5%
Abnormal Loss A/c (Loading)
Stock reserve A/c
Profit and Loss A/c
50,000
5,000
4,000
3,000
5,000
2,500
2,000
2,000
9,500
Goods Sent on Consignment A/c
(Loading) (` 50,000x100/125)
Y A/c –sale proceeds
Abnormal Loss A/c
Stock on Consignment A/c
10,000
50,000
11,000
12,000
83,000 83,000
Y A/c
Dr Cr
Consignment A/c
(Sales)
50,000 Consignment A/c:
Expenses
Commission (5000+2500)
Balance C/d
7,000
7,500
35,500
50,000 50,000
Abnormal Loss A/c
Dr Cr
Consignment A/c 11,000 Consignment A/c (Loading)
Bank-Insurance Claim A/c
Profit and Loss A/c
(Loss transferred)
2,000
5,000
4,000
11,000 11,000
62 | P a g e
A.Ajanthan Joint Venture & Consignment Accounts
Working Notes:
(1) Calculation of Loading:
I.P Load C.P
125 25 100
C.P=100/125*50000
= 40000
Invoice price is Lkr 50000, so Loading is Lkr 10,000(50,000 – 40,000)
Loading per set = Lkr 10,000 ÷ 100 = Lkr 100
(2) Valuation of goods lost – in – transit and unsold stock
LKR
Total invoice price 50,000
Add: consignor‘s expenses (5,000)
Invoice price of 100 sets 55,000
Less: lost in transit (55000/100*20) (11,000)
44,000
Add: Non recurring expenses of Mr. Y 4,000
Invoice price of 80 sets 48,000
For unsold stock of (100 – 20 -60) = 20 sets
48000/80*20=12000
(3) Loading on abnormal loss = 20 x Lkr 100 = Lkr 2,000
(4) Stock suspense = 20sets x Lkr 100 = Lkr 2,000
(5) Since Del Credere Commission is given there will not be any entry for bad debts.
Illustration: 26
From the following two statements, prepare Consignment A/c and Consignee‘s A/c in the
books of Consignor, presuming that the goods were invoiced at 20% above cost.
M/s Vijay & Company To: M/s Jyoti Electric House
Mumbai Pune
No 2355 Proforma Invoice Date: 21st April 2012
63 | P a g e
A.Ajanthan Joint Venture & Consignment Accounts
Particulars of goods sent on consignment: Amount
Lkr
Amount
Lkr
800 Fans @ Lkr 1680 per fan
Add: Expenses Paid:
Freight
Insurance 6000
Sundries
Total
E & O E Sign
Mumbai For Vijay & Company
4,000
6,000
2,000
13,44,000
12,000
13,56,000
M/s Jyoti Electric House To: M/s Vijay & Company
Pune Mumbai
(Account sales of 800 fans received from Vijay & Company, Mumbai)
Date: 21st September 2012
Sale proceeds of 600 Fans @ Lkr. 2000 per fan
Less: Expenses Paid :
Advertising
Insurance
Octroi
Commission @10%
Total
Less: Bill Accepted
Less: Bank draft enclosed
E & O E Sign
Mumbai Jyoti Electric House
Amount
(LKR)
Amount
(LKR)
4,500
1,500
12,000
1,20,000
12,00,000
(1,38,000)
10,62,000
7,50,000
3,12,000
Consignment to Pune Account
Dr Cr
64 | P a g e
A.Ajanthan Joint Venture & Consignment Accounts
Goods sent on Consignment A/c
Bank A/c - freight and insurance
& Sundries
M/s Jyoti Electric House‘s A/c:
Expenses
Commission
Stock reserve A/c (loading on
stock)
Profit and Loss A/c
13,44,000
12,000
18,000
1,20,000
56,000
2,16,000
M/s Jyoti Electric House‘s A/c
(sale proceeds)
Sent on Consignment A/c
12,00,000
2,24,000
3,42,000
17,66,000 17,66,000
Jyoti Electric House’s A/c
Dr Cr
Consignment A/c
(Sales)
12,00,000 Consignment A/c:
Expenses
Commission
Bank A/c
Bills Receivable A/c
18,000
1,20,000
3,12,000
7,50,000
12,00,000 12,00,000
Workings Notes:
Loading on consignment Lkr
Invoice price of fans consigned
Loading is 20% on cost
Thus loading to be removed 20/120 × 1680
Total loading removed (800 × 280)
Value of closing Stock
Original invoice value
Consignor‘s expenses
Consignee‘s non-recurring expenses (Octroi only)
Loading on consignment
Total fans sent
Fans sold
1,680
280
2,24,000
13,44,000
12,000
12,000
13,68,000
800
600
65 | P a g e
A.Ajanthan Joint Venture & Consignment Accounts
In Stock
Hence, stock value (13,68,000/800 × 200)
Loading to be removed (200 × 280)
200
3,42,000
56,000
Illustration: 27
On 1.7.2012, Mantu of Chennai consigned goods of the value of Lkr 50,000 to Pandey of
Patna. This made by adding 25% on cost. Mantu paid that on Lkr 2,500 for freight and Lkr
1,500 for insurance. During transit1/10 th
of the goods was totally destroyed by fire and a sum
of Lkr 2,400 was realized from the insurance company. On arrival of the goods, Pandey paid
Lkr 1,800 as carriage to godown. During the year ended 30th June 2013, Pandey paid Lkr
3,600 for godown rent and Lkr 1,900 for selling expenses.1/9 th
of the remaining goods was
again destroyed by fire in godown and nothing was received from the insurance company. On
1.6.2013, Pandey sold half (1/2) the original goods for Lkr 30,000 and changed a commission
of 5% on sales. As on 30.6.2013, Pandey sent a bank draft to Mantu for the amount so far due
from him. You are required to prepare the following ledger accounts in the books of Mantu of
Chennai for the year ended 30.6.2013.
(a) Consignment to Patna Account; (b) Goods Destroyed by Fire Account; and (c) Personal
Account of Pandey.
Solution:
Consignment to Patna Account
Dr Cr
Goods sent on Consignment A/c
Bank A/c - freight and insurance
(2500+1500)
Pandey A/c :
Carriage Inward 1800
Godown Rent 3600
Selling Expenses 1900
Commission (5% on Lkr 30,000)
Goods Destroyed by Fire A/c :
Loading
Stock reserve A/c (Loading on
unsold stock)
50,000
4,000
7300
1,500
2000
3000
Goods Sent on Consignment A/c
Pandey A/c : Sale Proceeds
Goods Destroyed by Fire A/c
Stock on Consignment A/c
10,000
30,000
11,000
16,800
66 | P a g e
A.Ajanthan Joint Venture & Consignment Accounts
17,66,000 17,66,000
Note: There is no normal Profit or Loss on Consignment.
Goods Destroyed by Fire A/c
Dr Cr
Consignment to Patna A/c :
In transit
In Godown
5,400
5,600
Consignment to Patna A/c :
Loading
Bank-Insurance Claim A/c
Profit and Loss A/c
2,000
2,400
6,600
11,000 11,000
Pandey Account
Dr Cr
Consignment to Patna A/c
(Sale proceeds)
30,000 Consignment A/c:
Expenses
Commission
Draft A/c
7,300
1,500
21,200
30,000 30,000
Working Notes:
Valuation of goods destroyed by fire and unsold stock
Total Insurance Claim
Add: Consignor‘s Expenses
Less: Lost-in-transit ( 1/10 x lkr 54,000)
Goods received (9/10 th of lkr 54,000)
Add: Non- recurring expenses of Pandey
Less: Value of goods destroyed by fire in godown ( 1/9 th
of lkr 50,400)
Value of 8/10 th
50,000
4,000
54000
(5400)
48,600
1,800
50,400
(5,600)
44,800
67 | P a g e
A.Ajanthan Joint Venture & Consignment Accounts
∴ Value of unsold stock 9/10 - (1/9 th
of 9/10) =9/10 -1/10 =8/10
Goods sold ½ i.e., = 8/10 – 1/2= 3/10 th
∴ Value of unsold stock = Lkr 44,800 x 3/10 x 10/8 = Lkr 16,800
Loading on goods destroyed = ` 10,000 x 2/10 = Lkr 2,000
Loading on unsold stock = ` 10,000 x 3/10 = Lkr 3,000.
Illustration: 28
Usha sent goods costing Lkr 75, 50,000 on consignment basis to Gayatri on 1st Feb 2012 @
8.5% commission. Usha spent Lkr 8, 25,000 on transportation. Gayatri spent Lkr 5, 25,000
on unloading. Gayatri sold 88% of the goods for Lkr 90, 00,000, 10% of the goods for Lkr
10, 00,000 and the balance is taken over by her at 10% below the cost price. She sent a
cheque to Usha for the amount due after deducting commission. Show Consignment to
Gayatri A/c and Gayatri‘s A/c in the books of Usha.
Solution:
Calculation of sales Cost (Lkr) Invoice (Lkr)
Goods sent 75, 50,000
88% of the goods 66, 44,000 90, 00,000
10% of goods 7, 55,000 10, 00,000
Total sales 73, 99,000 1, 00, 00,000
Goods taken over by Gayatri 1, 51,000 1, 35,900
There is no closing stock here as all unsold goods were taken over by Gayatri. The
commission is payable only on sales to outsiders and not on goods taken over by Gayatri.
Thus, commission is 8.5% on Lkr 10,000,000 i.e. Lkr 8, 50,000
The required ledger accounts are shown below.
Consignment to Gayatri A/c
Dr Cr
68 | P a g e
A.Ajanthan Joint Venture & Consignment Accounts
Goods sent on Consignment
A/c
Bank A/c - transportation)
Gayatri‘s A/c:
Unloading charges 5,25,000
Commission 8,50,000
To P & L A/c
75,50,000
8,25,000
13,75,000
3,85,900
Gayatri‘s A/c (sales)
Gayatri‘s A/c (goods taken
over)
10,000,000
1,35,900
1,01,35,900 1,01,35,900
Gayatri’s A/c
Dr Cr
Consignment A/c
(Sale proceeds)
1,01,35,900 Consignment A/c:
Expenses
Commission
Bank A/c
5,25,000
8,50,000
87,60,900
1,01,35,900 1,01,35,900
Calculation of Abnormal Loss: 250 kg of oil lost in transit
Cost of 250 kg @ 40/kg 10,000
Proportionate expenses of Babubhai
(250/10000*7500)
188 10,188
Calculation of closing stock Kg
Oil consigned to Delhi
Less: Lost in transit
Less: Normal loss due to leakage
Less: Quantity sold
Stock in hand
Basic cost of stock consigned @ ` 40
Less : Cost of abnormal loss
Cost of stock after normal loss of 100kg
Thus cost of 2150 kg
(389812/9650*2150)
10,000
(250)
(100)
(7,500)
2,150
400,000
(10,188)
389,812
86,849
69 | P a g e
A.Ajanthan Joint Venture & Consignment Accounts
Calculation of commission
Ordinary @ 3% on 450000
Del Credre @ 1.5% on 450000
13,500
6,750
20,250
As the consignee has paid Del Credre Commission, the responsibility of bad debts is his.
Hence no entry is needed to be passed in the books of consignor.
Illustration: 29
Sangita Machine Corporation sent 200 sewing machines to Rita agencies. It spent Lkr 7500
on packing. The cost of each machine was Lkr 2,000, but it was invoiced at 20% above cost.
20 machines were lost in transit & insurance company accepted claim of Lkr 20,000 only.
Rita agencies paid freight of Lkr 9,000, carriage Lkr 3,600, and Octroi Lkr 1,800 and rent Lkr
1800. They sold 150 machines at Lkr 3,500 per machine. They were entitled to commission
of 5% on invoice price and additional 20% of any excess realized on invoice price and 2%
Del Credre commission. They accepted a bill drawn by Sangita Machine Corporation for Lkr
3, 00,000 and remitted the balance by demand draft along with account sale. Draw up
necessary ledger accounts in the books of Sangita Machine Corporation and Rita Agencies.
Solution:
Consignment to Rita Agencies A/c
Dr Cr
Goods sent on Consignment A/c
Bank A/c - (Packing Expenses)
Rita Agencies A/c:
Freight 9,000
Carriage 3,600
Octroi 1,800
Rent 1,800
Commission 61,500
Abnormal loss A/c
(Load removed)
Stock Reserve A/c
P & L A/c
4,80,000
7,500
77,700
8,000
12,000
1,44,075
Rita Agencies‘ A/c (Sales)
(sales 150 @ 3500)
Abnormal Loss A/c
Consignment Stock A/c
Goods Sent on Consignment A/c
(Loading)
5,25,000
48,750
75,525
80,000
7,29,275 7,29,275
70 | P a g e
A.Ajanthan Joint Venture & Consignment Accounts
Rita Agencies A/c
Dr Cr
Consignment A/c
(Sale proceeds)
5,25,000 Consignment A/c:
Expenses
Commission
Bills Receivable A/c
Bank A/c
16,200
61,500
3,00,000
1,47,300
5,25,000 5,25,000
Calculation of abnormal loss 20 machines lost in transit:
Cost of 20 machines @ Lkr 2400 48,000
Proportionate expenses of Babubhai 750
(20/200*7500) 48750
Calculation of Closing Stock:
LKR
Invoice value of 30 machines @ 2400 72,000
Add: Consignor‘s proportionate expenses 1,125
Add: Consignee‘s proportionate expenses 2,400
75,525
Stock reserve 30 machines @ Lkr 400 12,000
Calculation of Commission:
Invoice price of machines sold (2400*150) 360,000
Commission @ 5% on this 18,000 (a)
Excess over invoice value
(525000-360000) 165,000
Commission @ 20% on this 33,000 (b)
Del Credre Commission @ 2% on 525000 10,500 (c)
Total Commission (a+b+c) 61,500
71 | P a g e
A.Ajanthan Joint Venture & Consignment Accounts
Books of Rita Agencies
Sangita Machine Corporation A/c
Dr Cr
Cash A/c (expenses)
Commission A/c
Bills Payable A/c
Bank A/c
16,200
61,500
3,00,000
1,47,300
Consignment A/c (sales) 5,25,000
5,25,000 5,25,000
Illustration: 30
Ram of Patna consigns to Shyam of Delhi for sale at invoice price or over. Shyam is entitled
to a commission @ 5% on invoice price and 25% of any surplus price realized. Ram draws
on Shyam at 90 days sight for 80% of the invoice price as security money. Shyam remits the
balance of proceeds after sales, deducting his commission by sight draft. Goods consigned by
Ram to Shyam costing Lkr 20,900 including freight and were invoiced at Lkr 28,400. Sales
made by Shyam were Lkr 26,760 and goods in his hand unsold at 31st Dec, represented an
invoice price of Lkr 6,920. (Original cost including freight Lkr 5,220). Sight draft received by
Ram from Shyam up to 31st Dec was Lkr 6,280. Others were in- transit. Prepare necessary
Ledger Accounts.
Solution:
Consignment to Delhi Account
Dr Cr
Goods sent on Consignment A/c
Delhi A/c- Commission
Stock Reserve A/c
`(6,920 – 5,220)
P & L A/c
28,400
2,394
1,700
8,686
Shyam A/c – Sale proceeds
(sales 150 @ 3500)
Goods Sent on Consignment A/c
(Loading) (28,400- 20,900)
Consignment Stock A/c
26,760
7,500
6,920
41,180 41,180
Shyam Account
Dr Cr
Consignment to Delhi A/c
(Sale proceeds)
Balance c/d (` 6,920 x 80%)
26,760
5,536
Bills Receivable A/c
Consignment to Delhi A/c
- commission
22,720
2,394
Advanced Financial Accounting - Volume 01
Advanced Financial Accounting - Volume 01
Advanced Financial Accounting - Volume 01
Advanced Financial Accounting - Volume 01
Advanced Financial Accounting - Volume 01
Advanced Financial Accounting - Volume 01
Advanced Financial Accounting - Volume 01
Advanced Financial Accounting - Volume 01
Advanced Financial Accounting - Volume 01
Advanced Financial Accounting - Volume 01
Advanced Financial Accounting - Volume 01
Advanced Financial Accounting - Volume 01
Advanced Financial Accounting - Volume 01
Advanced Financial Accounting - Volume 01
Advanced Financial Accounting - Volume 01
Advanced Financial Accounting - Volume 01
Advanced Financial Accounting - Volume 01
Advanced Financial Accounting - Volume 01
Advanced Financial Accounting - Volume 01
Advanced Financial Accounting - Volume 01
Advanced Financial Accounting - Volume 01
Advanced Financial Accounting - Volume 01
Advanced Financial Accounting - Volume 01
Advanced Financial Accounting - Volume 01
Advanced Financial Accounting - Volume 01
Advanced Financial Accounting - Volume 01
Advanced Financial Accounting - Volume 01
Advanced Financial Accounting - Volume 01
Advanced Financial Accounting - Volume 01
Advanced Financial Accounting - Volume 01
Advanced Financial Accounting - Volume 01
Advanced Financial Accounting - Volume 01
Advanced Financial Accounting - Volume 01
Advanced Financial Accounting - Volume 01
Advanced Financial Accounting - Volume 01
Advanced Financial Accounting - Volume 01
Advanced Financial Accounting - Volume 01
Advanced Financial Accounting - Volume 01
Advanced Financial Accounting - Volume 01
Advanced Financial Accounting - Volume 01
Advanced Financial Accounting - Volume 01
Advanced Financial Accounting - Volume 01
Advanced Financial Accounting - Volume 01
Advanced Financial Accounting - Volume 01
Advanced Financial Accounting - Volume 01
Advanced Financial Accounting - Volume 01
Advanced Financial Accounting - Volume 01
Advanced Financial Accounting - Volume 01
Advanced Financial Accounting - Volume 01
Advanced Financial Accounting - Volume 01
Advanced Financial Accounting - Volume 01
Advanced Financial Accounting - Volume 01
Advanced Financial Accounting - Volume 01
Advanced Financial Accounting - Volume 01
Advanced Financial Accounting - Volume 01
Advanced Financial Accounting - Volume 01
Advanced Financial Accounting - Volume 01
Advanced Financial Accounting - Volume 01
Advanced Financial Accounting - Volume 01
Advanced Financial Accounting - Volume 01
Advanced Financial Accounting - Volume 01
Advanced Financial Accounting - Volume 01
Advanced Financial Accounting - Volume 01
Advanced Financial Accounting - Volume 01
Advanced Financial Accounting - Volume 01
Advanced Financial Accounting - Volume 01
Advanced Financial Accounting - Volume 01
Advanced Financial Accounting - Volume 01
Advanced Financial Accounting - Volume 01
Advanced Financial Accounting - Volume 01
Advanced Financial Accounting - Volume 01
Advanced Financial Accounting - Volume 01
Advanced Financial Accounting - Volume 01
Advanced Financial Accounting - Volume 01
Advanced Financial Accounting - Volume 01
Advanced Financial Accounting - Volume 01
Advanced Financial Accounting - Volume 01
Advanced Financial Accounting - Volume 01
Advanced Financial Accounting - Volume 01
Advanced Financial Accounting - Volume 01
Advanced Financial Accounting - Volume 01
Advanced Financial Accounting - Volume 01
Advanced Financial Accounting - Volume 01
Advanced Financial Accounting - Volume 01
Advanced Financial Accounting - Volume 01
Advanced Financial Accounting - Volume 01
Advanced Financial Accounting - Volume 01
Advanced Financial Accounting - Volume 01
Advanced Financial Accounting - Volume 01
Advanced Financial Accounting - Volume 01
Advanced Financial Accounting - Volume 01

More Related Content

Similar to Advanced Financial Accounting - Volume 01

Factoring regulation act, 2011
Factoring regulation act, 2011Factoring regulation act, 2011
Factoring regulation act, 2011sengel_an
 
Factoring & forfaiting
Factoring & forfaitingFactoring & forfaiting
Factoring & forfaitingTanuj Poddar
 
Caiibfmwctlmoduled contd1 (1)
Caiibfmwctlmoduled contd1 (1)Caiibfmwctlmoduled contd1 (1)
Caiibfmwctlmoduled contd1 (1)summi gupta
 
Unit 8 (tijarah)
Unit 8 (tijarah)Unit 8 (tijarah)
Unit 8 (tijarah)Asad Hameed
 
Unit 5 Factoring and Forfaiting - FIS
Unit 5  Factoring and Forfaiting - FISUnit 5  Factoring and Forfaiting - FIS
Unit 5 Factoring and Forfaiting - FISVipul Kumar
 
Factoring
Factoring Factoring
Factoring Murali Raj
 
Session 6 merchandising1
Session 6 merchandising1Session 6 merchandising1
Session 6 merchandising1Arriz San Juan
 
Cambridge a level consignment accounts
Cambridge a level consignment accountsCambridge a level consignment accounts
Cambridge a level consignment accountsSanjaya Jayasundara
 
International Contracting And Import Finance 1
International Contracting And Import Finance 1International Contracting And Import Finance 1
International Contracting And Import Finance 1anshiiii
 
IAS 18
IAS 18IAS 18
IAS 18RS P
 
financial accounting
financial accounting  financial accounting
financial accounting Tiumhagos
 
Chapter20 International Finance Management
Chapter20 International Finance ManagementChapter20 International Finance Management
Chapter20 International Finance ManagementPiyush Gaur
 
QUESTION & ANSWER VOL III/THE SYSTEM OF ACCOUNTING
QUESTION & ANSWER VOL III/THE SYSTEM OF ACCOUNTINGQUESTION & ANSWER VOL III/THE SYSTEM OF ACCOUNTING
QUESTION & ANSWER VOL III/THE SYSTEM OF ACCOUNTINGAQEEL RAZA
 
Methods of international trade payments
Methods of international trade paymentsMethods of international trade payments
Methods of international trade payments1stChoicePharma
 

Similar to Advanced Financial Accounting - Volume 01 (20)

Factoring regulation act, 2011
Factoring regulation act, 2011Factoring regulation act, 2011
Factoring regulation act, 2011
 
12895730.ppt
12895730.ppt12895730.ppt
12895730.ppt
 
Factoring
FactoringFactoring
Factoring
 
Factoring & forfaiting
Factoring & forfaitingFactoring & forfaiting
Factoring & forfaiting
 
Consignment
ConsignmentConsignment
Consignment
 
Caiibfmwctlmoduled contd1 (1)
Caiibfmwctlmoduled contd1 (1)Caiibfmwctlmoduled contd1 (1)
Caiibfmwctlmoduled contd1 (1)
 
Unit 8 (tijarah)
Unit 8 (tijarah)Unit 8 (tijarah)
Unit 8 (tijarah)
 
Factoring
FactoringFactoring
Factoring
 
Unit 5 Factoring and Forfaiting - FIS
Unit 5  Factoring and Forfaiting - FISUnit 5  Factoring and Forfaiting - FIS
Unit 5 Factoring and Forfaiting - FIS
 
Factoring
Factoring Factoring
Factoring
 
Session 6 merchandising1
Session 6 merchandising1Session 6 merchandising1
Session 6 merchandising1
 
Cambridge a level consignment accounts
Cambridge a level consignment accountsCambridge a level consignment accounts
Cambridge a level consignment accounts
 
Factoring ppt
Factoring pptFactoring ppt
Factoring ppt
 
International Contracting And Import Finance 1
International Contracting And Import Finance 1International Contracting And Import Finance 1
International Contracting And Import Finance 1
 
IAS 18
IAS 18IAS 18
IAS 18
 
Factoring
FactoringFactoring
Factoring
 
financial accounting
financial accounting  financial accounting
financial accounting
 
Chapter20 International Finance Management
Chapter20 International Finance ManagementChapter20 International Finance Management
Chapter20 International Finance Management
 
QUESTION & ANSWER VOL III/THE SYSTEM OF ACCOUNTING
QUESTION & ANSWER VOL III/THE SYSTEM OF ACCOUNTINGQUESTION & ANSWER VOL III/THE SYSTEM OF ACCOUNTING
QUESTION & ANSWER VOL III/THE SYSTEM OF ACCOUNTING
 
Methods of international trade payments
Methods of international trade paymentsMethods of international trade payments
Methods of international trade payments
 

More from Rick Vogel

MBA Essay Writing Ideal Assignment Writing By Oxb
MBA Essay Writing Ideal Assignment Writing By OxbMBA Essay Writing Ideal Assignment Writing By Oxb
MBA Essay Writing Ideal Assignment Writing By OxbRick Vogel
 
SUPERHERO WRITING PAPER By FabFileFolders
SUPERHERO WRITING PAPER By FabFileFoldersSUPERHERO WRITING PAPER By FabFileFolders
SUPERHERO WRITING PAPER By FabFileFoldersRick Vogel
 
Business Letter Writing Basics
Business Letter Writing BasicsBusiness Letter Writing Basics
Business Letter Writing BasicsRick Vogel
 
The Introductory Paragraph
The Introductory ParagraphThe Introductory Paragraph
The Introductory ParagraphRick Vogel
 
Analytical Essay Analytical Academi
Analytical Essay Analytical AcademiAnalytical Essay Analytical Academi
Analytical Essay Analytical AcademiRick Vogel
 
Linkingwords Essay Transitions, Transitio
Linkingwords Essay Transitions, TransitioLinkingwords Essay Transitions, Transitio
Linkingwords Essay Transitions, TransitioRick Vogel
 
7 Ways To Make Your College Essays Stand Out
7 Ways To Make Your College Essays Stand Out7 Ways To Make Your College Essays Stand Out
7 Ways To Make Your College Essays Stand OutRick Vogel
 
Position Paper Format Mun Country Name At The Top O
Position Paper Format Mun Country Name At The Top OPosition Paper Format Mun Country Name At The Top O
Position Paper Format Mun Country Name At The Top ORick Vogel
 
IB Written Assignment Format Citation Writing
IB Written Assignment Format Citation WritingIB Written Assignment Format Citation Writing
IB Written Assignment Format Citation WritingRick Vogel
 
Welcome To The Most Sophisticated Custom Paper Writing Service. Our
Welcome To The Most Sophisticated Custom Paper Writing Service. OurWelcome To The Most Sophisticated Custom Paper Writing Service. Our
Welcome To The Most Sophisticated Custom Paper Writing Service. OurRick Vogel
 
Get Expert Tips To Teach Essay Writing To Your Chil
Get Expert Tips To Teach Essay Writing To Your ChilGet Expert Tips To Teach Essay Writing To Your Chil
Get Expert Tips To Teach Essay Writing To Your ChilRick Vogel
 
Ebook Great Writing 4 Great Essays - (4E) D.O.C
Ebook Great Writing 4 Great Essays - (4E) D.O.CEbook Great Writing 4 Great Essays - (4E) D.O.C
Ebook Great Writing 4 Great Essays - (4E) D.O.CRick Vogel
 
Why You Want To Be A Police Officer Essay. Why I
Why You Want To Be A Police Officer Essay. Why IWhy You Want To Be A Police Officer Essay. Why I
Why You Want To Be A Police Officer Essay. Why IRick Vogel
 
Pin By Sadia Abid On My Saves In 2021 Articles For Kids, Newspaper
Pin By Sadia Abid On My Saves In 2021 Articles For Kids, NewspaperPin By Sadia Abid On My Saves In 2021 Articles For Kids, Newspaper
Pin By Sadia Abid On My Saves In 2021 Articles For Kids, NewspaperRick Vogel
 
Buy My Essays Online - EssayPrince
Buy My Essays Online - EssayPrinceBuy My Essays Online - EssayPrince
Buy My Essays Online - EssayPrinceRick Vogel
 
Conclusion Of Research Paper Example. How To Make A Conclusion In
Conclusion Of Research Paper Example. How To Make A Conclusion InConclusion Of Research Paper Example. How To Make A Conclusion In
Conclusion Of Research Paper Example. How To Make A Conclusion InRick Vogel
 
Formal Analysis Essay Example. Definition And Ex
Formal Analysis Essay Example. Definition And ExFormal Analysis Essay Example. Definition And Ex
Formal Analysis Essay Example. Definition And ExRick Vogel
 
Why Essays Are A Bad Way To Teach Writing
Why Essays Are A Bad Way To Teach WritingWhy Essays Are A Bad Way To Teach Writing
Why Essays Are A Bad Way To Teach WritingRick Vogel
 
The Corporal Works Of Mercy
The Corporal Works Of MercyThe Corporal Works Of Mercy
The Corporal Works Of MercyRick Vogel
 
Sample Pdf Free Download Classles Democracy
Sample Pdf Free Download  Classles DemocracySample Pdf Free Download  Classles Democracy
Sample Pdf Free Download Classles DemocracyRick Vogel
 

More from Rick Vogel (20)

MBA Essay Writing Ideal Assignment Writing By Oxb
MBA Essay Writing Ideal Assignment Writing By OxbMBA Essay Writing Ideal Assignment Writing By Oxb
MBA Essay Writing Ideal Assignment Writing By Oxb
 
SUPERHERO WRITING PAPER By FabFileFolders
SUPERHERO WRITING PAPER By FabFileFoldersSUPERHERO WRITING PAPER By FabFileFolders
SUPERHERO WRITING PAPER By FabFileFolders
 
Business Letter Writing Basics
Business Letter Writing BasicsBusiness Letter Writing Basics
Business Letter Writing Basics
 
The Introductory Paragraph
The Introductory ParagraphThe Introductory Paragraph
The Introductory Paragraph
 
Analytical Essay Analytical Academi
Analytical Essay Analytical AcademiAnalytical Essay Analytical Academi
Analytical Essay Analytical Academi
 
Linkingwords Essay Transitions, Transitio
Linkingwords Essay Transitions, TransitioLinkingwords Essay Transitions, Transitio
Linkingwords Essay Transitions, Transitio
 
7 Ways To Make Your College Essays Stand Out
7 Ways To Make Your College Essays Stand Out7 Ways To Make Your College Essays Stand Out
7 Ways To Make Your College Essays Stand Out
 
Position Paper Format Mun Country Name At The Top O
Position Paper Format Mun Country Name At The Top OPosition Paper Format Mun Country Name At The Top O
Position Paper Format Mun Country Name At The Top O
 
IB Written Assignment Format Citation Writing
IB Written Assignment Format Citation WritingIB Written Assignment Format Citation Writing
IB Written Assignment Format Citation Writing
 
Welcome To The Most Sophisticated Custom Paper Writing Service. Our
Welcome To The Most Sophisticated Custom Paper Writing Service. OurWelcome To The Most Sophisticated Custom Paper Writing Service. Our
Welcome To The Most Sophisticated Custom Paper Writing Service. Our
 
Get Expert Tips To Teach Essay Writing To Your Chil
Get Expert Tips To Teach Essay Writing To Your ChilGet Expert Tips To Teach Essay Writing To Your Chil
Get Expert Tips To Teach Essay Writing To Your Chil
 
Ebook Great Writing 4 Great Essays - (4E) D.O.C
Ebook Great Writing 4 Great Essays - (4E) D.O.CEbook Great Writing 4 Great Essays - (4E) D.O.C
Ebook Great Writing 4 Great Essays - (4E) D.O.C
 
Why You Want To Be A Police Officer Essay. Why I
Why You Want To Be A Police Officer Essay. Why IWhy You Want To Be A Police Officer Essay. Why I
Why You Want To Be A Police Officer Essay. Why I
 
Pin By Sadia Abid On My Saves In 2021 Articles For Kids, Newspaper
Pin By Sadia Abid On My Saves In 2021 Articles For Kids, NewspaperPin By Sadia Abid On My Saves In 2021 Articles For Kids, Newspaper
Pin By Sadia Abid On My Saves In 2021 Articles For Kids, Newspaper
 
Buy My Essays Online - EssayPrince
Buy My Essays Online - EssayPrinceBuy My Essays Online - EssayPrince
Buy My Essays Online - EssayPrince
 
Conclusion Of Research Paper Example. How To Make A Conclusion In
Conclusion Of Research Paper Example. How To Make A Conclusion InConclusion Of Research Paper Example. How To Make A Conclusion In
Conclusion Of Research Paper Example. How To Make A Conclusion In
 
Formal Analysis Essay Example. Definition And Ex
Formal Analysis Essay Example. Definition And ExFormal Analysis Essay Example. Definition And Ex
Formal Analysis Essay Example. Definition And Ex
 
Why Essays Are A Bad Way To Teach Writing
Why Essays Are A Bad Way To Teach WritingWhy Essays Are A Bad Way To Teach Writing
Why Essays Are A Bad Way To Teach Writing
 
The Corporal Works Of Mercy
The Corporal Works Of MercyThe Corporal Works Of Mercy
The Corporal Works Of Mercy
 
Sample Pdf Free Download Classles Democracy
Sample Pdf Free Download  Classles DemocracySample Pdf Free Download  Classles Democracy
Sample Pdf Free Download Classles Democracy
 

Recently uploaded

Software Engineering Methodologies (overview)
Software Engineering Methodologies (overview)Software Engineering Methodologies (overview)
Software Engineering Methodologies (overview)eniolaolutunde
 
Grant Readiness 101 TechSoup and Remy Consulting
Grant Readiness 101 TechSoup and Remy ConsultingGrant Readiness 101 TechSoup and Remy Consulting
Grant Readiness 101 TechSoup and Remy ConsultingTechSoup
 
microwave assisted reaction. General introduction
microwave assisted reaction. General introductionmicrowave assisted reaction. General introduction
microwave assisted reaction. General introductionMaksud Ahmed
 
Measures of Dispersion and Variability: Range, QD, AD and SD
Measures of Dispersion and Variability: Range, QD, AD and SDMeasures of Dispersion and Variability: Range, QD, AD and SD
Measures of Dispersion and Variability: Range, QD, AD and SDThiyagu K
 
Introduction to Nonprofit Accounting: The Basics
Introduction to Nonprofit Accounting: The BasicsIntroduction to Nonprofit Accounting: The Basics
Introduction to Nonprofit Accounting: The BasicsTechSoup
 
IGNOU MSCCFT and PGDCFT Exam Question Pattern: MCFT003 Counselling and Family...
IGNOU MSCCFT and PGDCFT Exam Question Pattern: MCFT003 Counselling and Family...IGNOU MSCCFT and PGDCFT Exam Question Pattern: MCFT003 Counselling and Family...
IGNOU MSCCFT and PGDCFT Exam Question Pattern: MCFT003 Counselling and Family...PsychoTech Services
 
Sanyam Choudhary Chemistry practical.pdf
Sanyam Choudhary Chemistry practical.pdfSanyam Choudhary Chemistry practical.pdf
Sanyam Choudhary Chemistry practical.pdfsanyamsingh5019
 
Advance Mobile Application Development class 07
Advance Mobile Application Development class 07Advance Mobile Application Development class 07
Advance Mobile Application Development class 07Dr. Mazin Mohamed alkathiri
 
Disha NEET Physics Guide for classes 11 and 12.pdf
Disha NEET Physics Guide for classes 11 and 12.pdfDisha NEET Physics Guide for classes 11 and 12.pdf
Disha NEET Physics Guide for classes 11 and 12.pdfchloefrazer622
 
APM Welcome, APM North West Network Conference, Synergies Across Sectors
APM Welcome, APM North West Network Conference, Synergies Across SectorsAPM Welcome, APM North West Network Conference, Synergies Across Sectors
APM Welcome, APM North West Network Conference, Synergies Across SectorsAssociation for Project Management
 
Arihant handbook biology for class 11 .pdf
Arihant handbook biology for class 11 .pdfArihant handbook biology for class 11 .pdf
Arihant handbook biology for class 11 .pdfchloefrazer622
 
Beyond the EU: DORA and NIS 2 Directive's Global Impact
Beyond the EU: DORA and NIS 2 Directive's Global ImpactBeyond the EU: DORA and NIS 2 Directive's Global Impact
Beyond the EU: DORA and NIS 2 Directive's Global ImpactPECB
 
Kisan Call Centre - To harness potential of ICT in Agriculture by answer farm...
Kisan Call Centre - To harness potential of ICT in Agriculture by answer farm...Kisan Call Centre - To harness potential of ICT in Agriculture by answer farm...
Kisan Call Centre - To harness potential of ICT in Agriculture by answer farm...Krashi Coaching
 
Interactive Powerpoint_How to Master effective communication
Interactive Powerpoint_How to Master effective communicationInteractive Powerpoint_How to Master effective communication
Interactive Powerpoint_How to Master effective communicationnomboosow
 
Student login on Anyboli platform.helpin
Student login on Anyboli platform.helpinStudent login on Anyboli platform.helpin
Student login on Anyboli platform.helpinRaunakKeshri1
 
General AI for Medical Educators April 2024
General AI for Medical Educators April 2024General AI for Medical Educators April 2024
General AI for Medical Educators April 2024Janet Corral
 
SOCIAL AND HISTORICAL CONTEXT - LFTVD.pptx
SOCIAL AND HISTORICAL CONTEXT - LFTVD.pptxSOCIAL AND HISTORICAL CONTEXT - LFTVD.pptx
SOCIAL AND HISTORICAL CONTEXT - LFTVD.pptxiammrhaywood
 

Recently uploaded (20)

Software Engineering Methodologies (overview)
Software Engineering Methodologies (overview)Software Engineering Methodologies (overview)
Software Engineering Methodologies (overview)
 
Grant Readiness 101 TechSoup and Remy Consulting
Grant Readiness 101 TechSoup and Remy ConsultingGrant Readiness 101 TechSoup and Remy Consulting
Grant Readiness 101 TechSoup and Remy Consulting
 
microwave assisted reaction. General introduction
microwave assisted reaction. General introductionmicrowave assisted reaction. General introduction
microwave assisted reaction. General introduction
 
Measures of Dispersion and Variability: Range, QD, AD and SD
Measures of Dispersion and Variability: Range, QD, AD and SDMeasures of Dispersion and Variability: Range, QD, AD and SD
Measures of Dispersion and Variability: Range, QD, AD and SD
 
Introduction to Nonprofit Accounting: The Basics
Introduction to Nonprofit Accounting: The BasicsIntroduction to Nonprofit Accounting: The Basics
Introduction to Nonprofit Accounting: The Basics
 
Código Creativo y Arte de Software | Unidad 1
Código Creativo y Arte de Software | Unidad 1Código Creativo y Arte de Software | Unidad 1
Código Creativo y Arte de Software | Unidad 1
 
IGNOU MSCCFT and PGDCFT Exam Question Pattern: MCFT003 Counselling and Family...
IGNOU MSCCFT and PGDCFT Exam Question Pattern: MCFT003 Counselling and Family...IGNOU MSCCFT and PGDCFT Exam Question Pattern: MCFT003 Counselling and Family...
IGNOU MSCCFT and PGDCFT Exam Question Pattern: MCFT003 Counselling and Family...
 
Sanyam Choudhary Chemistry practical.pdf
Sanyam Choudhary Chemistry practical.pdfSanyam Choudhary Chemistry practical.pdf
Sanyam Choudhary Chemistry practical.pdf
 
Advance Mobile Application Development class 07
Advance Mobile Application Development class 07Advance Mobile Application Development class 07
Advance Mobile Application Development class 07
 
Disha NEET Physics Guide for classes 11 and 12.pdf
Disha NEET Physics Guide for classes 11 and 12.pdfDisha NEET Physics Guide for classes 11 and 12.pdf
Disha NEET Physics Guide for classes 11 and 12.pdf
 
APM Welcome, APM North West Network Conference, Synergies Across Sectors
APM Welcome, APM North West Network Conference, Synergies Across SectorsAPM Welcome, APM North West Network Conference, Synergies Across Sectors
APM Welcome, APM North West Network Conference, Synergies Across Sectors
 
Arihant handbook biology for class 11 .pdf
Arihant handbook biology for class 11 .pdfArihant handbook biology for class 11 .pdf
Arihant handbook biology for class 11 .pdf
 
Mattingly "AI & Prompt Design: The Basics of Prompt Design"
Mattingly "AI & Prompt Design: The Basics of Prompt Design"Mattingly "AI & Prompt Design: The Basics of Prompt Design"
Mattingly "AI & Prompt Design: The Basics of Prompt Design"
 
Beyond the EU: DORA and NIS 2 Directive's Global Impact
Beyond the EU: DORA and NIS 2 Directive's Global ImpactBeyond the EU: DORA and NIS 2 Directive's Global Impact
Beyond the EU: DORA and NIS 2 Directive's Global Impact
 
Kisan Call Centre - To harness potential of ICT in Agriculture by answer farm...
Kisan Call Centre - To harness potential of ICT in Agriculture by answer farm...Kisan Call Centre - To harness potential of ICT in Agriculture by answer farm...
Kisan Call Centre - To harness potential of ICT in Agriculture by answer farm...
 
Interactive Powerpoint_How to Master effective communication
Interactive Powerpoint_How to Master effective communicationInteractive Powerpoint_How to Master effective communication
Interactive Powerpoint_How to Master effective communication
 
Student login on Anyboli platform.helpin
Student login on Anyboli platform.helpinStudent login on Anyboli platform.helpin
Student login on Anyboli platform.helpin
 
Mattingly "AI & Prompt Design: Structured Data, Assistants, & RAG"
Mattingly "AI & Prompt Design: Structured Data, Assistants, & RAG"Mattingly "AI & Prompt Design: Structured Data, Assistants, & RAG"
Mattingly "AI & Prompt Design: Structured Data, Assistants, & RAG"
 
General AI for Medical Educators April 2024
General AI for Medical Educators April 2024General AI for Medical Educators April 2024
General AI for Medical Educators April 2024
 
SOCIAL AND HISTORICAL CONTEXT - LFTVD.pptx
SOCIAL AND HISTORICAL CONTEXT - LFTVD.pptxSOCIAL AND HISTORICAL CONTEXT - LFTVD.pptx
SOCIAL AND HISTORICAL CONTEXT - LFTVD.pptx
 

Advanced Financial Accounting - Volume 01

  • 1. See discussions, stats, and author profiles for this publication at: https://www.researchgate.net/publication/267151510 Advanced Financial Accounting - Volume 01 Book · January 2014 DOI: 10.13140/2.1.3194.1126 CITATIONS 0 READS 38,436 1 author: Some of the authors of this publication are also working on these related projects: Corporate Governance Practices, Customer centric approach and Firm performance View project Corporate Governance Mechanism and Working capital Management:a study of Selected Listed Companies in Sri Lanka View project Alagathurai Ajanthan University of Jaffna 12 PUBLICATIONS 19 CITATIONS SEE PROFILE All content following this page was uploaded by Alagathurai Ajanthan on 21 October 2014. The user has requested enhancement of the downloaded file.
  • 2. 1 | P a g e A.Ajanthan Joint Venture & Consignment Accounts CONSIGNMENT ACCOUNTS Consignment The sales activity of any business can be organized in different ways. With the customers spread all over, the business entity cannot afford to have only minimum selling points nor can it have its own resources to have the outlets all over. The business volumes cannot be limited in any case. The core competence of a manufacturing company is to produce a good quality product. It creates a network of its own outlets, dealers, commission agents, institutions etc to distribute its products efficiently and effectively. Thus the selling may be handled directly through own salesmen or indirectly through agents. In case of direct selling, the company usually has depots all over. The stocks are transferred to these depots and from their finally sold to ultimate customers. This involves huge expenses and problems of maintaining the same on a permanent basis. Hence, the firm could appoint agents to whom stocks will be given. These agents distribute the products to ultimate customers and receive commission from the manufacturer. One such way of indirect selling is selling through consignment agents. The relationship between consignor and consignee is that of Principal-Agent relationship. Consignment takes place where goods are transferred from the owner (consignor) to an agent (consignee) for the purpose of sale by the consignee on behalf of the consignor. It is important to understand that the relationship of principal (consignor) and agent (consignee) exists. Because of this agency relationship, ownership of the goods does not transfer to the consignee. The consignee, as the selling agent, is entitles to a commission for selling the goods; expenses may be incurred by both parties; and periodically or on completion of the consignment, settlement is effected between the parties. If any goods remain unsold then they are generally returned to the consignor. Consignment is a fairly common commercial transaction, perhaps more common than many people may think. Examples include: ï‚· A manufacturer supplies stock of a new product on consignment to a local distributor. ï‚· A primary producer forwards produce on consignment to an agent. ï‚· A car sales yard in a prominent position may accept motor vehicles on consignment from other motor dealers or from the general public.
  • 3. 2 | P a g e A.Ajanthan Joint Venture & Consignment Accounts Main Terms of Consignment Trade Consignment: The transfer of goods by one party called a consignor, to another party called the consignee, to be sold by the latter on behalf of the former. The ownership of the goods is retained by the consignor while the possession of the goods is transferred to the consignee. The consignment is outward consignment for the person who sends the goods and an inward consignment for the person who receives the goods for sale. Consignor: The party who sends the goods to agents for sale, e.g., a manufacturer or whole seller. Consignee: The party to whom the goods are sent for sale. Ordinary Commission or Consignee’s Remuneration: When the goods are sold by the consignee, he is paid a commission for his services at a fixed rate on the proceeds of the goods sold by him. In addition to this commission, he is to be reimbursed for all expenses incurred by him in connection with the consignment sales. Usually these expenses are in the nature of dock charges, custom duties, carriage, godown rent, advertisement, insurance of the goods while in his possession etc. Del Credere Commission: This is additional commission payable to the consignee for taking over additional responsibility of collecting money from customers. Usually the consignor advises the consignee to sell the goods consigned to him for cash only, because if such goods are sold on credit by the consignee and if any amount becomes irrecoverable from the debtors the loss will fall upon the consignor as the consignee acted as an agent only in effecting the sales, he does not become responsible for any debts. But sometimes an arrangement is made between the consignor and the consignee whereby the later guarantees payment and undertakes responsibility for bad debts. For this the consignee receives an additional commission known as ‗‗del credere commission’’ on the total sales. When del-credere commission is given to the consignee, the consignee will make payment to the consignor, whether he himself receives the payment or not from the purchaser(s). Overriding Commission: This type of commission is allowed to the consignee in addition to the normal commission (as distinct from del credere commission). The idea seems to be to provide addition incentive to the consignee for the purpose of creating market for new products. Account Sales: This is a summary of the transactions of the consignee. It is a means of conveying information to the consignor and shows the gross proceeds of sale of the goods,
  • 4. 3 | P a g e A.Ajanthan Joint Venture & Consignment Accounts expenses incurred by the consignee, commission due and the net amount owing to the consignor. The following example shows a specimen of an account sale (Account sales of 100 Sony Radios consigned to Mayuran Traders, Colombo by Alagu Traders, Jaffna. Particulars Amount (LKR) Sale Proceeds: 100 Radios sold at Lkr 9000 each Less: Expenses: Freight Carriage Godown rent and selling expenses Less: Commission @10% on sale proceeds (18,00,000 × 10/100) Less: Advance (Bank Draft) Balance due to Alagu traders remitted 5,000 2,100 4,300 18,00,000 (11,400) 17,88,600 (1,80,000) 16,08,600 (2,00,000) 14,08,000 E & O.E. For Mayuran Traders Colombo MAYURAN 31st January, 2010 Managing Partner Advance against Consignment: Until the goods are sold by the consignee, he is not indebted to the consignor and is not expected to pay for them. This results in a part of the consignor's Capital being locked up for a period. To overcome his difficulty, the consignee often remits a sum of money in advance to the consignor. This may be done in the form of an acceptance of a bill of exchange drawn by the consignor on the Consignee or a simple bank draft. An advance is readily sent against consignment by the consignee to the consignor when the consignment goods have become popular in the consignee‘s place. Pro-forma Invoice: When goods are consigned to an agent they are generally accompanied by a document called a ‗Pro-forma invoice’ giving indication of the price of the goods at which the consignee ought to sell the goods. Pro-Forma Invoice is a statement which is similar to that of an invoice, but it is called proforma because it does not make the consignee responsible to pay the amount named therein.
  • 5. 4 | P a g e A.Ajanthan Joint Venture & Consignment Accounts The consignor generally mentions a higher price than his cost so that consignee does not know the profit of the consignor. Difference between Invoice and Account sales Account sales Invoice Prepared by the consignee. Prepared by the seller. All expenses and commission are deducted in account sales. In invoice, expenses are added but discount and commission are deducted. All expenses incurred by the consignee are borne by the consignor. After sale, expenses are paid by the buyer. The relationship between two parties remains as principal and agent. The relationship between two parties is that of debtor and creditor. Operating Cycle of Consignment Arrangement i. Goods are sent by consignor to the consignee ii. Consignee may pay some advance or accept a bill of exchange iii. Consignee will incur expenses for selling the goods iv. Consignee maintains records of all cash and credit sale. v. Consignee prepares a summary of results called as Account sales vi. Consignor pays commission to the consignee Features of Consignment The following are the salient features of consignment: 1. Objects: Goods are forwarded by the consignor to the consignee with an objective of sale at a profit. 2. Ownership: In consignment, the consignee does not buy the goods. He merely undertakes to sell them on behalf of the consignor. Hence, the ownership in the goods remains with consignor till it is sold by the consignee. 3. Relationship: The relationship between the consignor and the consignee is that of a principal and an agent, and not of a debtor and creditor. An agent becomes in debited for amounts realized on behalf of the principal. 4. Risk: The consignor should bear all the risks connected with the goods until it is sold.
  • 6. 5 | P a g e A.Ajanthan Joint Venture & Consignment Accounts 5. Expenses: As consignment is not a sale, whatever the consignee does is on behalf of the consignor. Thus, the consignor should reimburse all legitimate expenses incurred by the consignee for selling and receiving the goods. 6. Stock of goods: Any stock remaining unsold with the consignee belongs to the consignor. 7. Commission: The consignee agrees to sell the goods for an agreed rate of commission. He is therefore, allowed to deduct his commission due from the sale proceeds. 8. Possession: The goods will be in the possession of the consignee until it is sold on behalf of the consignor. 9. Repossession: The consignor can repossess the goods from the consignee at any time. 10. Profit or loss: Since the consignee acts on behalf of the consignor, the profit or loss on sale of goods belongs to the consignor. Difference between Sale and Consignment 1. When goods are sold by one to another, the property in the goods immediately passes to the buyer, whereas when goods are sent on consignment, the property in the goods remains with the consignor. Only the possession is transferred to the consignee. 2. When goods are sold by one to another, it becomes a relationship of a buyer and seller or a debtor and a creditor between the two persons, whereas when goods are consigned by one to another, it becomes a relationship of a principal and an agent between the consignor and the consignee. 3. When goods are sold, the buyer cannot return the goods to the seller whereas when goods are sent on consignment the goods are returnable, if they remain unsold. 4. The risk in the goods is not transferred to the consignee despite the transfer of possession of goods. Any damage or loss to the goods is therefore borne by consignor. But in the case of sale, the risk is immediately transferred to the buyer even when the goods are still in the possession of the seller. 5. The expenses, in respect of freight, cartage, insurance, etc. Are met by the consignor in a consignment transaction, but in the case of sale the expense are borne by the purchaser unless otherwise provided in the agreement. 6. The transfer of possession (i.e. Delivery of goods) is essential in a consignment transaction. In a sale, however, the goods may be delivered at a later date. The consignee will be treated as a debtor only when goods or part of them have been sold by him.
  • 7. 6 | P a g e A.Ajanthan Joint Venture & Consignment Accounts But if goods remain unsold, the consignee will send them back to the Consignor and the Consignor will pay the Consignee all the expenses he has incurred in keeping the goods in safety and in attempting to push the goods in the market. Expenses on Consignment i. Non-recurring expenses: The expenses which do not arise repeatedly for a particular consignment are called non-recurring expenses. Non-recurring expenses are incurred for bringing goods to the godown of the consignee. Such expenses are generally incurred on the consignment as a whole. The non-recurring expenses are incurred partly by the consignor and partly by the consignee. The consignor usually incurs expenses, such as packing, cartage, loading charges, freight, etc., on sending the goods to the consignee. But the consignee usually incurs expenses, such as dock dues, customs duty, clearing charges, etc., on receiving the goods from the consignor. ii. Recurring expenses: The indirect expenses incurred repeatedly on the same consignment are called recurring expenses. Recurring expenses are incurred after the goods have reached the consignee‘s place or godown. Advertising, discount on bills, commission on collection of cheques, travelling expenses of salesman, bad debts, etc., are some examples of recurring expenses incurred by the consignor. On the other hand, godown rent, godown insurance, sales promotion, etc., are the examples of recurring expenses incurred by the consignee. Accounting for Consignment Business The consignor and consignee keep their own books of accounts. The consignor may send goods to many consignees. Also, a consignee may act as agent for many consignors. It is appropriate that both of them would want to know profit or loss made on each consignment. Books of the Consignor The transactions relating to ach consignment are recorded in such a way that the profit or loss of each consignment can be ascertained separately. It requires the preparation of a special account known as consignment account. A consignment account is a nominal account prepared to find out the profit or loss of a consignment. The account is debited with the cost of goods sent, expenses incurred by the consignor and consignee, and the commission due to the consignee. But the account is credited with the amount of sales affected and also with closing stock, if any. The balance of this account is either profit or loss.
  • 8. 7 | P a g e A.Ajanthan Joint Venture & Consignment Accounts In addition to the consignment account, the consignor also prepares the personal account of the consignee to ascertain the amount due by the consignee. This account is debited with the amount of sales affected by the consignee and credited with the amount of any advance received from him, expenses incurred by him and commission payable on sales. The balance in this account is the amount due by the consignee. Let us see the entries in the books of consignor as well as consignee. Situations Consignor’s Books On sending goods Consignment A/c …………...Dr Goods Sent on Consignment….……Cr On expenses for sending goods (by the consignor) Consignment A/c ……..….....Dr Cash/ Bank A/c………………....…. Cr On an advance made by the consignee Cash / Bank / Bills Receivable A/c ……Dr Consignee‘s Personal A/c…...…..Cr Bills received from the consignee discounted with the bank Bank A/c ……..….....Dr Discount A/c ….........Dr Bills Receivable A/c……….……Cr On expenses incurred by consignee Consignment A/c …………...Dr Consignee‘s Personal A/c……….…Cr On sales made by the consignee Consignee‘s Personal A/c .….Dr Consignment A/c………………..... Cr For consignee‘s commission Consignment A/c …………...Dr Consignee‘s Personal A/c…….…... Cr Goods returned by the consignee Goods Sent on Consignment...Dr Consignment A/c ……………….....Cr Bad debts incurred (when a consignee is entitled to del credere commission, no entry for bad debts is to be passed as such a loss is to be borne by the consignee himself. Otherwise, the loss on account of bad debts should be borne by the consignor. Consignment A/c …………....Dr Consignee‘s Personal A/c…….…... Cr Remittance by the consignee in full settlement Cash / Bank / Bills Receivable A/c ……Dr
  • 9. 8 | P a g e A.Ajanthan Joint Venture & Consignment Accounts Consignee‘s Personal A/c…….…... Cr Profit or loss on consignment (a) If there is a profit (b) If there is a loss Consignment A/c …………....Dr Profit and loss A /c………………...Cr Profit and loss A /c…………Dr Consignment A/c …………….…....Cr Closing entry for goods sent on consignment Goods Sent on Consignment…Dr Trading A / c………………….…….Cr On closing stock/ unsold Stock with the consignee Consignment Stock A/c ........Dr Consignment A/c………………..… Cr The Consignment account in the books of consignor will ultimately show the net profit or loss on account of consignment business. It must be noted that a separate consignment account must be opened for different agents. This will enable him to know profit or loss on each consignment. Books of the Consignee Situations Consignee’s Books Goods received from the consignor No Entry Expenses incurred by the consignor No Entry Advance made by the consignee Consignor‘s Personal A/c...Dr Bank / Cash / Bills Payable A/c………Cr Bill discounted by the consignor with the bank No Entry Sales of goods by the consignee Cash A / c (cash sales)….…Dr Consignment debtors A / c (credit sales)……………...Dr Consignor‘s Personal A/c.....................Cr Expenses incurred by the consignee Consignor‘s Personal A/c...Dr Cash/ Bank A/c…….…………....….. .Cr Commission due to the consignee Consignor‘s Personal A/c...Dr Commission A/c……………..…..…...Cr Return of goods to the consignor No Entry Payment received from debtors Cash/ Bank A/C…………. Dr Consignment debtors A/C……..……..Cr Bad debts incurred
  • 10. 9 | P a g e A.Ajanthan Joint Venture & Consignment Accounts (a) In case consignee does not get del credere commission, all bad debts have to be borne by the consignor himself. (b) In case del credere commission is paid to the consignee, bad debts are to be borne by him. When the bills payable accepted in favor of the consignor is met on the due date Consignor‘s Personal A/c...Dr Consignment debtors A/C…….……...Cr Bad debts A /c…………….Dr Consignment debtors A/C…………....Cr Bills payable A /c………...Dr Bank A/c…….………………….….....Cr Unsold stock in possession of the consignee No Entry Profit or loss on consignment No Entry *Note: The discount on bills may be accounted for in one of two ways; ï‚· As a normal operating expenses item and charged against the profit and loss account; or ï‚· As a special expense item related to the consignment and therefore charged to the consignment account. The method of accounting depends on whether the advance is interpreted as a method of financing the business generally or whether it is regarded as a transaction particularly related to the consignment activity. Format of Consignment Account LKR LKR To Consignment stock (opening balance if any) xxx By Consignee‘s Personal Account (Amount of gross proceeds (sales) realized by the Consignee) Xxxx To Goods Sent on Consignment xxxx By Goods Sent on Consignment (Difference in cost of goods sent and the proforma Invoice price) Xx To Cash/bank (Expenses incurred by the consignor) xx By Abnormal loss (Whether insured or not) Xxx To Consignee's Personal Account (Expenses paid by the Consignee– total amount) (Commission, including del-credere payable to the xx By Goods sent on Consignment (Returned by the Consignee) Xx
  • 11. 10 | P a g e A.Ajanthan Joint Venture & Consignment Accounts consignee) To Stock Reserve (Difference in the value of closing stock marked at Pro-forma invoice or loaded price & cost price) xx By Stock Reserve (Difference between the cost and pro-forma invoice price on the opening balance of consignment) Xx To Goods Sent on Consignment (Difference between cost price and Pro-forma invoice price on the goods returned by the consignee) xx By General Profit and Loss Account (For consignment loss) xx To General Profit and Loss Account (For Consignment profit) xxx Illustration: 1 Aju stores of Jaffna consigned on 1st January, 2010, 50 cases of goods at Lkr.200 each to Riyash Traders of Warakkapola for sale on commission at 10% on gross sales. Aju stores paid Lkr.500 for packing, freight and insurance. Riyash Traders took delivery of the goods on 11th January, 2010, after accepting a 15 days bill for Lkr. 5,000 and paid Lkr. 150 for carriage. They sold 40 cases of goods @ Lkr. 250 and balance for Lkr. 260 each. Their sales expenses amounted to Lkr. 200. On 31st January, 2005, Riyash Traders forwarded an account sale together with a draft for the balance. Prepare account sales rendered by Riyash Traders and also give journal entries and ledger accounts in the books of Aju stores and Riyash Traders. Solution: Account sales of 50 cases of goods received and sold on behalf of Aju stores, Jaffna. Particulars Amount(LKR) Sale Proceeds: 40 cases sold at Lkr 250 each 10 cases sold at Lkr 260 each Less: Expense: Carriage Sales expenses Commission @ 10% Net proceeds 10,000 2,600 150 200 1,260 12,600 (1,610) 10,990
  • 12. 11 | P a g e A.Ajanthan Joint Venture & Consignment Accounts Less: Advance (Bill) Balance sent by Draft (5,000) 5,990 Books of Aju Stores (Consignor) Journal Entries Description Dr (LKR) Cr (LKR) 1. Consignment to Warakkapola A/c Goods sent on Consignment A/c (Sent goods on consignment to Riyash Traders, Warakkapola) 10,000 10,000 2. Consignment to Warakkapola A/c Bank A/c (Expenses incurred on the Consignment) 500 500 3. Bill receivable A/c Riyash Traders A/c (Advance received from the Agent in the form of Bill) 5,000 5,000 5. Consignment to Warakkapola A/c Riyash Traders A/c (paid carriage and sales expenses by consignee) 350 350 6. Bank A/c Bills receivable A/c (The bill met on due date) 5,000 5,000 4. Riyash Traders A/c Consignment to Warakkapola A/c (Gross sale proceeds as per Account Sales) 12,600 12,600 7. Consignment to Warakkapola A/c Riyash Traders A/c (Commission on gross sales payable @ 10%) 1,260 1,260 8. Consignment to Warakkapola A/c Profit and Loss A/c (Transferred profit on consignment to profit and loss A/c) 490 490 9. Bank A/c Riyash Traders A/c (Amount received in draft along with account sales) 5,990 5,990
  • 13. 12 | P a g e A.Ajanthan Joint Venture & Consignment Accounts 10. Goods sent on Consignment A/c Trading A/c (Goods sent on consignment A/c closed by transfer to trading A/c) 10,000 10,000 Ledgers Consignment to Warakkapola Account Dr Cr Goods sent on Consignment A/c Bank A/c (expenses) Riyash Traders A/c Carriage 150 Sales expenses 200 Riyash Traders: Commission P & L A/c (Transfer) 10,000 500 350 1,260 490 Riyash Traders A/c (Sale proceeds) 12,600 12,600 12,600 Riyash Traders A/c Dr Cr Consignment to Warakkapola A/c 12,600 Bill Receivable A/c Consignment to Warakkapola A/c Consignment to Warakkapola A/c Bank A/c 5,000 350 1260 5,990 12,600 12,600 Bill Receivable A/c Dr Cr Riyash Traders A/c 5,000 Bank A/c 5,000 Goods sent on Consignment Account Dr Cr Trading A/c (Transfer) 10,000 Consignment to Warakkapola A/c 10,000
  • 14. 13 | P a g e A.Ajanthan Joint Venture & Consignment Accounts Profit & Loss A/c Dr Cr Consignment to Warakkapola A/c 490 Books of Riyash Traders (Consignee) Journal Entries Description Dr (LKR) Cr (LKR) 1. Aju stores A/c Bank / Cash A/c (Paid expenses on the Consignment received) 350 350 2. Aju stores A/c Bills payable A/c (Acceptance of bill drawn against the consignment) 5,000 5,000 3. Bills payable A/c Bank A/c (The bill met on due date) 5,000 5,000 4. Bank A/c Aju stores A/c (Sales effected for the Consignment received) 12,600 12,600 5. Aju stores A/c Commission A/c (Commission receivable on the goods sold) 1,260 1,260 6. Aju stores A/c Bank A/c (Amount remitted as final settlement) 5,990 5,990 Ledgers Aju Stores A/c Dr Cr Bank A/c (Expenses) Bills payable A/c Commission A/c Bank A/c (amount remitted) 350 5,000 1,260 5,990 Bank A/c (Sale proceeds) 12,600
  • 15. 14 | P a g e A.Ajanthan Joint Venture & Consignment Accounts 12,600 12,600 Bills payable A/c Dr Cr Bank A/c 5000 Aju stores A/c 5,000 Commission A/c Dr Cr Aju stores A/c 1,260 Unsold Stock at Balance Sheet Date Where all consigned goods are not sold by the end of the consignor‘s financial year, it is necessary to obtain an ‘account sales’ from the consignee. This is a document detailing particulars of transactions and the quantity of unsold stock on hand at that date. Unsold stock is valued to enable the profit on the consignment up to balance sheet date to be ascertained, and included with revenue from other trading activities. The basis for valuation of this stock is cost price unless deterioration or obsolescence requires the adoption of net realizable value. Determination of cost price involves a consideration not only of the original purchase price of the goods but also of any expenses in transporting the goods to the place of sale-the consignee‘s store. Thus it is proper to include the following in valuing unsold stock: (carriage and freight, loading charges, customs duty, clearing charges, dock dues, carriage paid up to the godown and unloading charges). ï‚· Purchase price ï‚· Inward charges to the consignor‘s place of business ï‚· Outward charges related to the dispatch to the consignee ï‚· Inward charges incurred by the consignee Expenses incurred by the consignee in selling the goods such as advertisement, salesman‘s salaries and commission, storage, insurance against fire or theft are not included in the valuation of unsold stock. These expenses do not relate to the goods unsold and are recorded as marketing expenses. In other words it can be said that all direct expense or all expenses made whether by the consignor or by the consignee in placing the goods in a saleable condition (all expenses till the goods reach the godown of the consignee) will be taken into account while valuing the closing stock. In most cases the amount of
  • 16. 15 | P a g e A.Ajanthan Joint Venture & Consignment Accounts expenses of both the consignor and the consignee to be added to purchase price is calculated as a proportion of the total relevant expenses of the whole consignment. The balance of consignment stock account is a current asset (appears in the asset side of Balance Sheet). At the commencement of the next financial period, consignment stock will be transferred to the consignment account, as a debit to enable the profit or loss on the sale of the remainder of the consignment to be determined. Illustration: 2 Suppose the Consignor sends to the Consignee, 2,000 Samsung mobile at Lkr.40 per unit and pays Costa duty, Lkr.3, 000; marine insurance, Lkr.1, 500. The Consignee pays, at the time of taking delivery, unloading charges of Lkr.500. The Consignee also pays godown rent Lkr.450 and advertisement Lkr.1, 500.if you assume that 400 Samsung mobile remain unsold, the value of its will be calculated as follows; LKR 400 Samsung mobile, i.e., 400 @ Rs.40 16,000 1/5th of Lkr.3,000, Costa duty 600 1/5th of Lkr.1,500, Marine Insurance 300 1/5th of Lkr.500, unloading charges paid by the Consignee 100 Total value of unsold Stock 17,000 The rule regarding valuation is cost or market price whichever is lower. In the market price of the unsold stock is more than Rs.17, 000, it will be valued at Rs.17, 000. If however, the market price is less than Rs.17, 000, it will be valued at the market price. Any loss or depreciation of stock should be duly taken into account. The unsold stock valued in the above manner will now be brought into books by passing an entry, as Consignment Stock A/c ……………..Dr Consignment A/c………………………….…. Cr Note: If the pro-forma invoice was made out at a price higher than the cost, stock will also be valued at invoice and not at cost. But it is wrong to show unsold stock in Balance Sheet at a figure higher than the cost. Hence for the difference (i.e., difference between value of stock at invoice price and value of stock at cost) reserve must be created, entry is as follows; Consignment A/c ……………………..Dr Stock Reserve A/c……………………….….. Cr
  • 17. 16 | P a g e A.Ajanthan Joint Venture & Consignment Accounts Illustration: 3 Y consigns goods to X valued at 8000 cost price. Expenses incurred by Y are: freight 40; insurance 100; cartage 20.Commission is allowed at 5% on sales. An advance of 5000 is made by the consignee. X incurs the following expenses: duty 80; cartage inward 40; advertising 200; and cash sales amounted to 7600. At balance date one-quarter of the goods are unsold. Calculate the value of unsold goods. LKR ¼ th of cost price (Lkr 8,000) 2,000 ¼ th of consignor‘s expenses [Lkr 160 (freight 40; insurance 100; cartage 20)] 40 ¼ th of consignee‘s relevant expenses [Lkr 120 (duty 80; cartage inward 40)] Total value of unsold Stock 30 2,070 Illustration: 4 Ramu of Cochin consigned goods of the cost of Lkr.10000 to his agent, Ajith of Agra and incurred Lkr.2000 for packing, forwarding and freight. Ajith took delivery of the goods after spending Lkr.3000 for duty and clearing charges. He sold 3 / 4th of the goods for Lkr.15000 for which he was entitled to a commission of 5%. His sales expenses amounted to Lkr.300. Prepare consignment account after showing the valuation of unsold stock. Solution: Valuation of stock: LKR Cost of stock at pro forma invoice = 10,000 * ¼ 2500 Add: proportionate non-recurring expenses: Incurred by Ramu 2,000 Incurred by Ramu 3,000 5,000 * ¼ 1,250 Value of stock 3,750 Consignment to Agra A/c Dr Cr Goods sent on Consignment A/c Bank A/c (Packing charges) Ajith A/c (Duty + selling charges) Ajith A/c: Commission P & L A/c (Transfer) 10,000 2,000 3,300 750 2,700 Ajith A/c (Sale proceeds) Stock on consignment 15,000 3,750
  • 18. 17 | P a g e A.Ajanthan Joint Venture & Consignment Accounts 18,750 18,750 Del Credere Commission and Bad Debts Sometimes the consignor allows an extra commission to the consignee in order to cover the risk of collection from customer on account of credit sales which is known as Del Credere Commission. Naturally, if debt is found to be irrecoverable the same must be borne by the consignee. There will be no effect in the books of consignor. In short, credit sales will be treated as cash sales to consignor. If no Del credere commission is given by the consignor to the consignee, the amount of Bad debts must be borne by the consignor. When there is no Del credere Commission; In the books of consignor: Consignment A/c………………..Dr Consignee‘s A/c……………………Cr In the books of consignee: Consignor‘s A/c………………….Dr Debtors A/c………………………...Cr Illustration: 5 Prasana Furniture‘s, Palghat consigned 100 chairs of Lkr 800 each to their agent Sudharaka Furniture‘s at Kelaniya for sale on commission at 5% on gross sale effected. Expenses at Palghat were Lkr.1500 for carriage and Lkr.1000 for insurance. Sudharaka Furniture‘s took delivery of the chairs after accepting a three-month bill for Lkr.40000 drawn against the consignment, which the consignor discounted for Lkr.38000. The consignees paid Lkr.150 for loading and unloading and Lkr. 600 for freight and carriage. They sold 70 chairs @ Lkr.850 for cash and 10 chairs on credit @ Lkr. 1000. A customer who bought two chairs became insolvent and nothing could be recovered from him. The balance of debt was fully collected. The sales expense of Sudharaka Furniture‘s amounted to Lkr. 360. Required to prepare ledger accounts in the books of consignor and journal entries in the books consignee
  • 19. 18 | P a g e A.Ajanthan Joint Venture & Consignment Accounts Solution: Ledger accounts (in Prasana Furniture’s) Consignment to Kelaniya A/c Dr Cr Goods sent on Consignment A/c Bank A/c (expenses) Sudharaka Furniture‘s A/c Sudharaka Furniture‘s A/c (Commission) Sudharaka Furniture‘s A/c (Bad debts) 80,000 2,500 1,110 3,475 2,000 Sudharaka Furniture‘s A/c: Cash sales 59,500 Credit sales 10,000 Stock on consignment Profit & loss A/c (Loss transferred) 69,500 16,650 2,935 89,085 89,085 Sudharaka Furniture’s A/c Dr Cr Consignment to Kelaniya A/c 69,500 Bill Receivable A/c Consignment to Kelaniya A/c Consignment to Kelaniya A/c Consignment to Kelaniya A/c Balance c / d 40,000 1,110 3,475 2,000 22,915 69,500 69,500 Bill Receivable A/c Dr Cr Sudharaka Furniture‘s A/c 40,000 40,000 Bank A/c Discount 38,000 2,000 40,000 Goods sent on Consignment Account Dr Cr Trading A/c (Transfer) 80,000 Consignment to Kelaniya A/c 80,000
  • 20. 19 | P a g e A.Ajanthan Joint Venture & Consignment Accounts Profit & Loss A/c Dr Cr Consignment to Kelaniya A/c 2935 Valuation of stock: Number of chairs in stock = 100 – 80 = 20 chairs LKR Original cost of 20 chairs (800 * 20) = 16,000 Add: proportionate non – recurring expenses: Incurred by consignor 2500 Incurred by consignee 750 3250 * (20 / 100) = 650 Stock value 16,650 *The entry for bad debt is: Consignment A /c………………Dr Sudharaka Furniture‘s A/c………………Cr Books of Sudharaka Furniture’s Journal Entries Description Dr (LKR) Cr (LKR) 1. Prasana Furniture‘s A/c Bank / Cash A/c (Paid expenses on the Consignment received) 1,110 1,110 2. Prasana Furniture‘s A/c Bills payable A/c (Acceptance of bill drawn against the consignment) 40,000 40,000 3. Bank A/c Debtors A/c Prasana Furniture‘s A/c (The amount of total sales) 59,500 10,000 69,500 4. Prasana Furniture‘s A/c Debtors A/c 2,000 2,000
  • 21. 20 | P a g e A.Ajanthan Joint Venture & Consignment Accounts (The amount of bad debts) 5. Bank A/c Debtors A/c (The amount collected from debtors) 8,000 8,000 When there is Del credere Commission; The consignee has to bear any amount of bad debts due to the insolvency of a debtor, if there is a provision for payment of del credere commission. In such a situation, the consignor need not make any entry in his book for bad debts. But the bad debt is a loss to the consignee. Hence, it should be recorded in his book through the following entry: Bad debt A/c……………..Dr Debtors A/c……………………..Cr Note: if there is any loss on account of non-payment of a customer on dispute regarding quality of goods should be borne by consignor although there is a provision for dl credere commission. Illustration: 6 Amirtha Paints, Jaffna, consigned 500 tins of paints to Arvind Paints, Cochin at Lkr. 60. They spent Lkr. 400 for packing and Lkr. 600 for freight and insurance, and drew against the consignment a bill for the amount o 80% of the cost of goods sent. On getting the acceptance, Amirtha Paints discounted the bill at a cost of Lkr. 1200. Arvind Paints, Cochin sold 400 tins of paints at Lkr.80 of which 50 tins were on credit. Their sales expenses amounted to Lkr. 300. They were to get a commission of 4% plus 2% del credere commission. A customer who bought 10 tins of paints on credit became insolvent and only Lkr. 400 was realized from him in full settlement. Prepare consignment account and consignee‘s account in the books of consignor, and also show journal entries in the books of consignee. Solution: Ledger of Amirtha Paints Consignment to Cochin A/c Dr Cr
  • 22. 21 | P a g e A.Ajanthan Joint Venture & Consignment Accounts Goods sent on Consignment A/c Bank A/c (expenses) Arvind paints A/c Arvind paints A/c (Commission) Profit & loss A/c 30,000 1000 300 1920 4980 Arvind paints A/c Stock on consignment 32000 6200 89,085 89,085 Valuation of stock: Original cost of stock 60*100 = Lkr. 6000 Proportionate expense of the consignor 1000 / 500 *100 = Lkr. 200 Value of stock = Lkr. 6200 Arvind Paints A/c Dr Cr Consignment to Cochin A/c 32,000 Bill Receivable A/c Consignment to Cochin A/c Consignment to Cochin A/c Balance c / d 24,000 300 1,920 5,780 32000 32000 Journal of Arvin Paints, Cochin (Consignee) Description Dr (LKR) Cr (LKR) 1. Amirtha paints A/c Bank / Cash A/c (Paid expenses on the Consignment received) 300 300 2. Amirtha paints A/c Bills payable A/c (Acceptance of bill drawn against the consignment) 24,000 24,000 3. Bank A/c Debtors A/c Amirtha paints A/c 28,000 4,000 3,2000
  • 23. 22 | P a g e A.Ajanthan Joint Venture & Consignment Accounts (The amount of total sales) 4. Amirtha paints A/c Commission A/c (Effected the commission on sales) 1,920 1,920 5. Bank A/c Bad debts A/c Debtors A/c (50% of the amount of Lkr. 800 realized in full settlement due from a customer) 400 400 800 Invoicing Goods at a Price Higher than Cost When the consignor does not want to reveal the actual cost of goods to the consignee, the goods are invoiced at a price which is higher than the cost price. Such a price is known as invoice price. In other words, invoice price is equal to the cost plus a certain amount of profit. The difference between invoice price and the cost price is termed as loading. In this connection, it should be noted that invoice price is not the same thing as selling price. The invoice price is the price at which the consignor sends the goods to the consignee. Selling price, on the contrary, is the price at which the consignee sells the goods to the customers. It is to be observed that IP (invoice price) is higher than CP (cost price), whereas SP (selling price) is higher than the CP as well as the IP. However, if the consignor directs the consignee to sell the goods at invoice itself, then the SP and IP will be the same. Loading The amount of profit which is added to the cost in order to arrive at the invoice price is known as loading. In other words, loading is the difference between the invoice price and the cost price. Loading = IP – CP For example, the invoice price is Lkr. 5000 and the cost price is Lkr.3750. Calculate the amount of loading. Loading = IP – CP or Number of units * (IP per unit – CP per unit) = 5000 – 3750 = Lkr. 1250
  • 24. 23 | P a g e A.Ajanthan Joint Venture & Consignment Accounts Items which involve loading Usually loading is involved in all such items which are recorded at the invoice price in the consignment account. Some such items are as follows: 1. Opening stock 2. Goods sent on consignment 3. Goods returned by the consignee 4. Closing stock Adjustment on Loading The usual adjustments required on loading are as follows: 1. Opening stock: It is always shown on the debit side of the consignment account. Hence, the difference between the invoice price and the cost price of the stock will be shown on the credit side of the consignment account through the following entry: Stock reserve A/c…………………….Dr Consignment A /c………………………….Cr 2. Goods sent on consignment: Such goods are shown on the debit side of the consignment account. Thus the difference between invoice price and cost price of goods sent on consignment will be shown on the credit side of the consignment account through the following entry: Goods sent on consignment…………Dr Consignment A /c…………………………Cr 3. Goods returned by the consignee: The return of goods is shown on the credit side of the consignment account. Therefore the adjustment for the loading will be made on the debit side of consignment account through the following entry: Consignment A /c……………………Dr Goods sent on consignment…….………..Cr 4. Closing stock: it is shown on the credit side of consignment account. Hence, the adjustment for the loading will be made on the debit side through the following entry: Consignment A /c……………………Dr Stock reserve A/c………………………… Cr In practice, loading done at a fixed percentage of profit on cost bears a fixed relation with the profit on invoice price of the goods.
  • 25. 24 | P a g e A.Ajanthan Joint Venture & Consignment Accounts For example, in case goods of the costs of Lkr.5000 are consigned at a profit of 25% on cost, the invoice price of the product will be 5000 + 25% of 5000. Invoice price of the product = 5000 + 1250 = Lkr. 6250 The same amount of loading is obtained on applying the percentage of profit on invoice price. It is ascertained as follows: Cost of goods is assumed to be 100. Loading (profit) on cost = 25 Invoice price (100 + 25) = 125 Loading (profit) on invoice price = 25 / 125 =1 / 5 = 20% Loading on the invoice price of Lkr. 6250 = 6250 *25 /15 = Lkr. 1250 Or = 6250 *1 /5 = Lkr. 1250 Or = 6250 *20 / 100 = Lkr. 1250 In the light of the above example, it is clear that 25% (1 / 4) of profit on cost of a product is equal to 20% (1 / 5) of the invoice price of that product. Illustration: 7 Ambika Electronics, Jaipur, consigned 1000 radios to Lakshmi Electronics, Agra, for sale on commission of 5% including 1% del credere commission. The cost price of a radio was Lkr.2400. But the invoice was made at Lkr. 3000. The expenses at Jaipur amounted to Lkr. 54000 and that at Agra before reaching the goods at godown was Lkr.46000. Lakshmi Electronics sold 800 radios @ Lkr. 3200, the sales expenses being Lkr. 28000. The consignee sent a draft for the amount due along with the account sales. Give entries and accounts in the books of both the parties. Books of Ambika Electronics Journal Entries Description Dr (LKR) Cr (LKR) 1. Consignment to Agra A/c Goods sent on Consignment A/c (Invoice price of radios sent on consignment) 3,000,000 3,000,000
  • 26. 25 | P a g e A.Ajanthan Joint Venture & Consignment Accounts 2. Consignment to Agra A/c Bank A/c (Expenses incurred on the Consignment) 54,000 54,000 3. Consignment to Agra A/c Lakshmi Electronics A/c (paid carriage and sales expenses by consignee) 74,000 74,000 4. Lakshmi Electronics A/c Consignment to Agra A/c (The amount of sales made by the consignee) 2,560,000 2,560,000 5. Consignment to Agra A/c Lakshmi Electronics A/c (Commission on gross sales payable @ 5 %) 128,000 128,000 6. Stock on consignment A/c Consignment to Agra A/c (The amount of unsold goods in stock) 620,000 620,000 7. Goods sent on Consignment A/c Consignment to Agra A/c (Loading on goods sent on consignment) 600,000 600,000 8. Consignment to Agra A/c Stock reserve A/c (Loading on consignment stock) 120,000 120,000 9. Consignment to Agra A/c Profit and Loss A/c (Transferred profit on consignment to profit and loss A/c) 404,000 404,000 10. Bank A/c Lakshmi Electronics A/c (Amount received in draft along with account sales) 2,358,000 2,358,000 Consignment to Agra A/c Dr Cr
  • 27. 26 | P a g e A.Ajanthan Joint Venture & Consignment Accounts Goods sent on Consignment A/c Bank A/c (Expenses) Lakshmi Electronics A/c (Exp.) Lakshmi Electronics A/c (Comm.) Stock Reserve A/c (Loading in stock) P & L A/c (Transfer) 3,000,000 54,000 74,000 128,000 120,000 404,000 Lakshmi Electronics A/c (Sale proceeds) Consignment Stock A/c Goods sent on Consignment A/c (Loading in goods sent) 2,560,000 620,000 600,000 3,780,000 3,780,000 Lakshmi Electronics A/c Dr Cr Consignment to Agra A/c 2,560,000 Consignment to Agra A/c Consignment to Agra A/c Bank 74,000 128,000 2,358,000 2,560,000 2,560,000 Goods sent on Consignment Account Dr Cr Consignment to Agra A/c Trading A/c 600,000 2,400,000 3,000,000 Consignment to Agra A/c 3,000,000 3,000,000 Consignment Stock Account Dr Cr To Consignment to Agra A/c 620,000 Balance c /d 620,000 620,000 620,000 Valuation of stock: Invoice price of stock (Lkr. 3000*200 units) 600,000 Add: proportionate non-recurring expenses: Incurred by consignor 54,000 Incurred by consignee 46,000
  • 28. 27 | P a g e A.Ajanthan Joint Venture & Consignment Accounts 100,000 = 100,000 / 1,000*200 = 20,000 Value of stock 620000 Stock reserve = loading in one unit * number of units in stock = 600 * 200 = Lkr. 120,000 Stock Reserve A/c Dr Cr Balance c /d 120,000 Consignment to Agra A/c 120,000 120,000 120,000 Books of Lakshmi Electronics Journal Entries Description Dr (LKR) Cr (LKR) 1. Ambika Electronics A/c Bank / Cash A/c (Expenses met by the consignor) 74,000 74,000 3. Bank A/c Ambika Electronics A/c (The amount of total sales) 2,560,000 2,560,000 4. Ambika Electronics A/c Commission A/c (Effected the commission on sales) 128,000 128,000 5. Ambika Electronics A/c Bank A/c (Remitted the amount with account sales) 2,358,000 2,358,000 Ambika Electronics A/c Dr Cr Bank A/c Commission A/c Bank A/c 74,000 128,000 2,358,000 Bank A/c 2,560,000 2,560,000 2,560,000
  • 29. 28 | P a g e A.Ajanthan Joint Venture & Consignment Accounts Illustration: 8 Riyash of Warahapolai sent to his agent, Ashan of Puttalam, 500 articles costing Lkr.15 per article at an invoice price of Lkr.20 per article. The following payments were made by Riyash in this connection: Freight and carriage Lkr. 450 Miscellaneous expenditure Lkr. 50 Ashan sent a bank draft for Lkr.3, 000 as an advance against the Consignment. Ashan sold 300 articles at a flat rate of Lkr.28 per article and sent an Account Sales showing deduction for storage charges Lkr.50, insurance Lkr.100 and his Commission of 3% plus 2% Del Credere on gross sale proceeds, and remitted the amount due on consignment. Ashan also informed Riyash that 50 articles were damaged in transit and thus they were valued at Lkr.550. Record the above transactions in the books of the consignor and consignee using cost price basis. Solution: Books of Riyash (Consignor) Journal Description Dr (LKR) Cr (LKR) 1. Consignment to Puttalam A/c Goods sent on Consignment A/c (500 articles sent to Ashan, Agent, and Cost being Lkr.15 per article). 7,500 7,500 2. Consignment to Puttalam A/c Bank A/c (Expenses incurred on the Consignment) Freight & Carriage Lkr. 450 Miscellaneous Exp. Lkr. 50 500 500 500 3. Bank A/c Ashan A/c (Advance received from the Agent in the form of Bank Draft.) 3,000 3,000 4. Ashan A/c Consignment to Puttalam A/c (Sales affected by Ashan as per Account Sales.) 8,400 8,400 5. Consignment to Puttalam A/c Ashan A/c 570 570
  • 30. 29 | P a g e A.Ajanthan Joint Venture & Consignment Accounts (Expenses incurred by Ashan Lkr.150 and Commission due to him, Lkr.420 (5% of Lkr.8, 400). 6. Bank A/c Ashan A/c (Amount due from the consignee received.) 4,830 4,830 7. P & Loss A/c Consignment to Puttalam A/c (Abnormal Loss on 50 damaged Articles) *250 *250 8. Consignment Stock A/c Consignment to Puttalam A/c (Value of stock unsold at Puttalam) Lkr. 150, goods articles, @ Lkr.15 2,250 Add: Freight and carriage( 450/500*150) 135 Miscellaneous expenditure(50/500*150) 15 50 damaged articles 550 2,950 2,950 2,950 9. Consignment to Puttalam A/c Profit & Loss Account (Profit on consignment transferred to Profit & Loss Account) 3,030 3,030 10. Goods sent on Consignment A/c Trading A/c (Goods sent on consignment A/c closed by transfer to trading Account) 7,500 7,500 Working Notes: Calculation of Abnormal Loss LKR Cost @ Lkr.15*50 articles 750 Proportionate Expenses: Freight and carriage (Lkr.450/500*50) 45 Miscellaneous expenditure (Lkr.50/500*50) 5 50 800
  • 31. 30 | P a g e A.Ajanthan Joint Venture & Consignment Accounts Damaged 50 articles have been have been valued at Lkr.550 Thus, there is a loss of Lkr.250*, (800- 550). Such a loss would be recorded as follows; Profit and Loss A/c ……………………..Dr 250 Consignment A/c……………………………....Cr 250 Ledgers Consignment to Puttalam Account Dr Cr Goods sent on Consignment A/c Bank A/c (expenses) Ashan A/c Expenses 150 Commission 420 P & L A/c (Transfer) 7,500 500 570 3,030 Ashan A/c (Sale proceeds) Profit & Loss A/c (Abnormal Loss) Consignment Stock A/c 8,400 250 2,950 11,600 11,600 Ashan A/c Dr Cr To Consignment to Puttalam A/c 8,400 Bank A/c Consignment to Puttalam A/c Bank A/c 3,000 570 4,830 8,400 8,400 Bank A/c Dr Cr Ashan A/c Ashan A/c 3,000 4,830 Consignment to Puttalam A/c 500 Goods sent on Consignment Account Dr Cr Trading A/c (Transfer) 7,500 Consignment to Puttalam A/c 7,500
  • 32. 31 | P a g e A.Ajanthan Joint Venture & Consignment Accounts Profit & Loss A/c Dr Cr Consignment to Madras A/c 250 Consignment to Puttalam A/c 3,030 Books of Ashan (Consignee) Journal Description Dr (LKR) Cr (LKR) 1. Riyash A/c Bank A/c (Advance sent to the Consignor against consignment) 3,000 3,000 2. Riyash A/c Bank A/c (Expenses incurred on the Consignment on behalf of Riyash) Storage 50 Insurance 100 150 150 150 3. Bank A/c Riyash A/c (Sale of 300 articles @ Rs.28 each out of the Consignment.) 8,400 8,400 4. Riyash A/c Commission A/c (5% Commission on Sales made on behalf of Riyash; 3% Commission + 2% Del Credere Com.) 420 420 5. Riyash A/c Bank A/c (Amount due to Riyash remitted). 4,830 4,830 Ledgers Riyash A/c Dr Cr Bank A/c (Advance) Bank A/c (Expenses) Commission A/c Bank A/c (amount remitted) 3,000 150 420 4,830 Bank A/c (Sale proceeds) 8,400
  • 33. 32 | P a g e A.Ajanthan Joint Venture & Consignment Accounts 8,400 8,400 Bank A/c Dr Cr Riyash A/c 8,400 Riyash A/c Riyash A/c Riyash A/c 3,000 150 4,830 Commission A/c Dr Cr Riyash A/c 420 Illustration: 9 Riyash of Warahapolai sent to his agent, Ashan of Puttalam, 500 articles costing Lkr.15/- per article at an invoice price of Lkr.20 per article. The following payments were made by Riyash in this connection: Freight and carriage Lkr. 450 Miscellaneous expenditure Lkr. 50 Ashan sent a bank draft for Lkr.3, 000 as an advance against the Consignment. Ashan sold 300 articles at a flat rate of Lkr.28 per article and sent an Account Sales showing deduction for storage charges Lkr.50, insurance Lkr.100 and his Commission of 3% plus 2% Del Credere on gross sale proceeds, and remitted the amount due on consignment. Ashan also informed Riyash that 50 articles were damaged in transit and thus they were valued at Lkr.550. Record the above transactions in the books of the consignor and consignee using invoice price basis. Solution: Books of Riyash (Consignor) Journal Description Dr (LKR) Cr (LKR) 1. Consignment to Puttalam A/c Goods sent on consignment A/c (500, articles consigned at an invoice price of Lkr.20 each (cost 10,000 10,000
  • 34. 33 | P a g e A.Ajanthan Joint Venture & Consignment Accounts Lkr.15) 2. Consignment to Puttalam A/c Bank A/c (Expenses incurred on the Consignment) Freight & Carriage Lkr. 450 Miscellaneous Exp. Lkr. 50 500 500 500 3. Bank A/c Ashan A/c (Advance received from the Agent in the form of Bank Draft.) 3,000 3,000 4. Ashan A/c Consignment to Puttalam A/c (Sales affected by Ashan as per Account Sales.) 8,400 8,400 5. Consignment to Puttalam A/c Ashan A/c (Expenses incurred by Ashan Lkr.150 and Commission due to him, Lkr.420 (5% of Lkr.8, 400). 570 570 6. Bank A/c Ashan A/c (Amount due from the consignee received.) 4,830 4,830 7. P & Loss A/c Consignment to Puttalam A/c (Abnormal Loss on 50 damaged Articles) 250 250 8. Consignment Stock A/c Consignment to Puttalam A/c (Value of stock unsold at Puttalam) Lkr. 150, goods articles, @ Lkr.20 3,000 Add: Freight and carriage( 450/500*150) 135 Miscellaneous expenditure(50/500*150) 15 50 damaged articles 550 3,700 3,700 3,700 9. Goods sent on Consignment A/c Consignment to Puttalam A/c Excess amount included in invoice price of articles sent to Puttalam 2,500 2,500
  • 35. 34 | P a g e A.Ajanthan Joint Venture & Consignment Accounts (Rs.5 each) credited on consignment A/c) 10. Consignment to Puttalam A/c Stock Reserve A/c [Reserve credited equal to excess amount above cost (Lkr.5 per articles*(200-50)] 750 750 11. Consignment to Puttalam A/c Profit and Loss A/c (Transfer of Profit on Consignment) 3,030 3,030 12. Goods sent on Consignment A/c Trading A/c (Goods sent on Consignment A/c closed by transfer to Trading A/c) 7,500 7,500 Ledgers Consignment to Puttalam A/c Dr Cr Goods sent on Consignment A/c Bank A/c (expenses) Ashan A/c Expenses 150 Commission 420 570 Stock Reserve A/c P & L A/c (Transfer) 10,000 500 570 750 3,030 Ashan A/c (Sale proceeds) Profit & Loss A/c (Abnormal Loss) Consignment Stock A/c Goods sent on Consignment A/c 8,400 250 3,700 2,500 14,850 14,850 Goods sent on Consignment Account Dr Cr Consignment to Puttalam A/c Trading A/c 2,500 7,500 10,000 Consignment to Puttalam A/c 10,000 10,000 Stock on Consignment Account Dr Cr
  • 36. 35 | P a g e A.Ajanthan Joint Venture & Consignment Accounts To Consignment to Puttalam A/c 3,600 Consignment to Puttalam A/c Balance sheet (Transfer)* 750 2,850 3,600 3,600 *In the Balance Sheet the stock on consignment will be shown at Rs.2, 850 [(Lkr.3, 000 – Reserve (Lkr.750)] Expenses on consignment to be borne by consignee In case expenses connected o a particular consignment at the place of the consignee is to be borne by him, he will be given only commission. In such a case, the expense met by the consignee is not recovered or reimbursed. But the consignee should treat it as his business expense, and is to be transferred to his profit and loss account. However, there will be no entry in the books of consignor for the expenses met by consignee. Illustration: 10 Pradap Traders consigned goods of the cost of Lkr. 30000 to their agent, Sancha Agencies Uduppiddy, at a profit of 20% on cost. Consignee was allowed a commission of 8% on gross sales for which he would bear all the expenses at his place. Pradap traders spent Lkr. 1500 for freight and got an acceptance for Lkr. 15000 from the consignee. Sancha Agencies paid Lkr. 600 for advertisement and Lkr. 400 for sales expenses. They sold ¾ th of the goods at a profit of 33 1/3% on original cost of it. Prepare consignment account the books of Pradap Traders and show journal entries in the books of Sancha Agencies. Solution: In the Books of Pradap Traders Consignment to Uduppiddy Account Dr Cr Goods sent on Consignment A/c Bank A/c (expenses) Sancha Agencies A/c (Commission) Stock Reserve A/c P & L A/c (Transfer) 36,000 1,500 2,400 1,500 3,975 Sancha Agencies A/c (Sale proceeds) Consignment Stock A/c Goods sent on Consignment A/c (Loading) 30,000 9,375 6,000 14,850 14,850
  • 37. 36 | P a g e A.Ajanthan Joint Venture & Consignment Accounts Working Notes: 1. Loading on goods sent: Invoice price – cost price (36,000 – 30,000) = 6,000 Or 36,000 * 20 / 100 = 6,000 2. Value of stock: Stock at invoice price, 36,000 * 1 / 4 = 9,000 Add: proportionate expenses of consignor 1,500 * 1 / 4 = 375 Value of stock = Lkr.9375 3. Stock reserve: Stock reserve = invoice price of stock – cost price of stock = [36,000*1/4] – [30,000*1/4] = 9,000 – 7,500 = Lkr. 1500 Or = 9,000*20/120 = Lkr. 1500 Books of Sancha Agencies (Consignee) Journal Entries Description Dr (LKR) Cr (LKR) 1. Pradap Traders A/c Bills payable A/c (Accepted the bill drawn against the consignment) 15,000 15,000 2. Advertisement A/c Sales expenses A/c Bank A/c (Expenses incurred by consignee cannot be recovered from consignor) 600 400 1,000 3. Bank A/c Pradap Traders A/c (The amount of sales) 30,000 30,000 4. Pradap Traders A/c 2,400
  • 38. 37 | P a g e A.Ajanthan Joint Venture & Consignment Accounts Commission A/c (8% Commission on Sales) 2,400 Losses on Consignment In case the goods sent on consignment are lost or damaged in transit or otherwise, the loss is that of the consignor and not of the consignee. Accordingly the consignor will have to make the entries for such loss. There are two types of losses which may arise in case of a consignment transaction, viz., Normal Loss and Abnormal Loss. Normal Loss Normal loss is natural, unavoidable and inherent in the nature of goods or commodities sent on consignment (due to evaporation, leakage & breaking the bulk into pieces). This type of loss is a part of the cost of the consignment, so the consignor does not make separate entry for such a loss. However, the normal loss has to be taken into consideration while valuating the unsold consignment stock in the hand of the consignee. Since normal loss is a charge against gross profit. No additional adjustment is required for this purpose. Moreover, the same is a part of cost of goods, when valuation of unsold stock is made in case of consignment account the quantity of such loss (not the amount) should be deducted from the total quantity of the goods received by the consignee in good condition The accounting treatment of normal loss is to charge the total cost of the goods to the remaining goods after the normal loss. In other words, the value of the unsold stock is calculated in proportion to the total cost of the goods consigned. Total cost of the goods sent Value of unsold stock = × Unsold quantity Units of Goods sent – Normal losses (units) …………….. (1) Or Unsold quantity Value of unsold stock = × Total value of goods sent Good quantity received by consignee ……………… (2)
  • 39. 38 | P a g e A.Ajanthan Joint Venture & Consignment Accounts Illustration: 11 Suppose 10,000 tons of coal is dispatched. The cost of 1 tons of coal is Lkr.80 and the freight incurred is Lkr.36, 000. To the Consignor the total cost comes to Lkr.8, 36,000. In the nature of coal some shortage is unavoidable. Suppose the Consignee receives only 9,500 tones. It is legitimate to say that the cost is Lkr.8, 36,000 for 9,500 tons. In that case the Consignor can properly say that the cost of 1 tons of coal is Lkr.8, 36,000/9500 or Lkr 88. If 2,000 tons of coal is left unsold with the Consignee, the value of stock will be 2,000×88 is equal to Lkr.1, 76,000. Illustration: 12 From the following particulars ascertain the value of unsold stock on consignment. LKR Goods sent (1,000 kgs) 20,000 Consignor‘s expenses 4,000 Consignee‘s non-recurring expenses 3,000 Sold (800 kgs) 40,000 Loss due to natural wastage (100 kgs) Solution: Value of unsold stock ` LKR Total cost of goods sent 20,000 Add: Consignor‘s expenses 4,000 Non-recurring expenses 3,000 Cost of 900 kgs (1,000 kgs – 100 kgs) 27,000 ∴Value of unsold stock 100 kgs (1,000 – 800 – 100) will be; 27,000 × 100 kgs (1000 kgs-100 kgs) = Lkr .3, 000 …………….. (1) 100 kgs × 27,000
  • 40. 39 | P a g e A.Ajanthan Joint Venture & Consignment Accounts 900 kgs = Lkr .3, 000 …………….. (2) Illustration: 13 Mr. Achchu Consigned to Mr. Kajan 10,000 kgs of flour, costing Lkr.33, 000. He spent Lkr.880 as forwarding charges. 12% of the Consignment was lost in weighing and handling. Mr. Kajan sold 8,200 kgs of flour at Lkr.6 per kg, his selling expenses being Lkr.3, 300 and Commission 5% on sales. Prepare the Consignment Account. Ledger of Mr. Achchu (Consignment Account) Dr Cr Goods sent on Consignment A/c Bank (forwarding Charges) Mr. Kajan A/c Selling Expenses 3,300 Commission 2,460 (@5% on Rs.49,200) P & L A/c (Transfer) 33,000 880 5,760 11,870 Mr. Kajan A/c [ (Sale proceeds, 8,200×6] Consignment Stock A/c 49,200 2,310* 51,510 51,510 Working Notes: 1. Calculation of Closing Stock: Kgs Total quantity of flour consigned 10,000 Less: Normal Loss 12% 1,200 Sales 8,200 (9,400) Closing Stock 600 2. Valuation of Closing Stock: Total Cost of the goods sent + non recurring expenses × Closing Stock (units) Units of Goods sent – Normal losses (units)
  • 41. 40 | P a g e A.Ajanthan Joint Venture & Consignment Accounts Lkr.33, 000 + Lkr.880 × 600 10,000 - 1,200 = Lkr 33880/8800*600 = Lkr 2,310* Abnormal Loss It arises due to abnormal factors or circumstances such as fire, theft pilferage, sabotage, negligence, inefficiency, etc. Before ascertaining the result of the consignment, value of abnormal loss should be adjusted. The method of calculation is similar to the method of calculating unsold stock. Sometimes insurance company admits the claim in part or in full. The same should also be adjusted against such abnormal loss. This loss is calculated by adding proportionate direct expenses incurred by the consignor and the consignee as the case may be to the original cost of the goods. The accounting entry is: Abnormal Loss A/c…………………….Dr Consignment A/c……………………………..Cr In case the stock is insured, the amount of claim admitted by the insurance company should be reduced from the abnormal loss and only the net loss amount should be debited to abnormal loss or P&L A/c, the entry will be: Insurance Company A/c (with the amount of claim admitted) Profit and Loss (Abnormal Loss A/c) (with the amount of loss) Consignment A/c (with the amount of Total abnormal loss) Dr Dr Cr The procedure for calculating the abnormal loss and the valuation of the remaining stock is summarized as under: (i) Calculation of Abnormal loss: Add: Cost of goods lost xxxx Proportionate expenses of the goods lost xxxx xxxx Less: Any amount of claim (xxxx) (If any received from the insurance company) xxxx
  • 42. 41 | P a g e A.Ajanthan Joint Venture & Consignment Accounts (ii) Valuation of Closing Stock (1) Cost of the goods Closing Stock × Cost of total goods consigned Total goods consigned Add: Proportionate Non-recurring (direct) expenses incurred before the loss Closing Stock × Expenses incurred before the loss Total goods consigned Add: Proportionate expenses (Direct only) incurred after the loss Quantity unsold × Expenses incurred after the loss (Total quantity sent - Goods lost) Illustration: 14 Aju smart of Jaffna dispatched 1,000 shirts at Lkr.700 each to Mohan Bros of Colombo, the consignors paid freight Lkr.7, 500, cartage Lkr.500 and insurance Lkr.2, 500. Mohan Bros. received only 900 shirts and incurred the following expenses. LKR Freight and other Expenses 1, 00,000 Cartage 5,000 Sales expenses 6,000 The consignee sold 600 shirts only. You are required to calculate the value of closing stock. Solution: Calculation of the value of unsold stock Shirts received 900- shirts sold 600 = unsold stock 300 (i) Cost of unsold stock 300 × 700 = 2,10,000
  • 43. 42 | P a g e A.Ajanthan Joint Venture & Consignment Accounts (ii) Add: Proportionate Expenses Paid by consignor 3/10* 10,500(7500 + 500 + 2500) = 3,150 (iii) Add: Proportionate Expense Freight 1,00,000 Paid by consignee Cartage 5,000 105,000 1, 05, 000 × 300 = 35,000 900 248,150 Illustration: 15 S of Bombay consigned 10,000 Liter of oil to D of Calcutta. The cost of oil was Lkr.2 per Liter. S paid Lkr.5, 000 as freight and insurance. During transit 250 Liter were accidentally destroyed for which the insurers paid directly to the consignors Lkr.450 if full settlement of the claim. D reported that 7,500 Liter was sold @ Lkr.3 per Liter. The expenses being on godown rent Lkr. 200, on advertisement Rs.1, 000 and on salesman salary Lkr.2, 000. D is entitled to a commission of 3% plus 1.5% Del credere. D reported a loss of 100 Liter due to leakage. D settled the accounts by bank draft. Prepare the accounts in the books of S. Consignment to Calcutta A/c Dr Cr Goods sent on Consignment A/c Bank- Freight & Insurance D A/c Expenses (200+1000+2000) Commission: Ordinary(22500* 3%) 675 Del Credere(22500*1.5%) 338 20,000 5,000 3,200 1,013 D A/c [ (Sale proceeds, 7500×3] Bank (Insurance company) P & L A/c (Abnormal loss) Consignment Stock A/c P & L A/c (Transfer) 22,500 450 175 5,430 658 29,213 29,213 Goods sent on Consignment A/c Dr Cr Trading A/c 20,000 Consignment to Calcutta A/c 20,000
  • 44. 43 | P a g e A.Ajanthan Joint Venture & Consignment Accounts Consignment Stock A/c Dr Cr Consignment to Calcutta A/c 5,431 Balance c/d 5,431 D A/c Dr Cr Consignment to Calcutta A/c 22,500 Consignment to Calcutta (Exp.) Consignment to Calcutta A/c (com.) Bank A/c 3,200 1,013 18,287 8,400 8,400 Working Notes: (A) Cost of Goods destroyed LKR Cost of 10,000 Liter @Lkr. 2 20,000 Freight 5,000 Total cost of 10,000 Liter 25,000 If 250 Liter were accidentally destroyed, 25,000/10,000*250 = Lkr.625 B) Value of stock still unsold Liter: Quantity received by D 9,750 [10,000-250(Accidental loss)] Less: Normal leakage (100) 9650 Cost of 9,650 Liter = Lkr. 25, 000 – Lkr. 625 = Lkr. 24, 375, So Cost of 2,150 Liter = 24375/9650*2150 = Lkr. 5430 Illustration: 16 Mithuna Traders of Jaffna purchased 10,000 Bags @100 per Bag. Out of these 6,000 Bags were sent on consignment to Nantha Traders of Kilinochchi at the selling price of 120 per Bag. The consignors paid 3,000 for packing and freight. Nantha Traders sold 5,000 Bags @125 per Bag and incurred 1,000 for selling expenses and remitted 5,00,000 to Jaffna on
  • 45. 44 | P a g e A.Ajanthan Joint Venture & Consignment Accounts account. They are entitled to a commission of 5% on total sales plus a further of 25% commission on any surplus price realized over 120 per Bag. 3,000 Bags were sold at Jaffna @ 110 per Bag. Owing to fall in market price, the value of stock of Bags in hand is to be reduced by 5%. You are required to prepare; (i) Consignment Account, and (ii) Nantha Traders Account. Solution: Consignment to Kilinochchi A/c Dr Cr Goods sent on Consignment A/c Bank A/c - (Packing and Freight) Nantha Traders A/c: Selling Expenses Commission (w1) P & L A/c 6,00,000 3,000 1,000 37,500 79,000 Nantha Traders A/c [ (Sale proceeds, 5000×125] Consignment Stock A/c (w2) 6,25,000 95,500 7,20,500 7,20,500 Note: 3,000 Bags which were sold at Jaffna @110 per Bag are not to be taken into consideration since it is not a consignment transaction and hence the same is extended from Consignment Account. Although the consignor purchased 10,000 Bags, only 6,000 Bags are related to consignment transaction, balance is not to be taken into Consignment Account at all. Nantha Traders Account Dr Cr Consignment to Kilinochchi A/c 6,25,000 Bank A/c (Advance) Consignment to Kilinochchi A/c (Selling expenses) Consignment to Kilinochchi A/c (Commission) Bank A/c 5,00,000 1,000 37,500 86,500 6,25,000 6,25,000 Workings Notes:
  • 46. 45 | P a g e A.Ajanthan Joint Venture & Consignment Accounts 1. Calculation of commission payable to Nantha Traders: LKR Total Sales @ 125 per Bag Less: Amount 120 per Bag Surplus Price Realized 5% Commission on total Sales (Lkr 625,000*5%) 25% Commission on surplus price realized (Lkr 25,000*25%) 6,25,000 (6,00,000) 25,000 31,250 6,250 Total commission payable 37,500 2. Valuation of unsold stock: Since market price has fallen by 5%, valuation of unsold stock on consignment will be calculated as under: LKR Total Cost (1,000 x 100) packing and freight(3000/6000*1000) Less: 5% in reduction 1,00,000 500 (5,000) Stock on Consignment 95,500 Illustration: 17 A company sent 300 bales of cotton to its consignee at profit 20% on sale. The cost of each bale to company is Lkr.600 per bale. The following are the expenses incurred in connection with this consignment: (a) Lkr.900 paid by the consignor for dispatching goods. (b) Lkr.2, 000 paid by the consignee by way of freight, duty and landing charges. (c) Lkr.1, 000 paid by the consignee by way of godown rent, salaries of salesman. Required: The valuation of stock at the end (at invoice price) if the consignee sells away 2/3rd of the consignment. Solution: Total bales sent 300 Less: bales sold 2/3rd of 300 (200) Bales unsold 100 Cost price of 100 bales at Rs.600 per bale 60,000
  • 47. 46 | P a g e A.Ajanthan Joint Venture & Consignment Accounts Add: Profit at 20% on sale or 25% on cost 15,000 75,000 Add (1/3rd direct expenses): Expenses paid by Consignor 900 Expenses paid by Consignor 2,000 1/3rd thereof 2900 2900*1/3 967 75,967 Note: In the consignment account, stock reserve account will appear at Rs.15, 000 on the debit side. Illustration: 18 Alagu sold goods on behalf of Aju Sales Corporation on consignment basis. On 1st January, 2002 he had with him a stock of Lkr.20, 000 on consignment. During the year, he received goods worth Lkr.2, 00,000. Alagu had instructions to sell goods at cost plus 25% and was entitled to a commission of 4% on sales in addition to 1% del credere commission. During the year ended 31st December, 2002 cash sales were Lkr.1, 20,000; credit sales Lkr.1, 05,000; Alagu‘s expenses relating to consignment Lkr.3, 000 being salaries and insurance & bad debts amounted to Lkr.3, 000. Required: Prepare necessary accounts in the books of Aju Sales Corporation. (Consignor) Solution: Consignment Account Dr Cr Consignment Stock b/d Goods sent on Consignment A/c Alagu A/c (Commission) Alagu A/c (Commission) Alagu A/c (salaries and insurance) P & L A/c 20,000 2,00,000 9,000 2,250 3,000 30,750 Alagu A/c Cash Sales 1,20,000 Credit Sales 1,05,000 Consignment Stock A/c 2,25,000 40,000 2,65,000 2,65,000 Alagu A/c Dr Cr
  • 48. 47 | P a g e A.Ajanthan Joint Venture & Consignment Accounts Consignment A/c (sales) 2,25,000 Consignment A/c (Commission) Consignment A/c (Commission) Consignment A/c (salaries and insurance) Balance c/d 9,000 2,250 3,000 2,10,750 2,25,000 2,25,000 Working Notes: (1) Calculation of Consignment Stock Sale Price = 100 + 25 = 125 Cost of Sales = Sales ×100/125 = 2, 25,000 × 100/125 = Lkr.1, 80,000 Cost of the goods available for sale = Lkr. 20,000(op. stock) + Lkr.2, 00,000 = Lkr.2, 20,000 Hence stock at the end = Lkr.2, 20,000 - Lkr.1, 80,000 = Lkr.40, 000 (2) Since Alagu is paid del-credere commission, bad debts of Rs.3, 000 would be borne by him. Illustration: 19 On 10 January 2010 Kumar Sangakara of Galle consigned 1000 calculators to Mahela, Kadawatta. The goods are invoiced at Lkr 30 per unit, the cost price being Lkr 20 per unit. Expenses incurred are: insurance Lkr 150; freight Lkr 1000; cartage and packing Lkr 300. The agent is to receive ordinary commission of 5% and del-credere commission of 4%.Mahela receive the goods on 31 January and pays cash for; freight and cartage Lkr 350;advertising Lkr 250. Repacking of calculators cost is Lkr 200. Mahela sent Kumar Sangakara a cheque for Lkr 5000 as an advance on 31 January 2010. The following sales are made by Mahela to 30 June: Date Cash Credit 2010 Feb 6 50 @ Lkr 40 Lkr 2,000 30 @ Lkr 50 Lkr 1,500 Mar 18 90 @ Lkr 50 Lkr 4,500 150 @ Lkr 40 Lkr 6,000
  • 49. 48 | P a g e A.Ajanthan Joint Venture & Consignment Accounts Apr 20 330 @ Lkr 40 Lkr 1,200 100 @ Lkr 40 Lkr 4,000 Jun 9 97 @ Lkr 50 Lkr 4,850 Mahela took three calculators for their own stock to be accounted for at the current selling price of Lkr 50. Kumar Sangakara balanced the accounts at the end of June and received an account sale from the consignee to this date: Prepare: a) The account sales received by the consignor on 30 June 2010 b) Ledger accounts in the books of the consignor c) Ledger accounts in the books of the consignee Solution: Date Account Sales 30 June 2010 Units for Cash Units on Credit Total Value Feb 6 Mar18 Apr20 Jun 9 50 @ Lkr 40 Lkr 2,000 30 @ Lkr 50 Lkr 1,500 90 @ Lkr 50 Lkr 4,500 150 @ Lkr 40 Lkr 6,000 330 @ Lkr 40 Lkr 13,200 100 @ Lkr 40 Lkr 4,000 97 @ Lkr 50 Lkr 4850 Taken from own stock 3 @Lkr 50 Less: Expenses & charges LKR Freight and cartage 350 Advertising 250 Repacking machine 200 Commission 2,270 Net proceeds Less: Advance 31 January Cheque enclosed Lkr 28130 Signed Mahela - Manager 3,500 10500 17,200 4,850 *150 36,200 (3,070) 33,130 (5,000) 28,130 *Where goods are to be used by the consignee in some other business activity, the debit is to the purchase account. If the goods are taken for private purposes the drawings account is debited.
  • 50. 49 | P a g e A.Ajanthan Joint Venture & Consignment Accounts **Commission calculated thus: 36200 × 5% = 1810 11500 × 4% = 460 2270 Ledger of Kumar Sangakara Consignment A/c - Mahela, Kadawatta Dr Cr Goods sent on Consignment A/c (1000*20) Bank A/c – expenses Insurance 150 Freight 1,000 Cartage & packing 300 Mahela A/c Freight & Cartage 350 Advertising 250 Repacking goods 200 Commission 2,270 P & L A/c 20,000 1,450 3,070 14,980 Mahela A/c Cash Sales 24,700 Credit Sales 11,500 Consignment Stock A/c 150 units @ lkr 20 3,000 (+) 2000/1000*150 300 36,200 3,300 39,500 39,500 Mahela A/c (Consignee) Dr Cr Consignment A/c (sales) 36,200 Bank A/c- Advance Consignment- Mahela A/c (expenses & commission) Bank A/c - settlement 5,000 3,070 28,130 36,200 36,200 Bank A/C Dr Cr
  • 51. 50 | P a g e A.Ajanthan Joint Venture & Consignment Accounts Mahela A/c - advance Mahela A/c - settlement 5,000 28,130 Consignment A/c Insurance 150 Freight 1,000 Cartage & packing 300 1450 Consignment Stock A/c Dr Cr Consignment A/c 3300 Goods sent on Consignment A/c Dr Cr Trading A/c 20,000 Consignment A/c 20,000 Ledger of Mahela Kumar Sangakara A/c (Consignor) Dr Cr Bank A/c- advance expenses commission received Bank - settlement 5,000 800 2,270 28,130 Bank A/c 24550 Account Receivable 11500 Purchases 36,050 150 36,200 36,200 Commission Received Dr Cr Kumar Sangakara A/c 2,270 Bank A/c Dr Cr
  • 52. 51 | P a g e A.Ajanthan Joint Venture & Consignment Accounts Kumar Sangakara A/c Mahela A/c - settlement 24,550 28,130 Kumar Sangakara A/c- advance Kumar Sangakara A/c- expenses Freight & Cartage 350 Advertising 250 Repacking goods 200 Kumar Sangakara A/c- settlement 5,000 800 28,130 Accounts Receivable Dr Cr Kumar Sangakara A/c 11,500 Illustration: 20 ARA & Co consigned 1,000 tin of Ghee costing Lkr.60 per tin to their agents, Anusha Stores, at Calcutta. The agents sold 400 tin at Lkr.80 per tin for cash, 400 tins at Lkr.82 per tin on credit and they took over the balance to their own stock at Lkr.82 per tin. ARA & Co paid freight and carriage Lkr.500 and miscellaneous expenses Lkr.200. They drew on Anusha Stores at 3 Months for Lkr.45, 000, which was duly accepted by the later. The expenses incurred by the Anusha Stores were: LKR Carriage 50 Octroi 40 Storage 110 Miscellaneous 100 They were entitled to 5% commission and 2% del credere commission on total gross sale proceeds. They sent their account sales to their principal showing as a deduction there from their commission and the various expenses incurred by them a month later. All the debtors except one who owed Lkr.200 paid cash and the Anusha Stores remitted the amounts due on consignment. You are required to show; a) The journal entries in the books of the consignor and b) Consignment account in the consignor‘s ledger
  • 53. 52 | P a g e A.Ajanthan Joint Venture & Consignment Accounts Solution: Journal Entries (In the books of Consignor) Description Dr (LKR) Cr (LKR) 1. Consignment A/c Goods sent on consignment A/c (being the goods sent on consignment) 60,000 60,000 2. Consignment A/c Bank A/c (Being the expenses incurred by consignor on account of consignment) 700 700 3. Consignment Account Anusha Stores A/c (Being the expenses incurred by consignee on account of consignment) 300 300 4. Anusha Stores A/c Consignment A/c (Being the sale effected by the consignee.) 81,200 81,200 5. Consignment A/c Anusha Stores A/c (Being the commission on sales). 5,684 5,684 6. Consignment A/c Profit & Loss A/c (Being the profit on consignment transferred to profit and loss account) 14,516 14,516 7. Goods sent on consignment A/c Purchase A/c (Being the value of goods sent on consignment) 60,000 60,000 8. Bills Receivable A/c Anusha Stores A/c (Being the bill drawn on consignment) 45,000 45,000 Ledger Consignment of Calcutta Account Dr Cr
  • 54. 53 | P a g e A.Ajanthan Joint Venture & Consignment Accounts Goods sent on Consignment A/c Bank-Expenses Anusha Stores A/c- Expenses Anusha Stores A/c- Commission P & L A/c 60,000 700 300 5,684 14,516 Anusha Stores A/c Cash Sales (400*80) 32,000 Credit Sales (400*82) 32,800 Balance of stock taken (200*82) 64,800 16,400 81,200 81,200 Illustration: 21 On January 1, 2002, A of Delhi sent on consignment to B of Bombay 200 packets of coffee, costing Lkr.80 and invoiced pro forma at Lkr.100 each. The freight and other charges paid by A amounted to Lkr.640. A sent the documents through Bank and drew upon B a bill for Lkr.10, 000 and discounted the same with the Bank for Lkr.9, 800. The bill was met on maturity. On March 15, B sent Account sales (together with the amount due) showing that 150 packets had realized Lkr.100 each and 25 packets Lkr.110 each and 25 packets were shown as unsold stock. B incurred Lkr.400 as expenses for the entire consignment. B is entitled to a commission of 6%. On March 31 B informed A that 15 packets were damaged due to bad packing and it was estimated that the selling price of the damaged packets would be about Lkr.20 per packet. Both A and B close their books on March 31. Prepare ledger accounts in the books of A and B. Solution: Books of A, Delhi Consignment of Bombay Account Dr Cr Goods sent on Consignment A/c Bank-Expenses B A/c- Expenses B A/c – Commission Stock Reserve Account P & L A/c 20,000 640 400 1,065 200 1,725 B A/c Goods sent on consignment (loading) Abnormal Loss (1) Stock on Consignment (2) Stock of damaged goods 17,750 4,000 648 1,032 600 24,030 24,030
  • 55. 54 | P a g e A.Ajanthan Joint Venture & Consignment Accounts B’s Account Dr Cr Consignment A/c (sales) 17,750 Bills Receivable Consignment A/c - expenses Consignment A/c commission Bank A/c - settlement 10,000 400 1,065 6,785 36,200 36,200 Goods sent of Consignment Account Dr Cr Consignment account Loading Purchase/ Trading A/c 4,000 16,000 Consignment A/c 20,000 36,200 36,200 Books of B A A/c Dr Cr Bills Payable Bank-Expenses Commission A/c Bank 10,000 400 1,065 6,785 Bank Balance c/d 17,750 500 18,250 18,250 Note: (i) Stock at the end (At Invoice Price) Lkr. 10 Packets @ Lkr.100 (Invoice Price) 1,000 Add: Proportionate expenses incurred by A i.e. 1/20th of Lkr.640 32 1,032 (ii) Abnormal Loss Cost of 15 packets damaged (15*80) 1,200 Add: Proportionate expenses 640/200*15 48 1248 Less: Value of 15 packets @ Lkr.20 per Packet (600)
  • 56. 55 | P a g e A.Ajanthan Joint Venture & Consignment Accounts 648 (iii) Since 10 Packets are still in the stock-in-hand, advance to that extent has not been adjusted. Hence Lkr.500 is carried forward i.e. 10,000 ×10/200= Lkr. 500 Where Normal and Abnormal Losses occur simultaneously If both normal and abnormal losses occur simultaneously in connection with the same consignment, the computation of the value of closing stock involves the following procedures: 1. Take the total cost of goods consigned and add all the non-recurring expenses (incurred by the consignor and consignee). 2. Deduct the quantity and cost of abnormal loss from the total number of goods consigned and the cost as obtained in (1) above, respectively. 3. Deduct the quantity of normal loss from the quantity worked out in (2) above without making any adjustment in cost. 4. Calculate cost per unit of goods units by dividing the cost (remaining after deducting the cost of abnormal loss) by the number of goods units. 5. Multiply the number of unsold units with the cost per unit obtained in (4) above to arrive at the value of unsold stock. Illustration: 22 Vegetables Oils Ltd., Polannaruwa, consigned 10,000 Liters of Ghee costing Lkr.20 per Liter to Ranga and Co. of Galle on 1st January 2012. Oils Ltd paid Lkr.50, 000 as freight and insurance. 250 Liters of Ghee were destroyed on 10-1-2012 in transit. The insurance claim was settled at Lkr.4, 500 and was paid directly to the consignors. Ranga and Co. took delivery of the consignment on 20th January 2012 and accepted a bill drawn upon them by Oils Ltd for Lkr 1, 00,000 for 3 months. On 31st March 2012 Ranga and co. reported as Follows. (i) 7,500 Liters were sold at Lkr.30 per Liter. (ii) Other expenses were: Godown rent Lkr.2, 000; Wages Lkr.20, 000 Printing and Stationary including advertising Lkr.10, 000. (iii) 250 Liters were lost due to leakage.
  • 57. 56 | P a g e A.Ajanthan Joint Venture & Consignment Accounts Ranga and Co are entitled to a commission of 4.5% on all the sales affected by them. They paid the amount due in respect of consignment on 31st March itself. Show the consignment account, the account of Ranga and Co. and loss-in-transit account in the books of consignor for the year ended 31st March 2002. Solution: Consignment to Galle Account Dr Cr Goods sent on Consignment A/c Bank A/c - freight and insurance Ranga and Co A/c [(Expenses + Commission), (2,000+20,000+10,000+10,125) 2,00,000 50,000 45,125 Ranga and Co A/c (7500*30) Loss-in-transit (w1) Stock on Consignment A/c (w2) Profit & Loss A/c 2,25,000 6,250 51,316 9,559 2,92,125 2,92,125 Loss-in-Transit A/c Dr Cr Consignment A/c 6,250 Insurance Co. A/c Profit & Loss A/c 4,500 1,750 6,250 6,250 Ranga and Co A/c Dr Cr Consignment A/c (sales) Balance c/d 2,25,000 20,000 Bill Receivable Consignment A/c (Expenses and Commission) Bank A/c 100,000 42,125 1,02,875 2,45,000 2,45,000 Working Notes: (1) Cost of ghee destroyed in transit LKR. Cost of 10,000 Kg of ghee @ Lkr. 20 2, 00,000 Freight and Insurance 50,000 Total cost of 10,000 Kg 2, 50,000
  • 58. 57 | P a g e A.Ajanthan Joint Venture & Consignment Accounts Cost of 250 Kg 2, 50,000/10,000*250 (6,250) Cost of 9,750 Kg of ghee 2, 43,750 (2) Value of stock at the end Kg Quantity of ghee received by the consignee 9,750 Less: Quantity lost through leakage (Normal Loss) (250) Quantity Available for sale 9,500 Total Cost of 9,500 Kg 2, 43,750 Cost of 2,000 Kg 243,750/9,500*2,000 51,316 (3) Since 2000 Kg (9500 – 7500) of ghee has not been sold. Proportionate amount of advance is (100,000×1/5) Lkr.20, 000 will not be adjusted. Illustration: 23 5,000 shirts were consigned by Raizada & Co. of Delhi to Zing of Tokyo at cost of 375 each. Raizada & Co. paid freight 50,000 and Insurance 7,500. During the transit 500 shirts were totally damaged by fire. Zing took delivery of the remaining shirts and paid 72,000 on custom duty. Zing had sent a bank draft to Raizada & Co. for 2, 50,000 as advance payment. 4,000 shirts were sold by him at 500 each. Expenses incurred by Zing on godown rent and advertisement etc. amounted to 10,000. He is entitled to a commission of 5%. One of the customer to whom the goods were sold on credit could not pay the cost of 25 shirts. Prepare the Consignment Account and the Account of Zing in the books of Raizada & Co. (Zing settled his account immediately. Nothing was recovered from the insurer for the damaged goods). Solution: Consignment A/c Dr Cr Goods sent on Consignment A/c Bank A/c - freight and insurance Zing A/c: [(Custom Duty +Godown Rent, Adv. Etc+ Commission), 18,75,000 57500 1,82,000 Zing A/c (3,975 x 500) Consignment Debtors A/c [Credit Sales (25 x 500)] Abnormal Loss A/c (w 1) 19,87,500 51,316 12,500 1,93,250
  • 59. 58 | P a g e A.Ajanthan Joint Venture & Consignment Accounts (72,000+10,000+100000)] Consignment Debtors A/c - Bad Debts Profit and Loss A/c 12,500 2,67,500 Stock on Consignment A/c(w2) 2,01,250 9,559 23,94,500 23,94,500 Zing A/c Dr Cr Consignment A/c (Sales) 19,87,500 Bank Draft A/c (Advance) Consignment A/c (Expenses and Commission) Bank A/c (Final Settlement) 2,50,000 1,82,000 15,55,500 19,87,500 19,87,500 Abnormal Loss A/c Dr Cr Consignment A/c 1,93,250 Profit & Loss A/c 1,93,250 1,93,250 1,93,250 Working Notes: 1.Valuation of goods Lost-in-transit and unsold Stock: Lkr Total Cost 18, 75,000 Add: Consignor‘s Expenses 57,500 Total Cost of 5,000 Shirts 19, 32,500 Less: Lost-in-transit 1932500/5000*500 1, 93,250 Add: Non-recurring Ex. of Consignee 72,000 Total Cost of 4,500 Shirts 18, 11,250 2. Value of under Stock `18, 11,250 /4500*500 = Lkr 201,250 Note: Since Del Credere Commission is not given by the consignor to the consignee, amount of bad debt is to be charged against Consignment Account.
  • 60. 59 | P a g e A.Ajanthan Joint Venture & Consignment Accounts Illustration: 24 Lubrizols Ltd. of Mumbai consigned 1,000 barrels of lubricant oil costing Liters 800 per barrel to Central Oil Co. of Kolkata on 1.1.2012. Lubrizols Ltd. paid Lkr 50,000 as freight and insurance. 25 barrels were destroyed on 7.1.2012 in transit. The insurance claim was settled at Lkr 15,000 and was paid directly to the consignor. Central Oil took delivery of the consignment on 19.1.2012 and accepted a bill drawn upon them by Lubrizols Ltd., for Lkr 5, 00,000 for 3 months. On 31.3.2012 Central Oil reported as follows: (i) 750 barrels were sold as Lkr 1,200 per barrel. (ii) The other expenses were: LKR Clearing charges 11,250 Godown Rent 10,000 Wages 30,000 Printing, Stationery, Advertisement 20,000 25 barrels of oil were lost due to leakage which is considered to be normal loss. Central Oil Co. is entitled to a commission of 5% on all the sales affected by them. Central Oil Company paid the amount due in respect of the consignment on 31st March itself. Show the Consignment Account, the Account of Central Oil Co., and the Lost –in-Transit Account as they will appear in the books of Lubrizols Ltd. Solution: Consignment to Kolkata Account Dr Cr Goods sent on Consignment A/c Bank A/c - freight and insurance Central Oil Co A/c: [(Freight +Godown Rent+ Wages+ Printing etc), (11250+10,000+30000+20000)] Commissions @5% Profit and Loss A/c 8,00,000 50,000 71,250 45,000 1,31,842 Central Oil Co. A/c (750 x 1200) Abnormal Loss A/c Stock on Consignment A/c 9,00,000 21,250 1,76,842 10,98,092 10,98,092
  • 61. 60 | P a g e A.Ajanthan Joint Venture & Consignment Accounts Central Oil Co. Ltd. A/c Dr Cr Consignment to Kolkata A/c (Sales) 9,00,000 Bills Receivable A/c (advance) Consignment to Kolkata A/c Expenses Commission Bank A/c (amount due) 5,00,000 1,82,000 71,250 45,000 2,83,750 9,00,000 9,00,000 Abnormal Loss A/c Dr Cr Consignment to Kolkata A/c 21,250 Bank-Insurance Claim A/c Profit & Loss A/c 15,000 6,250 21,250 21,250 Workings Notes: 1.Valuation of goods Lost-in-transit and unsold Stock: Lkr Total Cost (1000*800) 8, 00,000 Add: Consignor‘s Expenses 50,000 Total Cost of 1,000 barrels 8, 50,000 Less: Lost-in-transit 850000/1100*25 (21,250) Add: Non-recurring Ex. of Consignee 11,250 Total Cost of 950 barrels 8, 40,000 2. Value of under Stock `840000 /950*200 = Lkr 1, 76, 842 Illustration: 25 Mr. X, the consignor, consigned goods to Mr. Y 100 Radio sets valued Lkr 50,000. This was made by adding 25% on cost. Mr. X paid Lkr 5,000 for freight and insurance. 20 sets are lost – in- transit for which Mr. X received Lkr 5,000 from the Insurance Company. Mr. Y received remaining goods in good condition. He incurred Lkr 4,000 for freight and miscellaneous expenses and Lkr 3,000 for godown rent. He sold 60 sets for Lkr 50,000. Show the necessary ledger account in the books of Mr. X assuming that Mr. Y was entitled to an
  • 62. 61 | P a g e A.Ajanthan Joint Venture & Consignment Accounts ordinary Commission of 10% on sales and 5% Del Credere Commission on sales. He also reported that Lkr 1,000 were proved bad. Prepare the necessary Accounts. Solution: Consignment A/c Dr Cr Goods sent on Consignment A/c Bank A/c - freight and insurance Y A/c: Freight and Misc. Expenses Godown Rent Commission (ordinary) @ 10% Del credere Commission @ 5% Abnormal Loss A/c (Loading) Stock reserve A/c Profit and Loss A/c 50,000 5,000 4,000 3,000 5,000 2,500 2,000 2,000 9,500 Goods Sent on Consignment A/c (Loading) (` 50,000x100/125) Y A/c –sale proceeds Abnormal Loss A/c Stock on Consignment A/c 10,000 50,000 11,000 12,000 83,000 83,000 Y A/c Dr Cr Consignment A/c (Sales) 50,000 Consignment A/c: Expenses Commission (5000+2500) Balance C/d 7,000 7,500 35,500 50,000 50,000 Abnormal Loss A/c Dr Cr Consignment A/c 11,000 Consignment A/c (Loading) Bank-Insurance Claim A/c Profit and Loss A/c (Loss transferred) 2,000 5,000 4,000 11,000 11,000
  • 63. 62 | P a g e A.Ajanthan Joint Venture & Consignment Accounts Working Notes: (1) Calculation of Loading: I.P Load C.P 125 25 100 C.P=100/125*50000 = 40000 Invoice price is Lkr 50000, so Loading is Lkr 10,000(50,000 – 40,000) Loading per set = Lkr 10,000 ÷ 100 = Lkr 100 (2) Valuation of goods lost – in – transit and unsold stock LKR Total invoice price 50,000 Add: consignor‘s expenses (5,000) Invoice price of 100 sets 55,000 Less: lost in transit (55000/100*20) (11,000) 44,000 Add: Non recurring expenses of Mr. Y 4,000 Invoice price of 80 sets 48,000 For unsold stock of (100 – 20 -60) = 20 sets 48000/80*20=12000 (3) Loading on abnormal loss = 20 x Lkr 100 = Lkr 2,000 (4) Stock suspense = 20sets x Lkr 100 = Lkr 2,000 (5) Since Del Credere Commission is given there will not be any entry for bad debts. Illustration: 26 From the following two statements, prepare Consignment A/c and Consignee‘s A/c in the books of Consignor, presuming that the goods were invoiced at 20% above cost. M/s Vijay & Company To: M/s Jyoti Electric House Mumbai Pune No 2355 Proforma Invoice Date: 21st April 2012
  • 64. 63 | P a g e A.Ajanthan Joint Venture & Consignment Accounts Particulars of goods sent on consignment: Amount Lkr Amount Lkr 800 Fans @ Lkr 1680 per fan Add: Expenses Paid: Freight Insurance 6000 Sundries Total E & O E Sign Mumbai For Vijay & Company 4,000 6,000 2,000 13,44,000 12,000 13,56,000 M/s Jyoti Electric House To: M/s Vijay & Company Pune Mumbai (Account sales of 800 fans received from Vijay & Company, Mumbai) Date: 21st September 2012 Sale proceeds of 600 Fans @ Lkr. 2000 per fan Less: Expenses Paid : Advertising Insurance Octroi Commission @10% Total Less: Bill Accepted Less: Bank draft enclosed E & O E Sign Mumbai Jyoti Electric House Amount (LKR) Amount (LKR) 4,500 1,500 12,000 1,20,000 12,00,000 (1,38,000) 10,62,000 7,50,000 3,12,000 Consignment to Pune Account Dr Cr
  • 65. 64 | P a g e A.Ajanthan Joint Venture & Consignment Accounts Goods sent on Consignment A/c Bank A/c - freight and insurance & Sundries M/s Jyoti Electric House‘s A/c: Expenses Commission Stock reserve A/c (loading on stock) Profit and Loss A/c 13,44,000 12,000 18,000 1,20,000 56,000 2,16,000 M/s Jyoti Electric House‘s A/c (sale proceeds) Sent on Consignment A/c 12,00,000 2,24,000 3,42,000 17,66,000 17,66,000 Jyoti Electric House’s A/c Dr Cr Consignment A/c (Sales) 12,00,000 Consignment A/c: Expenses Commission Bank A/c Bills Receivable A/c 18,000 1,20,000 3,12,000 7,50,000 12,00,000 12,00,000 Workings Notes: Loading on consignment Lkr Invoice price of fans consigned Loading is 20% on cost Thus loading to be removed 20/120 × 1680 Total loading removed (800 × 280) Value of closing Stock Original invoice value Consignor‘s expenses Consignee‘s non-recurring expenses (Octroi only) Loading on consignment Total fans sent Fans sold 1,680 280 2,24,000 13,44,000 12,000 12,000 13,68,000 800 600
  • 66. 65 | P a g e A.Ajanthan Joint Venture & Consignment Accounts In Stock Hence, stock value (13,68,000/800 × 200) Loading to be removed (200 × 280) 200 3,42,000 56,000 Illustration: 27 On 1.7.2012, Mantu of Chennai consigned goods of the value of Lkr 50,000 to Pandey of Patna. This made by adding 25% on cost. Mantu paid that on Lkr 2,500 for freight and Lkr 1,500 for insurance. During transit1/10 th of the goods was totally destroyed by fire and a sum of Lkr 2,400 was realized from the insurance company. On arrival of the goods, Pandey paid Lkr 1,800 as carriage to godown. During the year ended 30th June 2013, Pandey paid Lkr 3,600 for godown rent and Lkr 1,900 for selling expenses.1/9 th of the remaining goods was again destroyed by fire in godown and nothing was received from the insurance company. On 1.6.2013, Pandey sold half (1/2) the original goods for Lkr 30,000 and changed a commission of 5% on sales. As on 30.6.2013, Pandey sent a bank draft to Mantu for the amount so far due from him. You are required to prepare the following ledger accounts in the books of Mantu of Chennai for the year ended 30.6.2013. (a) Consignment to Patna Account; (b) Goods Destroyed by Fire Account; and (c) Personal Account of Pandey. Solution: Consignment to Patna Account Dr Cr Goods sent on Consignment A/c Bank A/c - freight and insurance (2500+1500) Pandey A/c : Carriage Inward 1800 Godown Rent 3600 Selling Expenses 1900 Commission (5% on Lkr 30,000) Goods Destroyed by Fire A/c : Loading Stock reserve A/c (Loading on unsold stock) 50,000 4,000 7300 1,500 2000 3000 Goods Sent on Consignment A/c Pandey A/c : Sale Proceeds Goods Destroyed by Fire A/c Stock on Consignment A/c 10,000 30,000 11,000 16,800
  • 67. 66 | P a g e A.Ajanthan Joint Venture & Consignment Accounts 17,66,000 17,66,000 Note: There is no normal Profit or Loss on Consignment. Goods Destroyed by Fire A/c Dr Cr Consignment to Patna A/c : In transit In Godown 5,400 5,600 Consignment to Patna A/c : Loading Bank-Insurance Claim A/c Profit and Loss A/c 2,000 2,400 6,600 11,000 11,000 Pandey Account Dr Cr Consignment to Patna A/c (Sale proceeds) 30,000 Consignment A/c: Expenses Commission Draft A/c 7,300 1,500 21,200 30,000 30,000 Working Notes: Valuation of goods destroyed by fire and unsold stock Total Insurance Claim Add: Consignor‘s Expenses Less: Lost-in-transit ( 1/10 x lkr 54,000) Goods received (9/10 th of lkr 54,000) Add: Non- recurring expenses of Pandey Less: Value of goods destroyed by fire in godown ( 1/9 th of lkr 50,400) Value of 8/10 th 50,000 4,000 54000 (5400) 48,600 1,800 50,400 (5,600) 44,800
  • 68. 67 | P a g e A.Ajanthan Joint Venture & Consignment Accounts ∴ Value of unsold stock 9/10 - (1/9 th of 9/10) =9/10 -1/10 =8/10 Goods sold ½ i.e., = 8/10 – 1/2= 3/10 th ∴ Value of unsold stock = Lkr 44,800 x 3/10 x 10/8 = Lkr 16,800 Loading on goods destroyed = ` 10,000 x 2/10 = Lkr 2,000 Loading on unsold stock = ` 10,000 x 3/10 = Lkr 3,000. Illustration: 28 Usha sent goods costing Lkr 75, 50,000 on consignment basis to Gayatri on 1st Feb 2012 @ 8.5% commission. Usha spent Lkr 8, 25,000 on transportation. Gayatri spent Lkr 5, 25,000 on unloading. Gayatri sold 88% of the goods for Lkr 90, 00,000, 10% of the goods for Lkr 10, 00,000 and the balance is taken over by her at 10% below the cost price. She sent a cheque to Usha for the amount due after deducting commission. Show Consignment to Gayatri A/c and Gayatri‘s A/c in the books of Usha. Solution: Calculation of sales Cost (Lkr) Invoice (Lkr) Goods sent 75, 50,000 88% of the goods 66, 44,000 90, 00,000 10% of goods 7, 55,000 10, 00,000 Total sales 73, 99,000 1, 00, 00,000 Goods taken over by Gayatri 1, 51,000 1, 35,900 There is no closing stock here as all unsold goods were taken over by Gayatri. The commission is payable only on sales to outsiders and not on goods taken over by Gayatri. Thus, commission is 8.5% on Lkr 10,000,000 i.e. Lkr 8, 50,000 The required ledger accounts are shown below. Consignment to Gayatri A/c Dr Cr
  • 69. 68 | P a g e A.Ajanthan Joint Venture & Consignment Accounts Goods sent on Consignment A/c Bank A/c - transportation) Gayatri‘s A/c: Unloading charges 5,25,000 Commission 8,50,000 To P & L A/c 75,50,000 8,25,000 13,75,000 3,85,900 Gayatri‘s A/c (sales) Gayatri‘s A/c (goods taken over) 10,000,000 1,35,900 1,01,35,900 1,01,35,900 Gayatri’s A/c Dr Cr Consignment A/c (Sale proceeds) 1,01,35,900 Consignment A/c: Expenses Commission Bank A/c 5,25,000 8,50,000 87,60,900 1,01,35,900 1,01,35,900 Calculation of Abnormal Loss: 250 kg of oil lost in transit Cost of 250 kg @ 40/kg 10,000 Proportionate expenses of Babubhai (250/10000*7500) 188 10,188 Calculation of closing stock Kg Oil consigned to Delhi Less: Lost in transit Less: Normal loss due to leakage Less: Quantity sold Stock in hand Basic cost of stock consigned @ ` 40 Less : Cost of abnormal loss Cost of stock after normal loss of 100kg Thus cost of 2150 kg (389812/9650*2150) 10,000 (250) (100) (7,500) 2,150 400,000 (10,188) 389,812 86,849
  • 70. 69 | P a g e A.Ajanthan Joint Venture & Consignment Accounts Calculation of commission Ordinary @ 3% on 450000 Del Credre @ 1.5% on 450000 13,500 6,750 20,250 As the consignee has paid Del Credre Commission, the responsibility of bad debts is his. Hence no entry is needed to be passed in the books of consignor. Illustration: 29 Sangita Machine Corporation sent 200 sewing machines to Rita agencies. It spent Lkr 7500 on packing. The cost of each machine was Lkr 2,000, but it was invoiced at 20% above cost. 20 machines were lost in transit & insurance company accepted claim of Lkr 20,000 only. Rita agencies paid freight of Lkr 9,000, carriage Lkr 3,600, and Octroi Lkr 1,800 and rent Lkr 1800. They sold 150 machines at Lkr 3,500 per machine. They were entitled to commission of 5% on invoice price and additional 20% of any excess realized on invoice price and 2% Del Credre commission. They accepted a bill drawn by Sangita Machine Corporation for Lkr 3, 00,000 and remitted the balance by demand draft along with account sale. Draw up necessary ledger accounts in the books of Sangita Machine Corporation and Rita Agencies. Solution: Consignment to Rita Agencies A/c Dr Cr Goods sent on Consignment A/c Bank A/c - (Packing Expenses) Rita Agencies A/c: Freight 9,000 Carriage 3,600 Octroi 1,800 Rent 1,800 Commission 61,500 Abnormal loss A/c (Load removed) Stock Reserve A/c P & L A/c 4,80,000 7,500 77,700 8,000 12,000 1,44,075 Rita Agencies‘ A/c (Sales) (sales 150 @ 3500) Abnormal Loss A/c Consignment Stock A/c Goods Sent on Consignment A/c (Loading) 5,25,000 48,750 75,525 80,000 7,29,275 7,29,275
  • 71. 70 | P a g e A.Ajanthan Joint Venture & Consignment Accounts Rita Agencies A/c Dr Cr Consignment A/c (Sale proceeds) 5,25,000 Consignment A/c: Expenses Commission Bills Receivable A/c Bank A/c 16,200 61,500 3,00,000 1,47,300 5,25,000 5,25,000 Calculation of abnormal loss 20 machines lost in transit: Cost of 20 machines @ Lkr 2400 48,000 Proportionate expenses of Babubhai 750 (20/200*7500) 48750 Calculation of Closing Stock: LKR Invoice value of 30 machines @ 2400 72,000 Add: Consignor‘s proportionate expenses 1,125 Add: Consignee‘s proportionate expenses 2,400 75,525 Stock reserve 30 machines @ Lkr 400 12,000 Calculation of Commission: Invoice price of machines sold (2400*150) 360,000 Commission @ 5% on this 18,000 (a) Excess over invoice value (525000-360000) 165,000 Commission @ 20% on this 33,000 (b) Del Credre Commission @ 2% on 525000 10,500 (c) Total Commission (a+b+c) 61,500
  • 72. 71 | P a g e A.Ajanthan Joint Venture & Consignment Accounts Books of Rita Agencies Sangita Machine Corporation A/c Dr Cr Cash A/c (expenses) Commission A/c Bills Payable A/c Bank A/c 16,200 61,500 3,00,000 1,47,300 Consignment A/c (sales) 5,25,000 5,25,000 5,25,000 Illustration: 30 Ram of Patna consigns to Shyam of Delhi for sale at invoice price or over. Shyam is entitled to a commission @ 5% on invoice price and 25% of any surplus price realized. Ram draws on Shyam at 90 days sight for 80% of the invoice price as security money. Shyam remits the balance of proceeds after sales, deducting his commission by sight draft. Goods consigned by Ram to Shyam costing Lkr 20,900 including freight and were invoiced at Lkr 28,400. Sales made by Shyam were Lkr 26,760 and goods in his hand unsold at 31st Dec, represented an invoice price of Lkr 6,920. (Original cost including freight Lkr 5,220). Sight draft received by Ram from Shyam up to 31st Dec was Lkr 6,280. Others were in- transit. Prepare necessary Ledger Accounts. Solution: Consignment to Delhi Account Dr Cr Goods sent on Consignment A/c Delhi A/c- Commission Stock Reserve A/c `(6,920 – 5,220) P & L A/c 28,400 2,394 1,700 8,686 Shyam A/c – Sale proceeds (sales 150 @ 3500) Goods Sent on Consignment A/c (Loading) (28,400- 20,900) Consignment Stock A/c 26,760 7,500 6,920 41,180 41,180 Shyam Account Dr Cr Consignment to Delhi A/c (Sale proceeds) Balance c/d (` 6,920 x 80%) 26,760 5,536 Bills Receivable A/c Consignment to Delhi A/c - commission 22,720 2,394